Вы находитесь на странице: 1из 157

Applications

Translations
Translations
Applications
Photos

Video

Groups

Events

Notes

Chat - Rooms

Open Facebook Chat


Pop Out Chat
Notifications
Notifications
Chat (Offline)
Chat
Friend Lists
Options

Home
Profile
Friends
Recently Added
All Friends
Invite Friends
Find Friends
Inbox79
View Message Inbox (79)
Compose New Message

Logout
Settings
Account Settings
Privacy Settings
Application Settings
Maher Fawzy

Back to professional medical guide

Discussion Board

Topic View
Start New Topic

Topic:
Reply to Topic
Displaying posts 1 - 30 out of 30 by 4 people.

Post #1
1 reply
Hala Adel wrote6 hours ago
MRCS Part 1 Practice Questions ( Physiology ) - 1 of 3
Correct
ADH (Vasopressin) release in response to dehydration causes
Single best answer question choose ONE true option only
Decreased permeability of the collecting ducts to water f

Decreased urine osmolality f

Increased Na+ resorption in the ascending limb of the loop of Henle f

Increased Na+ resorption in the descending limb of the loop of Henle f

Increased permeability of the collecting ducts to water t


Your answer
ADH is released by the posterior pituitary in response to dehydration, from stimulation of
osmoreceptors adjacent to the supraoptic nucleus, as well as volume receptors in the aorta atria
and great veins. Water absorption in the collecting ducts is independent of sodium
concentration, and is under the control of ADH, which causes increased permeability of the

ducts. Increased ADH levels will increase the osmolality of the urine via this method.

In the descending limb of the loop of Henle, sodium and water are passively resorbed. The
ascending limb is impermeable to water, with active sodium resorption, producing a
concentration gradient in the renal medulla, which is essential for the maintenance of water
balance.
Correct
A woman, aged 55, presents with features consistent with Cushings syndrome. She is taking no
medication. Her basal cortisol and plasma ACTH levels are significantly raised. She has failed the
dexamethasone suppression test.

What is the most likely diagnosis? Single best answer question choose ONE true option only

Adrenal tumour

Carneys syndrome

Cushings disease
Your answer
Depression

Ectopic ACTH-secreting tumour

A raised adrenocorticotrophic hormone (ACTH) level with a raised cortisol implies the problem is
caused by excess ACTH production, otherwise negative feedback would suppress ACTH. A low
ACTH level would therefore be expected in patients with an adrenal tumour. Carneys syndrome
comprises atrial myxoma and freckles with high cortisol levels independent of ACTH. Pituitary
tumours producing ACTH and adrenal stimulation are the cause of Cushings disease. While
ectopic ACTH-secreting tumours are associated with significantly raised ACTH and cortisol levels,
its unusual to develop classic cushingoid features. Failure of the dexamethasone suppression
test can occur in patients with depression, but cushingoid features are not expected.
Correct

Babinski's sign is produced by:


Lateral cerebral sulcus lesions
Cerebellar lesions
Basal ganglia lesions
Lesions of vestibular nuclei
Lesions of the pyramidal tracts Your answer

Diseases of the pyramidal system cause upper motor neurone lesions. The normal flexor plantar
response becomes extensor (a positive Babinskis sign).
Correct
A 61-year-old woman with known COPD is about to have a laparoscopic cholecystectomy. Having
established a pneumonperitoneum the anaesthetist informs you that he is having difficulty
oxygenating the patient. Which of the following factors is most likely responsible for this
difficulty?
Single best answer question choose ONE true option only
Abdominal aorta compression

Increased FEV1 : FVC ratio

Increased peak airway pressures


Your answer
Increased respiratory rate

Reduced venous return

Laparoscopic surgery is performed through a transperitoneal or retroperitoneal approach with


insufflation of CO2 under pressure to create a working space. Cardiovascular, respiratory, renal,
and metabolic changes occur secondary to the raised intra-abdominal pressure (IAP) and
absorption of CO2.

In reference to the question the raised IAP associated with CO2 insufflation pushes the
diaphragm cephalad, reducing diaphragmatic movements. Functional residual capacity, vital
capacity, and pulmonary compliance all decrease with raised IAP and peak airway pressures may
increase by >50%. These changes are normally well tolerated but patients with underlying lung
disease have poor lung compliance and are unable to compensate. These patients often require
positive end expiratory pressure in order to achieve adequate gas exchange.
Incorrect
The infusion of 1 litre of which of the following solutions will initially lead to the greatest increase
in extracellular fluid volume?

Single best answer question choose ONE true option only


Gelatin colloid solution (e.g. Gelofusin or Haemaccel)
Your answer
Hypertonic NaCl
Correct answer
Normal (0.9 %) NaCl

5 % dextrose solution

Pure water

Colloids may be natural (e.g. blood, human albumin and gelatins) or synthetic (e.g. dextrans).
They comprise large branching molecules with molecular weights in excess of 30,000. Assuming
intact capillary integrity, the volume effects of colloid infusion are, at least initially, confined to
the plasma compartment. In contrast, crystalloids, such as NaCl solution, pass more readily from
the plasma fluid compartment and have more of a volume effect on the extracellular fluid
compartment. In the case of 5 % dextrose solution, the dextrose component is rapidly
metabolised and the remaining water distributes itself throughout the entire body water (i.e.
intracellular and extracellular compartments).

Therefore, of the options listed above, infusions of NaCl will have the greatest initial increase in
extracellular fluid volume. Hypertonic NaCl will have an even greater effect than normal
(approximately isotonic) NaCl, since hypertonic solutions will draw additional water from the
intracellular fluid compartment by osmosis.
Correct
In a lung function test, the functional residual capacity:

Single best answer question choose ONE true option only


Is the sum of the tidal volume and residual volume

Is the sum of the inspiratory reserve volume, the expiratory reserve volume and the tidal volume

Can be measured directly by spirometry

Is equal to the sum of the residual volume and the expiratory reserve volume
Your answer
Is that volume of air that remains in the lung after forced expiration

Spirometry traces are easy to understand if you remember the following two rules:

1. There are 4 lung volumes and 5 capacities that you need to remember.
2. A capacity is made up of 2 or more lung volumes

The 4 lung volumes are:


* Tidal volume = volume of air inspired or expired with each normal breath in quiet breathing;
approximately 500mls.
* Residual volume = that volume of air that remains in the lung after forced expiration.
* Inspiratory reserve volume = extra volume of air that can be inspired over and above the
normal tidal volume.
* Expiratory reserve volume = extra volume of air that can be expired by forceful expiration after

the end of a normal tidal expiration.

The 5 lung capacities are:


* Functional residual capacity = that volume of air that remains in the lung at the end of quiet
expiration. Equal to the sum of the residual volume and the expiratory reserve volume.
* Inspiratory capacity = inspiratory reserve volume + tidal volume
* Expiratory capacity = expiratory reserve volume + tidal volume
* Vital capacity = inspiratory reserve volume + tidal volume + expiratory reserve volume (or
total lung capacity residual volume)
* Total lung capacity = vital capacity + residual volume

The residual volume (and therefore functional residual capacity and total lung capacity) cannot
be measured directly by spirometry. They are measured by either whole body plethysmography,
or by using the helium dilution or nitrogen washout techniques.
Correct
Gluconeogenesis is best described as a process by which?

Single best answer question choose ONE true option only


Glucose is generated from carbohydrate precursors
Glucose is generated by the breakdown of glycogen stores

Glucose is generated from non carbohydrate sources


Your answer
Glucose is broken down to Acetyl CoA which enters the tri-carboxylic cycle

Glucagon is generated from carbohydrate precursors


Glucose is an essential source of nutrition for the central nervous system and red blood cells.
Glycogenolysis (the breakdown of glycogen stores to glucose) will maintain glucose levels for
around 8-12 hours after which gluconeogensis will takeover. The main substrates for
gluconeogensis include lactate (produced as the result of anaerobic respiration), glycerol
(derived from the breakdown of fat) and amino acids (derived from the breakdown of protein).
Correct
A 70kg woman receives approximately 27% full thickness burns in a house fire to her chest and
left arm circumferentially. How much fluid does she require over the initial 24hrs?
Single best answer question - choose ONE true option only
1280 ml

2870 ml

3000 ml

5320 ml

7560 ml
Your answer
The required fluids may be calculated by the following-

2-4 mls fluid per kg body weight per percent body surface area burns over 24hrs.

The rule of nines is a useful method used estimate the total body surface area (BSA) burns.
The adult BSA is divided up into areas of 9% (or multiples of 9%)-

Head, face, arms all equal 9% BSA


Chest, back, legs all equal 18% BSA

Thus, this patient BSA burned is 27% (arm 9% and chest 18%)

4 (ml fluid) x 70 (wt in kg) = 280 mls

280mls x 27 = 7560mls fluid requirement.

It must be stressed that the rule of nines only applies to adults as in children the head
represents a proportionally larger area. A useful estimation that can be used for any patient is
that the palmer surface of the patients hand (including the fingers) represents approximately 1%
BSA.
Incorrect
In a starving patient, which of the following fluid regimens would be most appropriate for a 70kg
man over a 24hr period?

Single best answer question choose ONE true option only


3L NSaline with 20mmols potassium chloride in each bag
3L Dextrose-saline

3L Hartmanns solution
Your answer
1L NSaline with 20 mmols potassium chloride and, 2L 5% dextrose with 20mmols potassium
chloride in each bag
Correct answer
3L 5% dextrose with 20mmols potassium chloride in each bag
The daily fluid and electrolyte requirements are 1-1.5 mmols Na+ /Kg/24 hours, 1mmols K+
/Kg/24 hours and 40ml H20 /Kg/24 hours.
However, additional fluid should be supplemented if there are 3rd space losses (that commonly

occur for instance in severe acute pancreatitis, burns and post major gastro-intestinal surgery)
and for other sources of fluid loss including vomiting, diuresis and insensible losses
Correct
Which of the following statements regarding the flow of air through the airways of the lung is
correct?

Single best answer question choose ONE true option only


Flow rate is proportional to the length of the airway

Flow rate is proportional to the cube of the radius of the airway

Flow rate is proportional to the viscosity of the gas passing along the airway

Flow rate is inversely proportional to the pressure gradient along the airway

None of the above


Your answer
This question tests knowledge and physiological application of Poiseuilles Law which states that
for a rigid, wide bore tube:-

Fluid flow rate = pr4(DP)


8hL

where: r = radius of the tube, DP is the pressure gradient along the tube, h is the viscosity of the
fluid running through the tube and L is the length of the tube.

Therefore, the flow rate is proportional to the fourth power of the radius and the pressure
gradient along the tube, but is inversely proportional to the viscosity of the fluid and the length
of the tube
Incorrect
A 77-year-old man presents with a history of vomiting undigested food. Routine biochemistry
shows a serum bicarbonate concentration of 38 mmol/l.

Which of the following findings would most suggest that he had a chronic metabolic alkalosis?
Single best answer question choose ONE true option only
Alkaline urine

Base excess 18 mmol/l

Elevated arterial p(CO2)


Correct answer
Hypokalaemia
Your answer
Hypomagnesaemia
The base excess provides no additional information: it is directly related to the high bicarbonate
concentration. In prolonged metabolic alkalosis, the urine may become acidic, reflecting
increased proximal bicarbonate resorption (a consequence of hypochloraemia). Gastric
secretions contain about 10 mmol/l potassium and, although potassium depletion is likely to
become more severe the longer vomiting occurs, hypokalaemia can develop at any time.
However, the development of hypercapnoea as compensation for metabolic alkalosis tends to
take some time. Although alkalosis inhibits respiration, the tendency for p(CO2) to increase acts
as a respiratory stimulant, though with time, the sensitivity of the respiratory centre to carbon
dioxide may decline so that significant hypoventilation does occur. Hypomagnesaemia is
frequently found in patients with potassium depletion
Correct
Which of the following is not associated with a metabolic acidosis?

Single best answer question choose ONE true option only


A fall in serum bicarbonate

Ketosis

Hypokalemia
Your answer
Hypovolaemic shock

Hyperventilation

Serum potassium levels are intimately linked with serum H+ levels via the sodium potassium
ATPase. This cell membrane pump principally exchanges intracellular sodium ions with
extracellular potassium ions in order to maintain the cell membrane potential. However,
potassium ions compete with hydrogen ions in the exchange pump and therefore in the
presence of hypokalemia, more hydrogen ions will move into the intracellular compartment via
this pump. Conversely, in the presence of hyperkalaemia, less hydrogen ions will move out of
the extracellular compartment which result in a metabolic acidosis.
Incorrect

A 54-year-old woman has undergone some blood tests as part of an employment health screen.
She reports she is in good health and, being very health conscious, takes regular vitamin and
mineral supplements. She is taking bendrofluazide 2.5 mg for hypertension and her blood
pressure is 132/82 mmHg. The only abnormality is a serum calcium concentration of 2.94
mmol/l.

Which of the following is the most likely cause? Single best answer question choose ONE true
option only

Diuretic treatment
Your answer
High dietary calcium intake

High dietary vitamin D intake

Occult malignancy

Primary hyperparathyroidism
Correct answer
Thiazides can cause hypercalcaemia but it is usually only mild. Vitamin D itself is physiologically
inactive and, whereas 1-hydroxylated derivatives can be a cause of hypercalcaemia, vitamin D
which has to be metabolised to activate it is less commonly so. Intestinal absorption of calcium
is subject to tight control, and a high intake does not cause hypercalcaemia. The two most
common causes of hypercalcaemia are primary hyperparathyroidism and malignancy. In an
asymptomatic individual, primary hyperparathyroidism is the more likely cause
Incorrect
Which ECG feature is classically present in hypothermia?
Single best answer question choose ONE true option only
Thyroxine

Reduced PR interval

Tachycardia
Your answer
U waves

J waves
Correct answer
The J wave may be present on the ECG in patients with hypothermia and is an additional upward
peak immediately following the QRS complex. The U wave may be present on the ECG in
hypokalaemia and is an additional upward peak which follows the T wave. Tachycardia and a
reduction in the RR interval are ECG features of hyperthermia.
Incorrect
A patient undergoes respiratory function tests. Which of the following are normal readings for a
70-kg man?

Single best answer question choose ONE true option only

Peak expiratory flow of 376 l/min


Your answer
Total lung capacity of 3.5 litres

Functional residual capacity of 3.5 litres

Tidal volume of 250 ml

Inspiratory reserve volume of 2 litres


Correct answer
Normal readings for such a patient would be:

peak expiratory flow


520700 l/min
total lung capacity
56.5 litres
functional residual capacity
23 litres

tidal volume
500700 ml
Correct
What is the half life of free triiodothyronine (T3) in the blood?
Single best answer question choose ONE true option only
1 minute

1 hour

1 day
Your answer
1 week

1 month

Most of the T3 and thyroxine (T4) are carried in plasma bound to thyroxine binding globulin, and
are inactive in this state. Only 1% of T3 and 0.05% of T4 is free. T3is the active hormone, and is
formed from the intracellular deiodination of T4 by type 2 deiodinase. The half life of T4 is 1
week, and of T3 1 day, suggesting that T4 acts as a source of T3, rather than an active hormone
in its own right
Correct
Cerebellar lesions produce:
Waddling gait
Festinant gait
Ataxic gait Your answer
Scissors gait
High-stepping gait
In disease of the lateral cerebellar lobes, the stance becomes broad based, unstable and
tremulous. The gait tends to veer towards the side of the more affected cerebellar lobe.
Weakness of proximal lower limb muscles (eg in polymyositis or muscular dystrophy) leads to
difficulty in rising from sitting or squatting. Once upright, the patient walks with a waddling gait,
as each lower limb, as it carries the full weight of the body, does not adequately support the
pelvis. Festinant, or hurried gait occurs in Parkinsons disease. Broad-based, high stepping or
stamping gait develops in peripheral sensory lesions (eg polyneuropathy) when there is loss of
proprioception. Spasticity causes stiffness and jerkiness while walking scissors gait.
Correct
Which one of the following hormones is secreted by the anterior pituitary?

Single best answer question choose ONE true option only


Testosterone

Oxytocin

TSH

Your answer
CRH

ADH

The pituitary gland (hypophysis) is the conductor of the endocrine orchestra. It is divided into
both an anterior part and posterior part. The anterior pituitary (adenohypophysis or pars distalis)
secretes 6 hormones namely:

FSH/LH: Reproduction
ACTH: Stress response
TSH: Basal metabolic rate
GH: Growth
Prolactin: Lactation

The posterior pituitary (neurohypophysis or pars nervosa) secretes only 2 hormones:

ADH (vasopressin): Osmotic regulation


Oxytocin: Milk ejection and labour

Testosterone is produced from Leydig cells in the testis and from the adrenal glands. CRH is
produced by the median eminence of the hypothalamus
Correct
Which of the following systemic effects are most likely to be caused by a space occupying lesion
in the brain?
Single best answer question choose ONE true option only
Bradycardia
Your answer
Hypotension

Tachycardia

Tachypnoea

Venous ulceration

The cranium is a fixed volume containing blood, CSF and brain tissue in equilibrium. Increases in

one component can be compensated by a decrease in the other components without increasing
intracranial pressure (the Monroe-Kellie doctrine). Beyond a certain point, this compensation is
insufficient, and raised intracranial pressure results (greater than 10-15mmHg).

The effects of raised intracranial pressure are hydrocephalus, cerebral ischaemia (due to
decreased cerebral perfusion pressure) and systemic effects. The systemic effects include
hypertension, bradycardia, slowed respiration and gastric ulceration (Cushings ulcer). These are
thought to be due to autonomic dysregulation resulting from hypothalamic compression.
Incorrect
Which of the following biochemical parameters would not be useful in distinguishing haemolysis
from haemorrhage in an anaemic patient?
Single best answer question choose ONE correct option only
Serum ferritin Correct answer
Serum haptoglobin
Serum LDH
Bilirubin
Serum iron Your answer
With haemolysis, iron is recycled by combining with serum haptoglobin which falls as a result.
Patients with haemolytic states do not therefore become iron deficient, unlike patients who are
bleeding who lose on average 1mg of iron with every mL of blood. Unconjugated bilirubin is
marker of haemolyis and is generated by the breakdown of the Haem ring from haemoglobin. In
addition, LDH is released from red blood cells if haemolysis is intravascular.
Incorrect
Which of the following physiological characteristics relates to the lining of the respiratory tract?
Single best answer question choose ONE true option only
About 1 litre of mucus is produced every day

The cilia are under the control of a physiological motor, dynein


Correct answer
The mucociliary escalator moves at 0.2 cm/minute

The bronchioles have cartilage in their wall

The bronchioles have diameters up to 5 mm


Your answer
About 100 ml of mucus is produced every day. The cilia are under the control of a physiological
motor, dynein (which is absent in Kartageners syndrome). The mucociliary escalator moves at 2
cm/minute. The bronchioles do not have cartilage in their wall (which distinguishes them from
bronchi). The bronchioles can be up to 1 mm in diameter
Correct
The Chief cells of the stomach produce which of the following substances?
Single best answer question choose ONE true option only
Gastric acid

Intrinsic factor
Pepsinogen Your answer
Mucus
Somatostatin
Chief cells produce pepsinogen which is a precursor and is activated to pepsin by gastric acid.
Pepsin digests protein. Gastric parietal cells produce gastric acid hydrochloric acid. Intrinsic
factor is also produced by parietal cells and is necessary for vitamin B12 absorption in the
terminal ileum. Mucus cells produce mucus which forms a protective layer over the gastric
mucosa preventing autodigestion.
Incorrect
Hypothyroidism due to disease of the thyroid gland is associated with increased plasma level of?
Single best answer question choose ONE true option only
Cholesterol
Correct answer
Albumin

RT3

Iodide

Thyroid binding globulin (TBG)


Your answer
Thyroid hormone lowers circulating cholesterol level. The plasma cholesterol level drops before
the metabolic rate rises
Correct
Regarding the clinical physiology of the adrenal gland in Cushings disease, which of the
following pertains? Single best answer question choose ONE true option only

The zona glomerulosa of the cortex is predominantly responsible for sex steroid production

The zona fasciculata is predominantly controlled by ACTH and is often hypertrophied


A 24-year-old woman undergoes resection of the terminal ileum with fashioning of an ileostomy
for Crohns disease. Some 2 weeks after surgery, she is making a good recovery, and is eating a
high-energy, low-residue diet, but has a high ileostomy volume, necessitating intravenous fluid
replacement. Her serum calcium concentration is 1.82 mmol/l, phosphate 1.28 mmol/l, alkal
Reply to HalaReport

Post #2
Hala Adel wrote6 hours ago
Your answer

The zona reticularis is predominantly responsible for mineralocorticoid production

About 15% of glucocorticoid production takes place in the adrenal medulla

The zona fasciculata is primarily responsible for mineralocorticoid production

The zona glomerulosa of the cortex is predominantly responsible for mineralocorticoid


production, the zona fasciculata for glucocorticoid production and the zona reticularis for sex
corticoid production. The adrenal medulla originates from the neural crest and hence there is
almost complete demarcation of function, with the medulla being responsible for the production
of catecholamine-related compounds
Incorrect
Which of the following metabolic effects is most likely to be caused by thyroid hormone?
Single best answer question choose ONE true option only
Decreased glycogenolysis in the liver

Increased glucose absorption in the gut


Correct answer
Decreased lipolysis

Decreased expression of adrenergic receptors


Your answer
Decreased oxygen uptake in the mitochondria

Thyroid hormone has widespread metabolic effects.


Increased glycogenolysis in the liver, increased glucose absorption in the gut and increased
insulin breakdown all tend to increase blood glucose. The glycogenolytic effects of
catecholamines are also potentiated. These effects can make the diagnosis and management of
diabetes in thyrotoxicosis difficult.
There is an overall lipolytic effect, with decreased serum cholesterol seen in thyrotoxicosis, and
an increase in hypothyroidism.
There is an increased expression of b-adrenergic receptors in many tissues including skeletal and
cardiac muscle. There is a positive inotropic effect with increased cardiac output and heart rate.
A raised metabolic rate and increased heat production are due to increased oxygen uptake and
ATP production in the mitochondria.
There are also effects on bone, with an overall breakdown of bone, sometimes leading to
hypercalcaemia. Increased serum 2,3 DPG leads to a right shift of the haemoglobin dissociation
curve. Thyroid hormones are also essential for fetal development, with deficiency leading to
cretinism. The fetus produces its own hormone from 18 weeks of gestation.
Correct
A patient in the intensive care unit following liver transplant surgery has a metabolic alkalosis.

Which of the following biochemical abnormalities is MOST specifically indicative of this? Single
best answer question - choose ONE true option only

Acidic urine

High arterial blood pH (low hydrogen-ion concentration)

High arterial partial pressure of carbon dioxide p(CO2)

High plasma bicarbonate concentration


Your answer
Hypochloraemia

Arterial pH is increased in both metabolic and respiratory alkalosis: plasma bicarbonate is always
increased in metabolic alkalosis and can be low in chronic respiratory alkalosis. A high p(CO2)
can occur in metabolic alkalosis as a result of respiratory compensation, but it is also a feature of
respiratory acidosis. Although the urine may become paradoxically acidic in metabolic alkalosis,
it is normally acidic, except sometimes immediately following a meal. Hypochloraemia is present
in metabolic alkalosis due to a loss of gastric acid, but may not occur with alkalosis from other
causes.
Correct
Which of the following is NOT a characteristic of the loop of Henle?
Single best answer question choose ONE true option only
Is under the control of aldosterone
Is permeable to water and electrolytes along its distal limb
Proximal limb absorption is isotonic
Generates high osmolality in the renal medulla
Is impermeable to water along its distal limb Your answer
The loop of Henles main function is to produce a high medullary osmolality which is the driving
force for water reabsorption from the collecting ducts. In the loop of Henle there is a
concentration and reduction in volume of filtrate as sodium and chloride pass into the
descending limb and water is osmotically moved out. In the ascending limb there is active
reabsorption of sodium chloride from the filtrate producing a low osmolality filtrate. Aldosterone
acts on the distal convoluted tubules and collecting ducts.
Correct
Which of the following hormones is synthesised in the hypothalamus and secreted from the
posterior pituitary?
Single best answer question choose ONE true option only
Anti diuretic hormone (ADH)
Your answer
Adrenocorticotrophic hormone (ACTH)

Corticotrophin releasing hormone (CRH)

Thyrotrophin releasing hormone (TRH)

Thyroid stimulating hormone (TSH)

Vasopressin (ADH) and oxytocin are synthesised in the hypothalamic nuclei and pass down
axons to the posterior pituitary where they are secreted into the blood stream.

In contrast, the trophic hormones such as CRH and TRH are secreted by the hypothalamus in
response to neural stimuli, and drain into the hypothalamohypophyseal portal vessels to the
anterior pituitary. There is then resultant stimulation of ACTH and TSH secretion. The other
hormones produced by a similar mechanism by the anterior pituitary are growth hormone (GH),
prolactin (PRL), lutenising hormone (LH) and follicle stimulating hormone (FSH).
Incorrect
A 21-year-old male medical student who has been feeling non-specifically unwell for several
days is noticed to have slightly icteric sclerae by his girlfriend and has liver function tests
performed. The results of these are normal apart from a serum bilirubin concentration of 44
mmol/l (317). His urine does not contain bilirubin.

Which of the following is the most likely diagnosis? Single best answer question choose ONE
true option only

DubinJohnson syndrome

Gilberts syndrome
Correct answer
Hereditary spherocytosis

Infectious mononucleosis
Your answer
Rotor syndrome

DubinJohnson, Rotor and Gilberts syndromes are all inherited disorders of bilirubin metabolism.
However, in the first two, there is a defect in the secretion of bilirubin from the liver and the
bilirubin that accumulates in the plasma is conjugated, water-soluble and thus is excreted in the
urine.
Infectious mononucleosis can cause hepatitis and jaundice but an elevated transaminase activity
would be expected. Hereditary spherocytosis is a chronic haemolytic disorder due to a defect in
the red cell membrane (most frequently in spectrin, a structural protein). It can present with a
wide range of severity, from jaundice at birth to asymptomatic anaemia or jaundice in adults,
but is much less common (approximately 1:5000 in Northern Europeans) than Gilberts
syndrome (approximately 1:20).
Correct
Which of the following is NOT a defining feature of the systemic inflammatory response
syndrome (SIRS)?

Single best answer question choose ONE true option only


Temperature >37.5 oC
Your answer
Heart rate >90/min

Respiratory rate >20/min

PaCO2 <32 mmHg (4.3 kPa)

White blood cell count of >12 x109/l

SIRS is the syndrome arising from the bodys inflammatory reaction to a damaging insult such as
infection, trauma, burns or acute pancreatitis. SIRS is recognised by the presence of the
following clinical criteria: Temperature >38 oC or <36 oC
Heart rate >90/min
Respiratory rate >20/min or PaCO2 <32 mmHg (4.3 kPa)
White blood cell count of >12 x109/l, or <4 x109/l, or the presence of >10 % immature forms
Correct
Which of the following statements fulfil the criteria for the correct definition of sepsis?

Single best answer question choose ONE true option only


The presence of micro-organisms in the blood stream
The presence of micro-organisms within a normally sterile viscus

Hypotension refractory to resuscitation in the presence of demonstrable infection

A systemic inflammatory response occurring as a direct result of infection Your answer


A raised respiratory rate , a high white cell count and the presence of a proven source of
infection

The American College of Chest Physicians and the Society of Critical Care have defined sepsis as
a systemic inflammatory response syndrome (SIRS) as the result of a confirmed infectious
process.
The SIRS is defined when two of the following are present:
*
*
*
*

Pyrexia
Tachycardia
Tachypnoea
A raised white cell count

As a SIRS can occur secondary to non infectious causes (e.g. trauma, malignancy), sepsis is
defined as a SIRS occurring as a direct result of infection.
Incorrect
The 3' 5' exonuclease activity possessed by some DNA polymerases that enables the enzyme
to replace misincorporated nucleotide is called what?

Single best answer question choose ONE true option only

Proofreading
Correct answer
Replication

Recombination
Your answer
Retrotransposition

Splicing

Retrotransposition is transposition via an RNA intermediate (transposition is the movement of a


genetic element from one site to another in a DNA molecule). Splicing is the removal of introns
from the primary transcript of a discontinuous gene.

Correct
Which of the following is produced by the duodenum?
Single best answer question choose ONE true option only
Cholecystokinin
Secretin Your answer
Amylase
Lipase
Elastase
The duodenum secretes secretin in response to acid chyme from the stomach. Secretin
promotes production of water and bicarbonate from the pancreatic duct cells.
Incorrect
A 25-year-old man is admitted to hospital with persistent vomiting. He is clinically dehydrated
and hypotensive. His serum sodium concentration is 124 mmol/l, potassium 4.9 mmol/l, urea 9.8
mmol/l, creatinine 96 mmol/l. Urine sodium concentration in a specimen passed on admission is
62 mmol/l.

Which of the following is the most likely cause of the hyponatraemia?


Single best answer question choose ONE true option only

Adrenal failure
Correct answer
Cerebral salt wasting

Gastrointestinal fluid loss


Your answer
Low sodium intake

Syndrome of inappropriate antidiuresis (SIAD)

Natriuresis in a dehydrated, hyponatraemic patient suggests that there is uncontrolled renal loss
of sodium, such as occurs in adrenal failure. Cerebral salt wasting can also cause dehydration
and hyponatraemia due to excessive natriuresis, but typically occurs following a head injury or
brain surgery. Hyponatraemia and dehydration due to gastrointestinal fluid loss or sodium
deficiency due to a low intake should lead to renal conservation of sodium. Although SIAD is an
important cause of hyponatraemia and sodium excretion may be high, the hyponatraemia is due
to water excess and patients are not dehydrated.
Correct
You are called to ICU to see a 65-year-old patient who requires controlled mechanical ventilation
after major non-cardiac surgery but is becoming hypoxaemic when the FiO2 is reduced from 0.4
to 0.3.

Which of the following statements is true? Single best answer question choose ONE true option
only

Simple indices of circulatory status such as urine output, blood pressure and CVP correlate
well with outcome from high-risk surgery

Survivors after major surgery decrease their cardiac index and oxygen delivery in the
perioperative period below baseline normal values

Measurement of mixed venous oxygen saturation (SVO2) requires a pulmonary venous (PV)
catheter to sample pulmonary capillary blood

Cardiac index and oxygen delivery correlate poorly with outcome from high-risk surgery

Pre- or perioperative beta-blockade can improve survival after major non-cardiac surgery in
patients with pre-existing cardiac disease
Your answer
Simple indices of circulatory status such as urine output, blood pressure and CVP correlate
poorly with outcome from high-risk surgery. Survivors after major surgery increase their cardiac
index and oxygen delivery in the perioperative period above baseline normal values.
Measurement of mixed venous oxygen saturation (SVO2) requires a pulmonary artery (PA)
catheter to sample pulmonary capillary blood. Cardiac index and oxygen delivery correlate well
with outcome from high-risk surgery. Two recent multicentre trials have confirmed the advantage
of using highly selective pre- or perioperative beta-blockade to improve survival after major noncardiac surgery in patients with pre-existing cardiac disease, eg previous heart failure, moderate
hypertension and myocardial infarction (MI). The regime is started 4872 h preoperatively and
continued for 1428 days post-surgery.
Correct
A 75 kg man has suffered acute loss of 25 % of his blood volume, has a pulse rate of 110/min, a
ventilatory rate of 25/min and a urine output of 25 ml/h. Which class of haemorrhagic shock
most appropriately describes this patient?

Single best answer question choose ONE true option only


Class I haemorrhagic shock

Class II haemorrhagic shock Your answer


Class III haemorrhagic shock

Class IV haemorrhagic shock

None of the above

The patient exhibits signs of class II haemorrhagic shock. ATLS guidelines classify
haemorrhagic shock into 4 categories as shown in the table below:-

Class Class I Class II Class III Class IV


Blood loss (ml) <750 750 - 1500 1500 - 2000 >2000
% blood vol lost <15% 15 - 30% 30 - 40% >40%
Pulse rate (min) <100 >100 >120 >140
Systolic BP Unchanged Unchanged Decreased Decreased
Diastolic BP Unchanged Increased Decreased Decreased
Pulse Pressure Unchanged Decreased Decreased Decreased
Urine output (ml/h) >30 20 - 30 5 - 15 Anuria
CNS features Slight anxiety Mild anxiety Anxiety/Confusion Confusion
Correct
Myeloid stem cells give rise to several different cell types. Which of the following is not one of
these?
Single best answer question choose ONE correct option only
Neutrophils
Monocytes
Platelets
Lymphocytes Your answer
Macrophages
Bone marrow produces pluripotential stem cells which give rise to two lines of cells myeloid
and lymphoid stem cells. Myeloid stem cells differentiate into the polymorphonuclear leucocytes
neutrophils, eosinophils and basophils. It also gives rise to monocytes and macrophages. The
lymphoid stem cell line produces lymphocytes, both T and B types.
Incorrect
A 75-year-old woman is being followed by her GP for suspected developing primary
hypothyroidism.

Which of the following biochemical changes would you most expect to occur first?
Single best answer question choose ONE true option only

Fall in serum free thyroxine


Your answer
Fall in serum thyroxine-binding globulin

Fall in serum free triiodothyronine

Fall in serum total triiodothyronine

Increase in serum TSH


Correct answer
Hypothyroidism develops gradually, often over many months or even years. In the early stages,
free thyroxine concentrations are maintained in the normal range by the increased secretion of
TSH. Patients with a slightly elevated TSH and lownormal thyroxine are said to have
compensated or borderline hypothyroidism. In some individuals, it appears that this state can
be maintained without progression to frank hypothyroidism. Triiodothyronine concentrations tend
to fall later than thyroxine concentrations in hypothyroidism; the concentration of thyroxinebinding globulin does not change significantly
Incorrect
In estimating the physiological clearance of 10 ml of an intravenous substance which has been
administered at 10 mg/ml, the plasma concentration at equilibration is 15 mg/litre, the urine
concentration is 150 mg/litre and the subject produces 1440 ml of urine during a 24h collection.

What is the clearance of the substance? Single best answer question choose ONE true option
only

1 ml/min

10 ml/min
Correct answer
0.1 ml/min

100 ml/min

Cannot say from the information given


Your answer
Clearance is calculated using the formula (U V)/P where U = urine concentration in mg/ml, V =
urine production in ml/min, P = plasma concentration in mg/ml.
The bolus size of the substance is irrelevant to the clearance.
Incorrect
Which of the following organs has the greatest blood flow per 100 g of tissue?

Single best answer question choose ONE true option only


Brain

Heart

Skin

Liver
Your answer
Kidneys
Correct answer

Organ
Blood flow in ml/100g/min
Kidneys
420.0
Heart
84.0
Liver
57.7
Brain
54.0
Skin
12.8
Incorrect
The action potential of skeletal muscle?
Single best answer question choose ONE true option only
Has a prolonged plateau phase

Spread inwards to all parts of the muscle via the T tubes


Correct answer
Causes immediate uptake of Ca into the sarcoplasmic reticulum

Your answer
Is longer than the action potential of cardiac muscle

Is not essential for contraction

The action potential of the skeletal muscle spreads out from the motor end plate, through the T
tube system this causes mobilization of Ca2+ from the sarcoplasmic reticulum to the cytoplasm
and this action potential is essential for contraction.

The action potential of cardiac muscle is longer than that of the skeletal muscle and has plateau
phase.
Incorrect
Botulinum toxin has which of the following features?
Single best answer question choose ONE true option only

It is produced by a Gram-positive, aerobic bacillus

The bacillus has 15 serotypes

Its main activity is at the presynaptic membrane


Your answer
It may be used in the treatment of myasthenia gravis

It may be used in the treatment of blepharospasm


Correct answer
Clostridium botulinum is a Gram-positive, spore-forming, obligate anaerobe. The bacillus has
seven serotypes, A to G. They have a wide range of therapeutic usage, from glabellar lines,
blepharospasm, spasticity, anismus, anal fissure to dystonia. However, myasthenia gravis would
be expected to worsen with such treatment.
Incorrect
A 34-year-old woman with a body mass index of 44 kg/m2 seeks medical help for her obesity.

Which one of the following treatments offers her the highest probability of achieving a long-term
reduction in weight? Single best answer question choose ONE true option only

An energy-deficient diet (600 kcal/day (~ 143 J/day) less than requirements) for 6 months

Jaw-wiring and milk feeding for 3 months

Treatment with orlistat for 12 months

Treatment with sibutramine for 12 months


Your answer
Vertical banded gastroplication
Correct answer
Both sibutramine and orlistat have been shown to induce and maintain a greater weight loss
than diet alone, but a patients weight often plateaus before adequate weight loss has occurred.
Orlistat is only licensed for use for 1 year in the UK, and sibutramine for 2 years. Energydeficient diets, particularly if coupled with increased exercise, are effective, but the lost weight is
almost invariably regained, as it is after jaw-wiring and milk feeding. Surgery offers the best
chance of achieving long-term weight loss, the results from vertical banded gastroplication
combined with a by-pass procedure being even better than those with gastroplication alone
Correct
A 32-year-old woman on nasogastric aspiration for paralytic ileus following surgery develops a
metabolic alkalosis.

Which of the following intravenous fluids would be the preferred treatment for the alkalosis?
Single best answer question choose ONE true option only

5% dextrose

Dextrose saline

Normal (0.9%) saline


Your answer
Ringers lactate

Twice normal (1.8%) saline

The metabolic alkalosis secondary to a loss of gastric acid is a hypochloraemic alkalosis. This is
perpetuated by the hyperchloraemia, which prevents renal excretion of the excess bicarbonate
since its proximal tubular reabsorption (with sodium) is enhanced. Provision of adequate chloride
ions allows the excess bicarbonate to be excreted and corrects the alkalosis. Dextrose 5%
contains no chloride and dextrose saline contains insufficient for this purpose. Twice normal
saline is occasionally used for treating severe hyponatraemia but has no place in this clinical
situation. Ringers lactate is inappropriate, since the metabolism of the lactate that it contains to
bicarbonate would exacerbate the alkalosis.
Correct
The actions of active Vitamin D include all of the following except:

Single best answer question choose ONE true option only


Increased calcium excretion from the kidneys
Your answer
Inhibition of PTH release from the parathyroid glands

Increased phosphate absorption from the intestines

Increased calcium absorption from the intestines

Increased bone mineralisation

Active Vitamin D plays a crucial role in the homeostasis of calcium. Whether the source is from
the skin or dietary, 25- and 1-alpha hydroxylation is required in the liver and kidney respectively
to convert Vitamin D into its metabolically active form. Its main function in calcium homeostasis
includes an increase in the absorption of both calcium and phosphate from the gut and the
stabilisation and the promotion of mineralization in bone. It also acts directly on the parathyroid
gland to inhibit the release of PTH. This provides a negative feedback mechanism as PTH is
required for hydroxylation of Vitamin D in the kidney.
Incorrect
You are asked to see a patient who had a chest drain removed 4 days ago. There appears to be
some infection.

What are the stages in the cell biology of normal wound healing?
Single best answer question choose ONE true option only

Demolition is the first phase


Your answer
Maturation and remodelling can continue for up to a year
Correct answer
Acute inflammation usually lasts for 612 hours

Epithelial cell proliferation is the hallmark of the demolition phase

Collagen deposition is the key process during demolition

The first phase in healing by first intention is the phase of acute inflammation that lasts up to 3
days, if uncomplicated. The initiating factor appears to originate from platelets activated by
mature collagen exposed in the wound. Platelets first aggregate then release a variety of active
agents including lysosomal enzymes, ATP, serotonin and wound cytokines. A fibrin clot develops,
which completes haemostasis and provides strength and support to the wound. The surface
dries to form a scab. Platelets and macrophage factors cause local vasodilatation, which
produces warmth and increases capillary permeability, allowing serum and white blood cells to
accumulate and cause swelling.
After the initial acute inflammation, macrophages become active as the main agents of
demolition, removing unwanted fibrin, dead cells and bacteria and creating fluid-filled spaces for
granulation tissue. Macrophages also release factors that stimulate the formation of new
capillary buds during this phase, and later they initiate and control fibroblast activity during
repair. Within the connective tissue, randomly orientated collagen begins to form after a few
days, reaching a peak of activity after 57 days.
Epithelial cells at the edge of the wound start to proliferate after 24 h and this phase can last for
up to 3 weeks.
Reply to HalaReport

Post #3
Hala Adel wrote6 hours ago
Finally, the phase of maturation and remodelling lasts for up to 12 months, during which time
the tensile strength of the wound increases and the random collagen is replaced by a more
stable form orientated along lines of stress.
Incorrect

Which of the following is the most important direct stimulus to respiration?


Single best answer question choose ONE true option only
Increased pCO2 of the CSF
Your answer
Increased H+ concentration of the CSF
Correct answer
Decreased arterial pO2

Decreased arterial pH

Decreased arterial pCO2

Chemoreceptors involved with the control of respiration are present in the central nervous
system and peripherally. The central chemoreceptors are situated in the ventral medulla, and
increase firing in response to the H+ concentration of the brain extra cellular fluid, which is
directly related to the H+ concentration in the CSF. CO2 / HCO3 cannot cross the blood brain
barrier, but CO2 does so readily. This frees H+ ions, causing a low CSF pH, increased firing of the
central chemoreceptors and increased ventilation.

Peripheral chemoreceptors are found in the carotid bodies and aortic arch, and increase their
firing rate in response to decreased PaO2, decreased arterial pH and increased paCO2. These are
much less important, however, in stimulating respiration than the central chemoreceptors.
Incorrect
What is the term for the volume of expired air at forced expiration?
Single best answer question choose ONE true option only
Forced vital capacity (FVC) Correct answer
Functional residual capacity (FRC)
Expiratory reserve volume (ERV) Your answer
Residual volume (RV)
Total volume (TV)
FVC is the amount of air expelled during forced expiration and has clinical significance. It is
reduced in restrictive disease.
FRC is the volume of gas left in the lung at the end of quiet respiration.
ERV is the maximum volume of expired air.
RV is the volume of air remaining in the lungs after forced expiration (FRC).
TV is the total volume of air in the lungs and includes the residual volume.
Incorrect
Reply to HalaReport

Post #4
Hala Adel wrote6 hours ago
MRCS part I 10 sept 2007 recalls
these are the questions i able to remember.
SBA questions
1) young man has pelvic fracture ---> sudden onset of acute urinary retention. What is the
possible cause?

a) urethral injury
b) bladder rupture
c) ureter injury
2) Causes of raise PSA
a) Prostatic ca
b) prostitis
3) Bee sting, presented with HR 120, BP 60/40
first treatment....
a) IV antihistamine
b) IV fluid
c) IV steroid
d) local antihistamine
e) s/c adrenaline
4) noradrenaline binds to...
a) a1 receptor
b) a2 receptor
c) b1 receptor
d b2 receptor
5) post op developed high glucose level. prior to op, pt is not DM. This is due to .....
GH secretion post-op?
6) pt has splenic rupture. denied any trauma. what infection can cause spenic rupture?
a) EBV
b) mumps
c) measles
7) recurrent UTI, pneumuria, and irregular bowel habits. CT shown mass involved both the
sigmoid and bladder.
a) diverticulitis
b) sigmoid ca
c) Crohn's ds
d) UC
loss of appetite, malaise, multiple lymphadenopathy involvement...axillary, inguinal
a) malignant lymphoma
9) what is the course of median nerve related to brachial a.
.......from medial to ant to lat to brachial a.
10) a cut above the ulnar olecranon cause unbale to extend UL. what is the tendon being cut?
---> tricep tendon
11) sprinting duirng playing football --> pain and post of the thigh. later severe pain and the lat
side of the knee. unable to extend knee dt pain. what is the tendon being involve?
--> tendon of biceps femoris
12) the first structure being noted after open up the popliteal fossa
a) popliteal vein
b) femoral n
c) popliteus
13) accident --> multiple tibial and fibula # --> intramedullary nailing done. 6 hours later --->
severe pain at leg
a) DVT
b) compartment syndrome
14) malaise, weight loss, cervical lymp nodes. biopsy --> epitheliod macrophages and giant cell
----> TB
15) the border of the snuff box
a) extensor pollicis longus
b) abductor pollicis brevis
c) flexor digitorum longus
16) cut at the midline between the base of the little finger and the wrist ---> cause loss of thumb
adduction power

what is the nerve being injured?


a) superficial ulnar n
b) deep ulnar n
c) medial nerve
d) radial n
17) after the varicose vein surgery, loss sensation and the dorsal of the foot, unable to dorsiflex
the foot
what is the nerve being injured?
a) common peroneal n
b) sup peroneal n
c) deep peroneal n
1 2-days old neonates, cyanoses at the LL. Weak pulse and LL. BP 60/40 and both UL.
what is the abn
a) pulmonary atresia
b) aortic arch abn
c) VSD with pul stenosis
19) trauma to the chest, CXR shown widening of the mediastinum. what of the structure being
rupture?
ascending aorta
descending aorta
20) a knife penetrate the midline of the sternal angle with injure
a) trachea
b) oesophagus
c) sup vena cava
d) azygos V
21) IN surgical ICU, pt develop metabolic acidosis. what is the most common cause?
a) vomiting
b) nasogastric aspiration
i will post later....pls other whom remember pls post
Reply to HalaReport

Post #5
Hala Adel wrote6 hours ago
what is the acute management for gaining a airway in acute resp distress?
a) chest tube
b) needle thro the cricothyroid membrane
fresh blood noted at the chest tube, the bleeding is from?
a) intersostal a
b) pericardiophrenic a
c) r ventricular
insulin dependent, h/o chest infection started with antibiotic, admitted with drowsiness --> DKA
what is the electrolyte imbalance
------> hyperkalemia
non-alcoholic, with palpable nodular liver, biopsy confirm is HCC. what is the cause for the
patient --> HBV cirrhosis
2 degree partial thickness burn, developed bilaterally LL swelling. what is the cause? ------>
hypoalbuminemia
hlo back pain, walking cause pain at the L LL, loss os the sensation over the surface of the knee,
what is the cord lesion ----> L3
profuse LGIB, contrast accumulate at the left iliac fossa, for vessel embolisation, which level of
artery is cannulate?
----> L3 ( inferior mesenteric artery )
during prolapsed interventricular disc, what is the structure compressed on the nerve? -->
nucleus pulposus

after the mastectomy, the woman has a wing scapula, what is the nerve being injured?
-----> long thoracic nerve
Reply to HalaReport

Post #6
Hala Adel wrote6 hours ago
woman presented with the lump at ant neck, move with swallowing, FNAC done confirm is
malignancy......what is the CA --> papillary thyroid ca
a surgery done for the sweating palm dissert at the base of the neck ant to the first rib, what is
the complication
--> phrenic nerve injury ? R diaphragm elevation
during hypotensive shock what is the first substances to be secreted
-- angiotensinogen
-- angintensinI
-- angintensin II
-- aldosterone
-- renin
a man hav a trauma over the medial part of the thigh, clean wound, closure done. few day later
patient developed pulsatile mass...
----> false aneurysm of the femoral a.
hip replacement woman, walk with the tredelenburg gait. what is the defect?
-- sciatic nerve
-- gluteus medium
-- femoral nerve
what is the mechanism of the counter-current in the nephron for the concentration of the urine?
--> impermeability of the thick ascending for the water
Reply to HalaReport

Post #7
Hala Adel wrote6 hours ago
MRCS09 JAN 13TH QUESTION/THEMES
[b]1. WHAT IS THE ANTERIOR BOUNDARY OF FEMORAL CANAL

Inguinal ligament

2. WHAT IS THE POSTERIOR WALL OF FEMORAL CANAL Pectineal ligament and superior

pubic ramus
3. DIRECT INGUINAL HERNIA IS THE WEAKNESS OF WHICH WALL NAME
4. anterior surface of heart is formed by
5. heart valve is made of
6. MEN II b comprises of
7. medullary carcinoma presnts as
8. 1st web is supplied by which nerve
9. distribution of supf peroneal nerve
10. level of bifurcation of aorta
11. level of hilum of kidney
12. level of inf mesenteric artery

13. level of ext iliac artery


14. distribution of genitor femoral nerve
15. definition of hypoxia
16. tissue development in # atrophy
17. tissue development in spina bifida hypoplasia
18. post gastrectomy deficiency of
19. pt with persistent vomiting biochem abnormality
20. passive rectal incontinence sphinter envolved
21. presentation of long standing catheter
22. tubercular cystitis
23. nerve supply of ant aspect of knee jt
24. relation of str at popliteal fossa
25. muscle attach to lat side of popliteal
26. nes of adductor of thigh
27. flexion at distal ip jt at ring finger
28. flexion at distal ip jt at thumb
29. relation of ulnar nerve with ulnar artery
30. ulnar injury at elbow
31. foment sign test for
32. nerve for adduction of thumb
33. deep br of ulnar nerve supply
34. function of cortisol in stress
35. ecg changes in pulm embolism
36. nerve supply at angle of mouth
37. lateral border of tongue is supplied by
38. gastric lymphoma are mc of which type
39. cut injury at side of face will cut
40. epydymorchitis in sexually active male without uti
41. cause of swelling scrotum with sec neck
42. multiple swelling all over neck/axill /inguinal malignant lymphoma
43. follicular tumour thyroid IOC
44. sesation over deltoid muscle
45. injury medial to deltopectoral groove
46. head of radius articulate with
47. Head of radius is kept in place by which ligament

48. cereberal perfusion pressure


49. central chemo receptors
50. baroreceptors
51. foramina for transmission of mandibular nerve
52. foramina for transmission of vagus/ hypoglossal
53. foramina for transmission of middle meningeal artery
54. nucleus of 6th and 7th cr. Nerve is at which part of brain
55. surface marking of heart valve
56. cardiac tamponade can cuase sudden death
57. peptic acid secretion is stimulated by
58. bee sting with bp 80/60 and p.rate 122/mt TtOC
59. dorsum of foot supplied by which nerve
60. ligation of varicocele ultimately vein leadin to gonadal vn
61. relation of vein at renal hilum
62. COPD
63. blood gas analysis
64. blood gas analysis
65. blood gas analysis
66. difficulty in smiling nerve for it
67. pain in inf molar nerve responsible
68. ganglion for lacrimal dut
69. nerve involved in submandibular gland excision
70. type of reaction rhinorrhea with rashes after two hrs
71. intrinsic factor absorption
72. anemia in gastrctomy
73. iron deficiency anaemia
74. granulomatous intestinal dis crohn.
75. biopsy of gastric antrum
76. causative agent for gastric ulcer and bladder carcinoma
77. virus implicated for cervical cancer
78. virus implicated for anal cancer
79. virus implicated for Kaposi sarcoma
80. pus in diabetic cholycystitis e. g. of
81. injury at third intercostals space at sternum
82. third ventricle to fourth cereberal duct of aqueduct

83. SOL lt in parital causes herniation of


84. vertebral artey supplies which cortex
85. pituitary tumour causing pressure at optic chiasma
86. erosion at lateral angle of eye by swell in three years
87. position of ampulla of vater opening
88. SVR/CO/HR in crdiogenic shock
89. SVR/CO/HR in hemmorrafgicc
90. SVR/CO/HR in septic
91. child with ear discharge fever convulsion high grade fever
92. best criteria fo acute pancreatitis
93. diagnostic marker of carcinoid
94. in aneurysm defect in arterial wall is
95. popliteal aneurysm size 2.5 cnm
96. nodule in rheumatoid arthritis
97. wrist drop wekness at elbow
98. sensation of middle finger root value
99. sensation at lat calf disc protrusion
98 sensation loss at great toe spinal levet
100. lspinal level in uretric pain
101. passage of small stone cause
102. ns of lateral part of arm
103 parasympathetic causes arterial vaso constriction /dilatation
104 sa node is inervated by
105 adrenaline causes st of which receptors
106 role of adrenaline
107 role of dobutamine
108 role of ephedrtine
109 changes in lung capacity in emphysema
110 FEV1/FCV is reduced in
111 Ist to reponse to haemmorrage
112 Patent ductus arteriosus develop fromn which arch
113 Trauma to medial third of clavicle will injure
114 Bendrofluadizide acts at
115 Inrease urine osmolalty in
op adissonian crisis\
117 Left femoral hernia op for small bowel obstruction lead to chest pain?

118 Factor controlling bp from kidney


119 Post operative hypocalcemia in thyroid ectomy
120 Mode of action in pth in ca metabolism
121 Ca breast role of in osteoporotic Calcitonin
122 Cuases of hypercalcemia burn?
123 b/l swelling in 60% burn pt due to hypoprotienemia
124 insulin is increased with c peptide
125 partail gastrectomy result in dumping syndrome
126 multiple neurological envolvement in diabetes
127 insulin given in hyper glycemic will
128 effect of cortisol on various hormone
129 barret esophagus > 5cm pathology envolved
130 para neoplastic syndrome see in small cell tumour
131 in fetal circulation blood passes from rt atrium to left atrium
132 intermittent positive pressure incr/decreases venous return
133 basic mechanism in pulm embolism
134 post splenectomy diffuse opacity lung
caused by pneumoccocal pneumonia
135 ca brest operation nerve envolved in winging of scapula
136 upper arm injury with swelling and pain fasciotomy
Reply to HalaReport

Post #8
Hala Adel replied to Hala's post5 hours ago
MRCS Part 1 Practice Questions ( Physiology ) - 2 of 3
Correct
In a patient with anaphylaxis, which of the following should be given to inhibit the important
late-phase reaction? Single best answer - choose ONE true option only

Antihistamines

Epinephrine

Leukotriene inhibitor

Hydrocortisone
Your answer
NSAID

Hydrocortisone blocks the generation of leukotrienes and prostaglandins, and hence prevents
the late-phase reaction often characterised by asthma. It should be given
intravenously/intramuscularly at a dose of 100200 mg. None of the other agents listed above
affect this aspect of anaphylaxis. Approximately 30% of deaths related to anaphylaxis occur as a
consequence of this late-phase reaction.
Correct
Which of the following is true of bile?
Single best answer question choose ONE true option only
Is secreted into the terminal ileum
Is necessary for protein absorption
Contains urobilinogen Your answer
Is produced by the cells lining the common bile duct
Is concentrated in hepatocytes
Bile is a solution of bile salts (bilirubin), pigments and cholesterol. It is secreted by the
hepatocytes and concentrated in the gall bladder. Following ingestion of a fat-containing meal,
cholecystokinin stimulates the gall bladder which in turn contracts and expels bile through the
cystic duct into the common bile duct. Bile is secreted into the duodenum.
Correct
In a patient with small bowel ischaemia, what metabolic picture would most likely be seen on
blood gas analysis?
Single best answer question choose ONE true option only
Compensated metabolic acidosis

Metabolic acidosis and increased anion gap


Your answer
Metabolic acidosis and normal anion gap

Metabolic alkalosis

Respiratory acidosis

This patient has had a significant operation during which infarcted bowel has been resected. The
most likely abnormality is a metabolic acidosis secondary to mesenteric ischaemia and
hypovolaemia resulting in anaerobic metabolism and accumulation of lactic acid. By definition,
this patient will have a low arterial pH and a low bicarbonate concentration. An increased anion
gap will also be seen.
The anion gap may be calculated by;

[Na+] + [K+] [CL-] [HCO3-]

The normal anion gap is 8-16mmol/L. The anion gap is a useful tool in differentiating between an
acidosis due to the accumulation of organic acids e.g. lactic acid (as in this patient) and acidosis
that are secondary to the loss of base or ingestion of acid where there will be a normal anion
gap.
Correct
Absorption of calcium from the digestive tract?
Single best answer question choose ONE true option only
Takes place mostly in the proximal jejunum
Your answer
Is prevented by the presence of small amounts of phytic acid in the diet , even when an excess
calcium is ingested

Is facilitated by the presence of fat in food

Can be reversed (calcium is secreted into bowel lumen) when plasma calcium concentration is
raised by a calcium infusion

Is about as rapid as that of sodium

Phytic acid produces insoluble calcium phytate, when all phytic acid has been precipitated the
excess calcium is absorbed.

Fatty acids form insoluble calcium salts (soaps).

The shift of calcium ions across the intestinal mucosa is virtually one way.

Sodium is absorbed at a speed fifty times that for calcium absorption


Correct
Following a motorbike RTC, a young patient has a heart rate of 42 , BP of 70/45 mmHg, and
warm peripheries. His blood pressure does not improve despite IV fluids. What is the likely
diagnosis?
Single best answer question choose ONE true option only
Cardiogenic shock

Hypovolaemic shock

Neurogenic shock

Your answer
Septic shock

Septic shock

Neurogenic shock is a result of interruption of the descending sympathetic pathways of the


spinal cord causing loss of vasomotor tone. There is subsequent pooling of blood in the
extremities and the development of hypotension. Additionally if the lesion is above T6 there may
be associated loss of cardiac sympathetic innervation, therefore, these patients are often
bradycardic or are unable to mount an appropriate tachycardic response to hypovolaemia. As
the primary problem in these patients is loss of sympathetic tone the observed hypotension does
not respond to fluids and must be corrected with the use of vasopressors that increase vascular
tone and atropine if indicated to counter the bradycardia.
Correct
Which of the following respiratory physiology tests would be consistent with a diagnosis of
moderately established cryptogenic fibrosing alveolitis?

Single best answer question choose ONE true option only

Diffusion capacity decreased, FEV1/FVC normal, total lung capacity reduced


Your answer
Diffusion capacity increased, FEV1/FVC normal, total lung capacity increased

Diffusion capacity normal, FEV1/FVC reduced, total lung capacity reduced

Diffusion capacity decreased, FEV1/FVC normal, total lung capacity n ormal

Diffusion capacity decreased, FEV1/FVC increased, total lung capacity increased

Diffusion capacity is characteristically decreased in restrictive lung disorders. FEV1/FVC reduced


would be seen in obstructive airways disease, which would be reversible in asthma and
irreversible in COPD. In restrictive conditions FEV1/FVC ratio is normal or increased. Total lung
capacity is reduced in restrictive lung disease, whilst it is normal or increased in obstructive
airways disease.
Correct
The plateau phase of the cardiac action potential is due to:

Single best answer question choose ONE true option only


Magnesium influx

Potassium influx

Calcium influx
Your answer
Chloride efflux

Sodium influx

The most important source of activator calcium in cardiac muscle remains its release from the
sarcoplasmic reticulum. Calcium however also enters from the extracellular space during the
plateau phase of the action potential. This calcium entry provides the stimulus that induces
calcium release from the sarcoplasmic reticulum (calcium induced calcium release).

The result is that tension generated in cardiac, but not in skeletal, muscle is profoundly
influenced both by extracellular calcium levels and factors that affect the magnitude of the
inward calcium current. This is of practical value in two key clinical situations; in heart failure
where digoxin is utilised to increase cardiac contractility (by increasing the intracellular calcium
concentration) and in hyperkalaemia where calcium gluconate is used to stabilise the
myocardium.

The plateau phase of the action potential in cardiac muscle (principally due to calcium influx)
maintains the membrane at a depolarised potential for as long as 500ms. The result is that the
cell membrane is refractory throughout most of the mechanical response, largely due to the
inactivation of fast sodium channels. This prevents tetany upon repetitive stimulation which
would be detrimental to cardiac output. Furthermore, the prolonged refractory period in cardiac
muscle allows the impulse that originates in the sino-atrial node to propagate throughout the
entire myocardium just once, thereby preventing re-entry arrhythmias.
Correct
Which components of the nephron are most important with regulation of extracellular fluid
osmolality?

Single best answer question choose ONE true option only


Proximal convoluted tubule and distal convoluted tubule

Glomerulus and distal convoluted tubule

Loop of Henle and collecting ducts


Your answer
Glomerulus and proximal convoluted tubule

Glomerulus and loop of Henle

Each component of the nephron is associated with particular predominant functions. The
glomerulus is involved with passive filtration of the plasma and formation of tubular filtrate. The
proximal convoluted tubule is mainly involved with conservation of filtered solutes and water as
well as secretion of certain waste products. The distal convoluted tubule plays a role in
regulating preferential reabsorption of Na+ ions at the expense of K+ and H+ ions, under the
control of aldosterone. It is the loop of Henle and collecting ducts which play the most important
role in regulating extracellular fluid osmolality. The loop of Henle creates the large medullary
interstitial osmotic driving force for the reabsorption of water through the walls of the collecting
ducts whose permeability is regulated by antidiuretic hormone (arginine vasopressin).
Correct
A patient with a significant head injury has a GCS of 6, a dilated left pupil and is found to be
coning. Which of the following vital signs is this patient likely to exhibit?
Single best answer question choose ONE true option only
Hypertensive and bradycardic
Your answer
Hypertensive and tachycardic

Hypotensive and bradycardic

Hypotensive and normal heart rate

Normotensive and tachycardic

This patient has signs of raised intra-cranial pressure (ICP). The dilated left pupil reflects
oculomotor nerve compression secondary to transtentorial cerebral herniation. Hypertension and
bradycardia may be observed in such patients, this is known as Cushings reflex and it reflects
an attempt to maintain cerebral perfusion in the face of rising ICP.

Cerebral perfusion pressure (CPP) is the Mean arterial pressure (MAP) minus the intracranial
pressure (ICP).
i.e. CPP = MAP ICP.

The Monroe-Kellie hypothesis states that the skull is a rigid box that contains brain, CSF, and
blood,
therefore, ICP = VCSF + VBrain + VBlood.

It stands to reason that if the volume of any one of these components increases i.e. an
intracerebral haemorrhage, the ICP will rise. The rise in ICP may be minimally compensated by a
decrease in the two other components, but after this point the ICP will rise steeply.
Incorrect
Which one of the following statements about renin secretion is true?

Single best answer question choose ONE true option only


Renin is secreted by the epithelial cells of the renal glomerulus
Your answer
Reduced delivery of NaCl to the macula densa cells of nephrons increases renin secretion
Correct answer
A rise in pressure in the renal afferent arteriole increases renin secretion

Renin secretion is reduced by increased activity in the renal sympathetic nerves

Renin secretion is reduced by inhibition of angiotensin-converting enzyme

Renin is an enzyme involved in activating the angiotensin-aldosterone system. It is produced and


secreted by modified smooth muscle cells of the afferent arterioles of the kidney. Renin secretion
is stimulated by a local fall in blood pressure in the afferent arterioles, by reduced delivery of
filtered NaCl to the macula densa cells of the nephrons (tubulo-glomerular feedback) and by
increased activity in the renal sympathetic nerves. Renin secretion is increased by inhibition of
angiotensin-converting enzyme since the resulting reduction in angiotensin II and aldosterone
levels reduces the negative feedback effect on renin secretion.
Correct
Aldosterone is secreted from the:

Single best answer question choose ONE true option only


Liver

Zona glomerulosa of the adrenal cortex


Your answer
Juxtaglomerular apparatus

Adrenal medulla

Zona fasciculata of the adrenal cortex

The adrenal gland comprises an outer cortex and an inner medulla, which represent two
developmentally and functionally independent endocrine glands within the same anatomical
structure. The adrenal medulla secretes adrenaline (70%) and noradrenaline (30%). The adrenal
cortex consists of 3 layers, or zones. The layers from the surface inwards may be remembered
by the mnemonic GFR:

G = Zona glomerulosa (secretes aldosterone)


F = Zona fasciculata (secretes cortisol and sex steroids)
R = Zona reticularis (secretes cortisol and sex steroids)

Aldosterone is a steroid hormone that facilitates the reabsorption of sodium and water and the
excretion of potassium and hydrogen ions from the distal convoluted tubule and collecting ducts.
Conns syndrome is characterised by increased aldosterone secretion from the adrenal glands.
Incorrect
A patient on enteral nutrition develops constipation. What could explain the underlying clinical
physiology? Single best answer question choose ONE true option only

Hyperosmolar feed
Your answer
Bacterial contamination

Low feed temperature

Inadequate fluid replacement


Correct answer
Reduced intestinal absorptive capacity

Hyperosmolar feed, bacterial contamination, low feed temperature, too rapid or irregular
administration, lactose intolerance, reduced intestinal absorptive capacity can all explain
diarrhoea.
Incorrect
An overweight 32-year-old woman presents with a short history of painless jaundice. There is no
previous history of illness and, apart from the jaundice, she has no signs of chronic liver disease.
Initial investigations reveal a haemoglobin of 12.7 g/dl, MCV 105 fl, serum bilirubin 162 mmol/l,
AST 145 U/l, alkaline phosphatase 224 U/l, gamma-glutamyltransferase 200 U/l.

Which of the following is the most likely diagnosis? Single best answer question choose ONE
true option only

Alcoholic liver disease


Correct answer
Autoimmune chronic hepatitis

Carcinoma of the head of the pancreas

Cholecystitis
Your answer
Hepatitis A infection

Jaundice with an elevation of both AST and alkaline phosphatase suggests mixed hepatocellular
damage and cholestatic liver disease, typical of acute alcoholic hepatitis on a background of
chronic liver disease (and is not excluded by the lack of physical signs). The high gammaglutamyltransferase lends support to this (although it may be increased in liver disease of any
cause). Macrocytosis is typical of chronic excessive alcohol intake and is not a feature of the
other conditions; although were it not present, autoimmune liver disease would need to be
considered.
In hepatitis A, AST is typically higher than alkaline phosphatase, while the reverse is true of
pancreatic carcinoma. Chronic cholecystitis can cause jaundice but it would be unusual for there
to be no history of acute episodes.
Correct
Which of the following physiological characteristics relates to the lining of the respiratory tract?
Single best answer question choose ONE true option only
About 1 litre of mucus is produced every day

The cilia are under the control of a physiological motor, dynein


Your answer
The mucociliary escalator moves at 0.2 cm/minute
Reply to HalaReport

Post #9
Hala Adel wrote5 hours ago
The bronchioles have cartilage in their wall

The bronchioles have diameters up to 5 mm

About 100 ml of mucus is produced every day. The cilia are under the control of a physiological
motor, dynein (which is absent in Kartageners syndrome). The mucociliary escalator moves at 2

cm/minute. The bronchioles do not have cartilage in their wall (which distinguishes them from
bronchi). The bronchioles can be up to 1 mm in diameter.
Correct
Which of the following hormones is secreted by the kidney in response to sympathetic nervous
stimulation?
Single best answer question choose ONE true option only
Aldosterone

Angiotensin I

Angiotensin II

Erythropoetin

Renin
Your answer
Renin is produced by the juxtaglomerular apparatus of the kidney in response to hypovolaemia,
via 3 mechanisms:
1. increased catecholamine levels secondary to sympathetic stimulation from arterial receptors
2. direct effect of hyponatraemia on the juxtaglomerular apparatus
3. reduction of renal perfusion pressure via afferent arteriolar baroreceptors

Renin acts to cleave angiotensin I from angiotensinogen produced in the liver. Angiotensin
converting enzyme is present in many tissues, especially the lungs, and converts angiotensin I to
angiotensin II. Angiotensin II is a powerful vasoconstrictor, causing vasoconstriction of renal
arteries, as well as a positive inotropic effect on the heart. It also causes release of ADH and
adrenaline. Along with aldosterone, whose release is also stimulated, Angiotensin II conserves
Na+ and H2O in the gut. Aldosterone acts to conserve Na+ and H2O in the distal renal tubule
and collecting ducts. These mechanisms combine to restore the plasma volume in
hypovolaemia.
Erythropoetin is released by the kidney in response to hypoxia and high levels of the products of
red cell breakdown, and increases the rate of red cell production
Correct
In metabolic alkalosis associated with prolonged nasogastric aspiration in postoperative ileus,
what is the most important cause of the acidbase disturbance?

Single best answer question choose ONE true option only

Hypoventilation

Increased renal bicarbonate reabsorption

Loss of gastric acid


Your answer
Potassium depletion

Secondary aldosteronism

Loss of unbuffered gastric acid is the cause of the metabolic alkalosis seen under these
circumstances if there is inadequate replacement of the fluid lost with intravenous physiological
saline. Increased renal bicarbonate reabsorption (needed to allow adequate renal sodium
reabsorption in the presence of hypochloraemia), potassium depletion (gastric secretions contain
about 10 mmol/l of potassium) and secondary aldosteronism (a result of extracellular fluid loss)
all help to maintain the alkalosis, but they do not cause it. Hypoventilation is a compensatory
change: on its own, hypoventilation causes carbon dioxide retention and a respiratory acidosis.
Correct
Which of the following malignancies could be responsible for a hypercalcaemia and low serum
phosphate level?

Single best answer question choose ONE true option only


Osteoclastoma

Squamous cell carcinoma of the lung


Your answer
Prostate cancer

Transitional cell carcinoma of the Bladder

Basal cell carcinoma

Metastatic cancerous bone lesions can result in the release of mineralised calcium and
phosphate into the blood stream which can result in both hypercalcaemia and
hyperphosphatemia.
Squamous cell carcinoma of the lung can result in hypercalcaemia with a normal or low
phosphate level due to the release of PTH related peptide in a paraneoplastic phenomenon. PTH
related peptide acts in similar fashion to PTH (although it will not be detected by standard PTH
assays) by increasing the activation of Vit D and therefore increasing the absorption of calcium
and phosphate from the intestines.
In addition, calcium and phosphate is released from bone by a direct action on osteoclasts.
However, PTH also increases the renal excretion of phosphate and the net effect can be a low or
normal serum phosphate level
Correct

The ejection fraction is defined as:

Single best answer question choose ONE true option only


The ratio of the end diastolic volume to stroke volume

The ratio of stroke volume to end diastolic volume


Your answer
End diastolic volume minus end systolic volume

End systolic volume divided by stroke volume

The ratio of stroke volume to end systolic volume

During diastole, filling of the ventricles normally increases the volume of each ventricle to about
120mls. This volume is known as the end diastolic volume. Then, as the ventricles empty in
systole, the volume decreases about 70mls, which is known as the stroke volume. The remaining
volume in each ventricle, about 50mls, is known as the end systolic volume and acts as a
reserve which can be utilised to increase stroke volume in exercise.

The fraction of end diastolic volume that is ejected is called the ejection fraction usually equal
to about 60%. The ejection fraction is often used clinically as an indirect index of contractility. It
is a particularly useful in assessing the state of the myocardium prior to aortic aneurysm repair
where cross-clamping of the aorta places particular stress on the myocardium.
Correct
Haemolytic disease of the newborn is typically restricted to the presence of Rhesus antigens on
red cells rather than ABO antigens. Predominantly, such anti-Rh antibodies cross the placenta
during the third trimester.

Which of the following statements best explains the background physiology?

Single best answer - choose ONE true option only


Antibodies to ABO blood groups are IgM, whereas antibodies to Rhesus antigens are IgG
Your answer
Antibodies to ABO blood groups are IgG, whereas antibodies to Rhesus antigens are IgM

Antibodies to ABO blood groups are IgA, whereas antibodies to Rhesus antigens are IgG

Antibodies to Rhesus antigens are IgD, whereas anti-ABO blood groups are IgM

Antibodies to Rhesus antigens are IgE, whereas anti-ABO blood groups are IgG

IgG antibodies to Rhesus antigens can cross the placenta during the last trimester, whereas ABO
antibodies are IgM and hence cannot cross the placenta. The function of serum IgD is unknown.
The transplacental passage of immunoglobulin only applies to IgG.
Correct
Which of the following TFTs is suggestive of Graves disease?
Single best answer question choose ONE true option only
Raised TSH, free T4, raised free T3

Normal TSH, raised free T4, decreased T3

Decreased TSH, raised free T4, raised free T3


Your answer
Decreased TSH, decreased free T4, decreased free T3

Raised TSH, normal free T4, normal free T3

The thyroid produces T3 and T4 upon stimulation from TSH released from the anterior pituitary,
which in turn is regulated by the hypothalamic secretion of TRH (Thyrotophin releasing
hormone). TRH is transported to the anterior pituitary along the hypophyseal tract. The negative
feedback effects of T3 and T4 levels regulate the whole mechanism.
Graves disease is the commonest cause of hyperthyroidism and is a result of IgG antibodies
binding to TSH receptors, stimulating thyroid hormone production. The TFTs in such patients
classically show a much-reduced TSH concentration with inappropriately raised T3 and T4 levels.
Correct
In the two step hydroxylation process for activation of Vitamin D, where does the first
hydroxylation take place?
Single best answer question choose ONE true option only
Kidney
Lung
Liver Your answer
Skin
Duodenum
Vitamin D is a fat soluble vitamin that is derived from our diet or via the skin from direct
sunlight. It is converted to 25-hydroxycholecalciferol in the liver and is further hydroxylated in
the kidney to 1, 25-hydroxycholecalciferol. In this active form it increases calcium uptake in the
gut and promotes phosphate absorption too. It increases kidney reabsorption of calcium and
phosphate and at very high concentration will promote osteoclastic reabsorption of bone.
Incorrect
A 45-year-old woman with type-2 diabetes is making an apparently good recovery 7 days after a

partial resection of the small intestine following trauma sustained in a stabbing incident. She is
receiving parenteral nutrition with additional normal saline and, because of a history of deep
vein thrombosis some 10 years previously, is on prophylactic heparin. Before her admission she
was well, with no ongoing medical problems and taking no regular medication. Serum electrolyte
results are as follows: sodium 129 mmol/l, potassium 6.5 mmol/l, bicarbonate 24 mmol/l, urea
8.5 mmol/l, creatinine 120 mol/l, glucose 10.2 mmol/l. Her potassium concentration has risen
over the past 3 days. The potassium content of the parenteral feed has been reduced from 60 to
20 mmol/24 h during this period. Urine output is appropriate to her fluid input. Her red cell, white
cell and platelet counts are all normal.

What is the most likely cause of the hyperkalaemia? Single best answer question - choose ONE
true option only

Heparin treatment
Correct answer
Overprovision of potassium in the parenteral feed
Your answer
Primary adrenal failure (Addisons disease)

Pseudohyperkalaemia

Renal impairment

Approximately 20 mmol/24 h is the minimum obligatory potassium output, while the typical
potassium requirements for patients on parenteral feeding are 4080 mmol/24 h.
Pseudohyperkalaemia is hyperkalaemia occurring as a result of a loss of potassium from white
cells and platelets during clotting, usually seen in patients with high white cell or platelet counts.
Typically, the plasma potassium concentration is significantly lower than the serum potassium
concentration in this condition. The elevated urea may be due to an excessive provision of
amino acids, but neither it nor the creatinine level suggest sufficient renal impairment to cause
such a severe hyperkalaemia. Incipient adrenal failure could have been made overt by the stress
of surgery, but this is uncommon. The heparin is more likely to be responsible: heparin inhibits
aldosterone secretion by the adrenal cortex, leading to impaired renal potassium excretion,
particularly in patients with diabetes or those who are acidotic
Correct
A medical SHO is required to give a blood sample to check his HepB status. He received a course
of vaccinations nine months ago.

What is his blood test likely to show? Single best answer - choose ONE true option only.

Anti-HBeAb

Anti-HBsAb
Your answer
Anti-HBsAb + anti-HBcAb

HBsAg + HBcAg

IgM to HBcAg

Surface and core antigens (HBsAg, HBcAg) are detectable during acute infection. HbeAg
(envelope) is a good marker of high infectivity, while anti-HbeAg suggests a patient who is less
infective. Acute infection is also implied by IgM to HbcAg, while IgG to HBcAg suggests a
previous infection. Viral clearance and recovery correlate with the disappearance of antigens
and the appearance of antibodies. Previous vaccination is suggested by the presence of only
anti-HBsAb.
Correct
In which of the following types of shock is the primary problem due to loss of peripheral vascular
resistance mediated by microorganisms?
Single best answer question choose ONE true option only
Cardiogenic
Anaphylaxis
Septic Your answer
Neurogenic

Hypovolaemic

Septic shock is due to bacteria-mediated vasodilation. This results in a relative loss of circulating
blood volume. Patients are peripherally warm and pink in contrast to other types of chock where
the skin is cold, clammy and shutdown. Cardiogenic shock arises due to a failure of the hearts
pump mechanism, usually post-myocardial infarction. Anaphylaxis is a severe allergic reaction
resulting in a profound release of histamine and other inflammatory mediators. There is a
relative hypovolaemia due to vasodilatation, however bacteria are not implicated in the process.
Incorrect
Which of the following metabolic effects is most likely to be caused by thyroid hormone?
Single best answer question choose ONE true option only
Decreased glycogenolysis in the liver

Increased glucose absorption in the gut

Correct answer
Decreased lipolysis
Your answer
Decreased expression of adrenergic receptors

Decreased oxygen uptake in the mitochondria

Thyroid hormone has widespread metabolic effects.


Increased glycogenolysis in the liver, increased glucose absorption in the gut and increased
insulin breakdown all tend to increase blood glucose. The glycogenolytic effects of
catecholamines are also potentiated. These effects can make the diagnosis and management of
diabetes in thyrotoxicosis difficult.
There is an overall lipolytic effect, with decreased serum cholesterol seen in thyrotoxicosis, and
an increase in hypothyroidism.
There is an increased expression of b-adrenergic receptors in many tissues including skeletal and
cardiac muscle. There is a positive inotropic effect with increased cardiac output and heart rate.
A raised metabolic rate and increased heat production are due to increased oxygen uptake and
ATP production in the mitochondria.
There are also effects on bone, with an overall breakdown of bone, sometimes leading to
hypercalcaemia. Increased serum 2,3 DPG leads to a right shift of the haemoglobin dissociation
curve. Thyroid hormones are also essential for fetal development, with deficiency leading to
cretinism. The fetus produces its own hormone from 18 weeks of gestation.
Correct
Which of the following factors is involved in the extrinsic coagulation cascade?
Single best answer question choose ONE true option only
VII Your answer
VIII
IX
XI
XII
The clotting cascade is the ordered stepwise enzyme-controlled activation of soluble clotting
factors to produce an insoluble fibrin mesh; a thrombus. There are two different pathways,
intrinsic and extrinsic. The intrinsic pathway is so called as all the elements necessary for its
activation are in the blood. It is triggered by exposure of collagen in damaged vascular
endothelium. The extrinsic pathway requires the release of tissue factors from damaged tissues
to start the process. Both pathways converge in the common pathway. Sequential activation of
factors XII, XI, IX and VIII comprises the intrinsic pathway. The extrinsic pathway involves tissue
factor and activated factor VII. Both the intrinsic and extrinsic pathways activate factor X, II and I
to form fibrin and this is the common pathway.
Correct
A 75-year-old woman undergoes total gastrectomy for carcinoma of stomach.

With which of the following nutrients is she most likely to require parenteral replacement? Single
best answer - choose ONE true option only

Ascorbic acid

Folic acid

Iron

Vitamin B12
Your answer
Vitamin D

No significant absorption of nutrients takes place in the stomach. However, because of the lack
of secretion of pepsin, and hence reduced activation of pancreatic proenzymes, and the fact that
the ability to eat normal amounts of food may be greatly decreased, patients who have had total
gastrectomies may require general nutritional supplementation, eg with proprietary high-energy,
high-protein liquids. However, the absorption of vitamin B12, although it takes place in the
terminal ileum, is critically dependent on the availability of intrinsic factor, which is only secreted
by the parietal (oxyntic) cells of the stomach.
Correct
Reply to HalaReport

Post #10
Hala Adel wrote5 hours ago
A 25-year-old man is admitted to hospital with persistent vomiting. He is clinically dehydrated
and hypotensive. His serum sodium concentration is 124 mmol/l, potassium 4.9 mmol/l, urea 9.8
mmol/l, creatinine 96 mmol/l. Urine sodium concentration in a specimen passed on admission is
62 mmol/l.

Which of the following is the most likely cause of the hyponatraemia?


Single best answer question choose ONE true option only

Adrenal failure
Your answer
Cerebral salt wasting

Gastrointestinal fluid loss

Low sodium intake

Syndrome of inappropriate antidiuresis (SIAD)

Natriuresis in a dehydrated, hyponatraemic patient suggests that there is uncontrolled renal loss
of sodium, such as occurs in adrenal failure. Cerebral salt wasting can also cause dehydration
and hyponatraemia due to excessive natriuresis, but typically occurs following a head injury or
brain surgery. Hyponatraemia and dehydration due to gastrointestinal fluid loss or sodium
deficiency due to a low intake should lead to renal conservation of sodium. Although SIAD is an
important cause of hyponatraemia and sodium excretion may be high, the hyponatraemia is due
to water excess and patients are not dehydrated.
Incorrect
Which of the following statements regarding precautions with using colloids is true?

Single best answer question choose ONE true option only


Dextrans do not carry a risk of anaphylaxis

Dextrans are less likely to interfere with blood cross-matching than starches
Your answer
Gelatins are less likely to cause pruritis or anaphylaxis than starch solutions

Haemaccel and blood are compatible through the same IV cannula

Colloids may worsen peripheral oedema


Correct answer
Dextrans (e.g. Dextran 40 or 70) comprise solutions of multiply-branched polysaccharides. They
carry a risk of anaphylaxis, interfere with blood cross-matching and may reduce platelet
adhesion.

Gelatins (e.g. Gelofusin and Haemaccel) are formed from the hydrolysis of bovine collagen.
They are much more likely than starch-based colloid solutions to cause pruritis or anaphylaxis. In
addition, the calcium content of Haemaccel can cause blood to clot if infused through the
same cannula.

All colloids may worsen peripheral oedema if there is loss of capillary wall integrity with resultant
leak of the colloid into the interstitial fluid compartment.
Incorrect
With respect to vomiting which of the following statements is the best answer?
Single best answer question choose ONE true option only
Chief cells

The CTZ is outside the blood brain barrier


Correct answer
5HT3 agonists may be effective in controlling cisplatin induced vomiting

H2 receptors are abundant in the vomiting centre

The vomiting centre is present in the reticular formation of the mid brain
Your answer
The vomiting centre is present in the reticular formation of the medulla, the CTZ is outside the
blood brain barrier and the main receptors are dopaminergic D2 receptors. 5HT3 antagonist is
effective in controlling vomiting. H1 receptors have been identified in the vomiting centre.
Correct
You review a 39-year-old sportsman who complains of knee pain. Arthroscopy reveals damage to
the cartilage.

Which of the following stems best describes a property of hyaline cartilage?


Single best answer question choose ONE true option only

It has a blood supply from small arterioles

It is rich in type 1 collagen

Chondrocytes secrete collagen only

It is avascular
Your answer
Pressure from normal joint loading accelerates damage to cartilage

Hyaline cartilage forms the articular surface and is avascular, relying on diffusion from synovial
fluid for nutrients. It is rich in type II collagen and forms a meshwork containing proteoglycan
molecules that retain water. Intermittent pressure from joint loading is essential to maintain
normal cartilage function. Chondrocytes secrete proteoglycans and collagen and are embedded
in the cartilage. They migrate to the joint surface along with the matrix that they produce.
Incorrect
Flow through a vessel or lumen is:

Single best answer question choose ONE true option only


Is inversely proportional to the pressure head of flow

Is inversely proportional to the radius


Your answer
Is directly proportional to the length of the tube

Is directly proportional to the viscosity of blood passing through it

Is directly proportional to the fourth power of radius


Correct answer

The Hagen-Poiseuille law states that the flow through a vessel is:
* Directly proportional to the pressure head of flow
* Directly proportional to the fourth power of radius
* Inversely proportional to the viscosity
* Inversely proportional to the length of the tube

The radius of the tube is therefore the most important determinant of flow through a blood
vessel. Thus, doubling the radius of the tube will lead to a 16-fold increase in flow at a constant
pressure gradient. The implications of this are several fold.

First, owing to the fourth power effect on resistance and flow, active changes in radius constitute
an extremely powerful mechanism for regulating both the local blood flow to a tissue and central
arterial pressure. The arterioles are the main resistance vessels of the circulation and their
radius can be actively controlled by the tension of smooth muscle within its wall.

Second, in terms of intravenous fluid replacement in hospital, flow is greater through a


peripheral cannula than through central lines. The reason is that peripheral lines are short and
wide (and therefore of lower resistance and higher flow) compared to central lines, which are
long and possess a narrow lumen. A peripheral line is therefore preferential to a central line
when urgent fluid resuscitation, or blood, is required.
Correct
In estimating the physiological clearance of 10 ml of an intravenous substance which has been
administered at 10 mg/ml, the plasma concentration at equilibration is 15 mg/litre, the urine
concentration is 150 mg/litre and the subject produces 1440 ml of urine during a 24h collection.

What is the clearance of the substance? Single best answer question choose ONE true option

only

1 ml/min

10 ml/min
Your answer
0.1 ml/min

100 ml/min

Cannot say from the information given

Clearance is calculated using the formula (U V)/P where U = urine concentration in mg/ml, V =
urine production in ml/min, P = plasma concentration in mg/ml.
The bolus size of the substance is irrelevant to the clearance.
Incorrect
How much of 1 litre of 5% dextrose infused intravenously will remain in the intravascular
compartment?

Single best answer question choose ONE true option only


250 mls Your answer
400 mls

<100 mls
Correct answer
500 mls

333.33 mls

5% dextrose has no oncotic properties (the dextrose is absorbed) and therefore 1 litre of 5%
dextrose will be distributed equally amongst the total body water. 1/3 of total body water is
extracellular and 2/3 intracellular. In addition, around of extracellular fluid is intravascular and
therefore only 1/12th (1/3 x ) of infused 5% dextrose will remain in the intravascular space.
In comparison of 0.9% NSaline will remain in the intravascular space as it contains 154
mmols/l of Na+ which is similar to the concentration Na+ found in the extracellular
compartment.

Correct
What is the site of action of antidiuretic hormone (ADH) in a nephron?
Single best answer question choose ONE true option only
Proximal convoluted tubule
Ascending limb of loop of Henle
Descending limb of loop of Henle
Distal convoluted tubule
Collecting duct Your answer
ADH is produced by the posterior pituitary gland in response to reduced extracellular osmolality,
blood volume and blood pressure. It promotes reabsorption of water from the collecting ducts,
resulting in reduced osmolality and expanded blood volume.
Incorrect
Which immunoglobulin can fix complement via the alternative pathway?
Single best answer - choose ONE true option only

IgA
Correct answer
IgM

IgG
Your answer
IgE

IgD

IgA is unusual in that it can fix complement via the alternative pathway. IgG and IgM can fix
complement via the classical pathway through the Fc portion of the immunoglobulin.
Correct
By which process are particles moved along a concentration gradient across a selectively
permeable membrane?

Single best answer question choose ONE true option only

Endocytosis

Diffusion
Your answer
Exocytosis

Osmosis

Phagocytosis

Fat-soluble molecules, such as glycerol, can diffuse through the membrane easily. They dissolve
in the phospholipid bilayer and pass through it in the direction of the concentration gradient,
from a high concentration to a low concentration. Water, oxygen and carbon dioxide can also
diffuse through the bilayer, passing easily through the temporary small spaces between the tails
of the phospholipids.
Incorrect
Some 24 hours after sustaining major trauma in a road traffic accident, a 22-year-old man, not
known to have diabetes, is found to have a high blood glucose concentration.

Increased secretion of which of the following substances is most likely to be responsible? Single
best answer - choose ONE true option only
Adrenaline (epinephrine)
Correct answer
Cortisol
Your answer
C-reactive protein

Growth hormone

Insulin

During the metabolic response to trauma, there is increased secretion of catecholamines,

cortisol, glucagon and growth hormone. The first three of these tend to increase blood glucose
concentration; catecholamines, cortisol and glucagon act directly, whereas growth hormone
appears to potentiate the action of cortisol and opposes the action of insulin. Adrenaline and
glucagon act most rapidly, by stimulating glycogenolysis; cortisol tends to act more slowly,
through the stimulation of gluconeogenesis. Insulin is a hypoglycaemic hormone. C-reactive
protein is a marker of inflammation, but does not affect glucose homeostasis.
Incorrect
Calcitonin
Increases plasma calcium levels
Promotes osteoclastic bone resorption
Increases renal excretion of phosphate Correct answer
Is produced in the parathyroid glands
Deficiency causes osteoporosis Your answer
Calcitonin is produced by thyroid C cells. Total thyroidectomy (absent calcitonin) has no
significant skeletal effects. Plasma calcitonin levels rise with increasing serum calcium.
Calcitonin inhibits osteoclastic bone resorption and increases renal excretion of calcium and
phosphate.
Correct
The 3' 5' exonuclease activity possessed by some DNA polymerases that enables the enzyme
to replace misincorporated nucleotide is called what?

Single best answer question choose ONE true option only

Proofreading
Your answer
Replication

Recombination

Retrotransposition

Splicing

Retrotransposition is transposition via an RNA intermediate (transposition is the movement of a


genetic element from one site to another in a DNA molecule). Splicing is the removal of introns
from the primary transcript of a discontinuous gene.
Incorrect
An 81-year-old, nursing-home resident is admitted to hospital in an unconscious state. His blood
sugar is measured as 1.5 mmol/l (normal 36 mmol/l). You administer glucagon.

Which of the following best describes one of the main actions of glucagon?
Single best answer question choose ONE true option only

Decreased ketone body production from fatty acids

Increased lipogenesis in adipose tissue

Decreased glycogenolysis

Decreased gluconeogenesis
Your answer
Increased glycogenolysis and gluconeogenesis
Correct answer
Glucagon is produced by pancreatic islet cells and its main action is on the liver to promote
glycogenolysis and gluconeogenesis. It also increases lipolysis in adipose tissue and increases
ketone body production from fatty acids. The actions of glucagon on adipose tissue are mediated
by cyclic AMP to stimulate lipolysis, producing free fatty acids that can act as a major alternative
energy source. Catecholamines act in a similar way to glucagon, but in addition have effects on
muscle. Insulin promotes the synthesis of glycogen, protein and fat, inhibiting lipolysis and
gluconeogenesis.
Incorrect
What is the average daily volume of gastric secretions (ml per day)?

Single best answer question choose ONE true option only


500

1,000

1,500
Your answer
2,000
Correct answer

2,500

Approximate average fluid secretion volumes (ml per day) for each of the component parts of
the adult human gastrointestinal tract are given below:-

Secretion
ml/day
Saliva
1,500
Gastric
2,000
Bile
500
Pancreatic
1,500
Small intestinal
1,500

The vast majority of this secreted fluid is reabsorbed by the small intestine.
Incorrect
Increased venous return to the heart is most likely to be caused by
Single best answer question choose ONE true option only
Deep inspiration
Correct answer
Forced expiration
Your answer
Hypovolaemia

Positive pressure ventilation

Tension pneumothorax

Blood returns to the heart from the lower limbs via the action of the calf muscle pumps, valves in
the veins of the leg, and the effect of negative intra-thoracic pressure generated during
inspiration. Anything causing the intra thoracic pressure to become less negative will decrease
the venous return to the right atrium. Tension pneumothorax, positive pressure ventilation and
forced expiration all cause this effect, and therefore reduce the venous return. Although
hypovolaemia may cause vasoconstriction in an attempt to increase venous return, it is unlikely
to increase above normal levels.
Correct
A 34-year-old woman with a body mass index of 44 kg/m2 seeks medical help for her obesity.

Which one of the following treatments offers her the highest probability of achieving a long-term
reduction in weight? Single best answer question choose ONE true option only

An energy-deficient diet (600 kcal/day (~ 143 J/day) less than requirements) for 6 months

Jaw-wiring and milk feeding for 3 months

Treatment with orlistat for 12 months

Treatment with sibutramine for 12 months

Vertical banded gastroplication


Your answer
Both sibutramine and orlistat have been shown to induce and maintain a greater weight loss
than diet alone, but a patients weight often plateaus before adequate weight loss has occurred.
Orlistat is only licensed for use for 1 year in the UK, and sibutramine for 2 years. Energydeficient diets, particularly if coupled with increased exercise, are effective, but the lost weight is
almost invariably regained, as it is after jaw-wiring and milk feeding. Surgery offers the best
chance of achieving long-term weight loss, the results from vertical banded gastroplication
combined with a by-pass procedure being even better than those with gastroplication alone.
Correct
A 15-year-old youth with haemophilia A has suffered recurrent bleeding episodes into his joints.
As a consequence he has arthropathies in his knees, elbows and wrists.

What is the most likely coagulation deficiency causing his bleeding tendency?
Single best answer question choose ONE true option only

Thromboxane

Factor X

Protein C

Factor IX

Factor VIII
Your answer

Deficiency of either factor VIII (haemophilia A) or factor IX (haemophilia B), which together make
up the factor VIIIa/factor IXa intrinsic tenase enzymatic complex, results in the clinical phenotype
commonly known as haemophilia. Haemophilia principally presents with haematoma formation,
easy bruising and bleeding at the site of venepuncture during the toddler period.
The disease exists in severe, moderate and mild forms. These are classified as such on the basis
of a clinical laboratory blood coagulation test, which is performed to assess the level of
functional coagulant protein (per cent activity of factor VIII or factor IX). The pathological
problem in both haemophilia A, factor VIII deficiency and haemophilia B, factor IX deficiency
(also called Christmas disease) is the inability to form a functional tenase complex to activate
factor X to factor Xa.
The clinical features of haemophilia predominantly include bleeding into joints and soft tissues.
The incidence of central nervous system bleeding has dramatically decreased with concentrate
therapy. The life expectancy of people with severe haemophilia had increased from 11 years at
the beginning of the twentieth century to approximately 60 years in the early 1980s, before the
devastating effects of blood-borne viral disease again shortened average life expectancy.
Correct
A 21-year-old male medical student who has been feeling non-specifically unwell for several
days is noticed to have slightly icteric sclerae by his girlfriend and has liver function tests
performed. The results of these are normal apart from a serum bilirubin concentration of 44
mmol/l (317). His urine does not contain bilirubin.

Which of the following is the most likely diagnosis? Single best answer question choose ONE
true option only

DubinJohnson syndrome

Gilberts syndrome
Your answer
Hereditary spherocytosis

Infectious mononucleosis

Rotor syndrome

DubinJohnson, Rotor and Gilberts syndromes are all inherited disorders of bilirubin metabolism.
However, in the first two, there is a defect in the secretion of bilirubin from the liver and the
bilirubin that accumulates in the plasma is conjugated, water-soluble and thus is excreted in the
urine.
Infectious mononucleosis can cause hepatitis and jaundice but an elevated transaminase activity
would be expected. Hereditary spherocytosis is a chronic haemolytic disorder due to a defect in
the red cell membrane (most frequently in spectrin, a structural protein). It can present with a
wide range of severity, from jaundice at birth to asymptomatic anaemia or jaundice in adults,
but is much less common (approximately 1:5000 in Northern Europeans) than Gilberts
syndrome (approximately 1:20).
Correct
Reply to HalaReport

Post #11
Hala Adel wrote5 hours ago
Which of the following is the most important causative factor in the development of obesity in
the majority of patients?

Single best answer question choose ONE true option only

Energy intake in excess of expenditure


Your answer

Genetic predisposition

Insulin resistance

Intrauterine malnutrition

Leptin deficiency

The ultimate cause of obesity is always an intake of energy in excess of expenditure, but many
factors govern both intake and expenditure. There is undoubtedly a genetic predisposition in
some individuals. Intrauterine malnutrition may be important in others. Leptin deficiency is a
very rare cause of obesity: more frequently, obese individuals are resistant to the actions of
obesity. Insulin resistance is probably a consequence of obesity, not a cause.
Correct
The oxygenhaemoglobin dissociation curve is shifted to the left by which of the following
factors? Single best answer - choose ONE true option only

Rise in pH
Your answer
Rise in 2,3-DPG (2,3-diphosphoglycerate)

Rise in plasma temperature

Rise in blood CO2 content

Fall in plasma bicarbonate concentration

All the above shift the dissociation curve to the right, with the exception of a rise in pH.
Correct
Which of the following cells secretes intrinsic factor?

Single best answer question choose ONE true option only


Goblet cells

Kupffer cells

Peptic cells

Chief cells

Parietal cells
Your answer
Goblet cells are mucus-secreting cells, widely distributed throughout epithelial surfaces, but
especially dense in the gastrointestinal and respiratory tracts.

Kupffer cells have phagocytic properties and are found in the liver. They participate in the
removal of ageing erythrocytes and other particulate debris.

The gastric mucosa contains many cell subtypes, including acid-secreting cells (also known as
parietal or oxyntic cells), pepsin secreting cells (also known as peptic, chief or zymogenic cells)
and G-cells (gastrin-secreting cells). Peptic cells synthesise and secrete the proteolytic enzyme,
pepsin. Parietal cells actively secrete hydrochloric acid into the gastric lumen, accounting for the
acidic environment encountered in the stomach. However parietal cells are also involved in the
secretion of the glycoprotein, intrinsic factor.

Intrinsic factor plays a pivotal role in the absorption of vitamin B12 from the terminal ileum.
Autoimmune damage to parietal cells leads to a lack of intrinsic factor and hydrochloric acid,
leading to vitamin B12 deficiency and achlorhydria. This is known as pernicious anaemia.
Pernicious anaemia is associated with a 3-fold increase in gastric cancer risk.
Correct
Nociception (pain)
Single best answer question choose ONE true option only
Is transmitted faster through C fibers than through A delta fibers

Pain impulse received in the dorsal horn can be modulated by other descending spinal inputs
Your answer
Opioids act on receptors in the peripheral nerves

Side effects of opioids can be reversed by neostigmine

Glycine is excitatory pain neurotransmitter

Pain impulse received by dorsal horn can be modulated by other ascending and descending
spinal inputs (Gate Theory). Pain is transmitted faster in myelinated A delta fibers, opioids act on
and ? opioid receptors in the central nervous system and their effects can be reversed by
naloxone. Glycine is an inhibitory neurotransmitter.
Incorrect
A 24-year-old woman undergoes resection of the terminal ileum with fashioning of an ileostomy
for Crohns disease. Some 2 weeks after surgery, she is making a good recovery, and is eating a
high-energy, low-residue diet, but has a high ileostomy volume, necessitating intravenous fluid
replacement. Her serum calcium concentration is 1.82 mmol/l, phosphate 1.28 mmol/l, alkaline
phosphatase 82 U/l (normal < 150), albumin 30 g/l, creatinine 80 m mol/l. Prior to surgery, her
serum calcium concentration was 2.18 mmol/l, albumin 36 g/l.

What is the most likely cause of her hypocalcaemia? Single best answer question choose ONE
true option only

Formation of insoluble calcium salts in the intestine


Your answer
Hypoalbuminaemia

Hypomagnesaemia
Correct answer
Malabsorption of calcium

Malabsorption of vitamin D

Impaired fat absorption can lead to the formation of insoluble calcium salts in the gut. Fat and
calcium are absorbed in the proximal small intestine, so, too, is vitamin D. Although bile salts are
absorbed distally, and impaired absorption can lead to a secondary decrease in proximal fat
absorption, this is unlikely to be responsible for hypocalcaemia developing so quickly. The
normal alkaline phosphatase level also militates against vitamin D deficiency. Hypocalcaemia
would normally be expected to stimulate parathyroid hormone secretion and cause the plasma
phosphate concentration to fall (PTH is phosphaturic). Patients with ileostomies can lose large
amounts of magnesium through their stomas; hypomagnesaemia impairs PTH secretion and can
cause hypocalcaemia that is resistant to an increased provision of calcium.
Correct
A patient is found to have hyponatraemia. Which condition should be excluded by subsequent
investigations?

Single best answer question choose ONE true option only

Diabetes insipidus

Syndrome of inappropriate antidiuretic hormone secretion (SIADH)


Your answer
Diabetes mellitus

Conns syndrome

Cushings syndrome

SIADH causes excess water retention over sodium retention by promoting water reabsorption in
the collecting ducts of the kidneys. This results in hyponatraemia. Diabetes insipidus and
diabetes mellitus may both cause hypernatraemia by resulting in excess water loss over Na loss.
In contrast, Conns syndrome and Cushing syndrome cause hypernatraemia by promoting excess
sodium retention over water retention.
Correct
A 2-day-old male infant is referred for a surgical opinion after his parents bring him into the
Emergency Department with abdominal distension and what his parents describe as green
vomiting. Which of the following would NOT be on your list of differential diagnoses?
Single best answer question choose ONE true option only
Duodenal Atresia
Hirschprungs disease
Pyloric stenosis Your answer
Malrotation
Meconium Ileus
Pyloric stenosis usually presents between 3-12 weeks of age. The vomit is NON-BILIOUS due to
the high level of obstruction (the thickened pyloric muscle), which is above the entrance of the
common bile duct into the duodenum. Distension is not usually a feature. Bilious vomiting in a
newborn is a surgical emergency until proved otherwise. Malrotation is the diagnosis, which
needs exclusion with an upper GI contrast to look at the layout of the intestine, specifically
whether the duodenal-jejunal flexure is on the correct side of the vertebrae the LEFT is the
correct side.
Correct
The cardiovascular effects of raised intracranial pressure include:

Single best answer question choose ONE true option only


decreased blood pressure, decreased heart rate, decreased cerebral perfusion pressure

decreased blood pressure, increased heart rate, decreased cerebral perfusion pressure

increased blood pressure, increased heart rate, decreased cerebral perfusion pressure

increased blood pressure, decreasd heart rate, decreased cerebral perfusion pressure
Your answer
decreased blood pressure, increased heart rate HR, increased cerebral perfusion pressure

The important relationship between the cerebral perfusion, mean arterial blood pressure and
intracranial pressure is as follows:

CPP = MABP ICP, where CPP = cerebral perfusion pressure


MABP = mean arterial blood pressure
ICP = intracranial pressure

It stems from the fact that the adult brain is enclosed in a rigid, incompressible box, with the
result that the volume inside it must remain constant (Monroe-Kelly doctrine). A rise in
intracranial pressure therefore decreases cerebral perfusion pressure (and hence cerebral blood
flow).

In raised intracranial pressure, as the brainstem becomes compressed, local neuronal activity
causes a rise in sympathetic vasomotor drive and thus a rise in blood pressure. This is known as
the Cushings reflex. This elevated blood pressure evokes a bradycardia via the baroreceptor
reflex. The Cushings reflex helps to maintain cerebral blood flow and protect the vital centres of
the brain from loss of nutrition if the intracranial pressure rises high enough to compress the
cerebral arteries.
Correct
With regard to CO2 transported in the blood, most of the CO2 is
Single best answer question choose ONE true option only
Dissolved in plasma

In the form of carbamino compounds formed from plasma proteins

In the form of carbamino compounds formed from haemoglobin

Bound to Chloride

In the form of HCO3Your answer


Carbon dioxide is transported in three main ways:

Carbamino compounds between CO2 and proteins. Most of these reactions are with the globin
portion of haemoglobin, accounting for 20-30% of the transported CO2.
Dissolved CO2 accounts for about 10% of the transported CO2.
HCO3- accounts for about 60-70% of the transported CO2.
Correct
The glomerular filtration rate is increased by?
Single best answer choose ONE true option only

Increased plasma colloid osmotic pressure


Constriction of the glomerular afferent arterioles
Constriction of the glomerular efferent arterioles Your answer
Saline depletion
Respiratory alkalosis
Constriction of the glomerular efferent arterioles increases the hydrostatic pressure within the
glomerulus and hence the filtration pressure.
Correct
Following a decrease in core body temperature, what causes a rise in circulating plasma
thyroxine?

Single best answer question choose ONE true option only


An increase production of thyroxine by the thyroid gland

A decrease in renal excretion of thyroxine

Release of thyrotrophin releasing hormone from the hypothalamus


Your answer
Release of thyroid stimulating hormone from the anterior pituitary gland

An increase in iodine absorption from the intestines

The hypothalamus is thought to be the control centre for thermoregulation. Studies have shown
that cooling the hypothalmic area in the brain will result in an increase in the secretion of
thyrotrophin releasing hormone from the hypothalamus. This in turn will result in an increase in
the secretion of thyroid stimulating hormone from the anterior pituitary gland which acts directly
on the thyroid gland to increase the secretion of thyroxine. Thyroxine counters a decrease in
body temperature by increasing the cellular metabolic rate in a process that can take several
weeks and can result in hypertrophy of the thyroid gland.
Correct
You are asked to see a patient who had a chest drain removed 4 days ago. There appears to be
some infection.

What are the stages in the cell biology of normal wound healing?
Single best answer question choose ONE true option only

Demolition is the first phase

Maturation and remodelling can continue for up to a year


Your answer

Acute inflammation usually lasts for 612 hours

Epithelial cell proliferation is the hallmark of the demolition phase

Collagen deposition is the key process during demolition

The first phase in healing by first intention is the phase of acute inflammation that lasts up to 3
days, if uncomplicated. The initiating factor appears to originate from platelets activated by
mature collagen exposed in the wound. Platelets first aggregate then release a variety of active
agents including lysosomal enzymes, ATP, serotonin and wound cytokines. A fibrin clot develops,
which completes haemostasis and provides strength and support to the wound. The surface
dries to form a scab. Platelets and macrophage factors cause local vasodilatation, which
produces warmth and increases capillary permeability, allowing serum and white blood cells to
accumulate and cause swelling.
After the initial acute inflammation, macrophages become active as the main agents of
demolition, removing unwanted fibrin, dead cells and bacteria and creating fluid-filled spaces for
granulation tissue. Macrophages also release factors that stimulate the formation of new
capillary buds during this phase, and later they initiate and control fibroblast activity during
repair. Within the connective tissue, randomly orientated collagen begins to form after a few
days, reaching a peak of activity after 57 days.
Epithelial cells at the edge of the wound start to proliferate after 24 h and this phase can last for
up to 3 weeks.
Finally, the phase of maturation and remodelling lasts for up to 12 months, during which time
the tensile strength of the wound increases and the random collagen is replaced by a more
stable form orientated along lines of stress.
Correct
A 25-year-old patient suffered recurrent deep vein thromboses and also one pulmonary
embolism. She was extensively investigated and diagnosed with protein C deficiency.

What pathological process is most likely to be responsible for her venous thromboembolisms?
Single best answer question choose ONE true option only

Reduced degradation of factors Va and VIIIa


Your answer
Reduced factor Xa complex

Reduced inhibition of tissue-factor expression

Reduced protein S

Reduced synthesis of antithrombin III

Protein C acts to inactivate the active forms of the procoagulant cofactors, factors Va and VIIIa.
Protein C is a vitamin K-dependent serine protease structurally similar to factors VII, IX and X.
Thrombin activates protein C when bound to thrombomodulin, a protein which acts like an
endothelial-cell receptor for thrombin. Symptomatic manifestations of protein C deficiency are
similar to those of antithrombin III deficiency. Deep venous thrombosis, with or without
pulmonary embolism, occurs in 50% of patients by the a
Reply to HalaReport

Post #12
Hala Adel wrote5 hours ago
MRCS Part 1 Practice Questions ( Physiology ) - 3 of 3
Correct
Which of the following physiological abnormalities occurs as a direct consequence of septic
shock?

Single best answer question choose ONE true option only


A decrease in cardiac output

A decrease in systemic vascular resistance


Your answer
A decrease in vascular permeability

A decrease in intravascular volume

An increase in cardiac contractility

Septic shock is defined as shock (decreased tissue perfusion resulting in end-organ dysfunction)
secondary to a demonstrable source of infection most commonly bacterial in origin. Exotoxin in
bacterial cell walls results in the production of cytokines and other inflammatory mediators that
reduce vasuclar tone and increase vascular permeability. This in turn can result in a loss of
intravascular fluid across capillaries and intravascular volume depletion as a secondary event.
Cardiac output can both increase and decrease in septic shock due to the normal physiological
response to a decreased blood pressure or cardiac dysfunction caused by circulating cytokines
and inflammatory mediators respectively.
Correct

Bile salt reuptake principally occurs in the:

Single best answer question choose ONE true option only


Duodenum

Jejenum

Ileum
Your answer
Colon

Caecum

90-95% of the bile salts are absorbed from the small intestine and then excreted again from the
liver; most are absorbed from the terminal ileum. This is known as the enterohepatic circulation.
The entire pool recycles twice per meal and approximately 6-8x per day.

Disruption of the enterohepatic circulation, either by terminal ileal resection or through a


diseased terminal ileum (e.g. Crohns disease), results in decreased fat absorption and
cholesterol gallstone formation. The latter is believed to result because bile salts normally make
cholesterol more water-soluble through the formation of cholesterol micelles. Loss of reuptake
also results in the presence of bile salts in colonic contents, which alters colonic bacterial growth
and stool consistency.
Incorrect
A 24-year-old, unconscious man is admitted to A&E. No history is available. The results of arterial
blood gas analysis are: [H+] 80 nmol/l (pH 7.1), p(CO2) 7.0 kPa, p(O2) 8.2 kPa, [HCO3] 17.1
mmol/l.

These results indicate which one of the following acidbase disturbances?


Single best answer question choose ONE true option only
Metabolic acidosis with respiratory compensation

Mixed metabolic and respiratory acidosis


Correct answer
Respiratory acidosis
Your answer
Respiratory acidosis with metabolic alkalosis

Uncompensated metabolic acidosis

The high hydrogen-ion concentration (low pH) indicates acidosis. The elevated p(CO2) indicates
a respiratory component; in compensated metabolic acidosis, p(CO2) is reduced; in an
uncompensated metabolic acidosis (a very unusual situation, since the respiratory response to a
metabolic acidosis is usually a rapid one), it would be normal. The hydrogen-ion concentration is
too low to be accounted for by a respiratory acidosis alone: there must therefore be a metabolic
acidosis in addition (as the low bicarbonate concentration also indicates).
Incorrect
Concerning Glomerular Filtration, which of the following is true of the proximal convoluted
tubule?
Single best answer question choose ONE true option only
Reabsorbs water by sodium secretion
Reabsorbs phosphate Correct answer
Increases the volume of reabsorbed fluid under aldosterone stimulation
Contains renin-secreting cells
Receives most of its blood supply from the vasa recta Your answer
The proximal convoluted tubule actively reabsorbs sodium. This sets up an osmotic gradient and
water is drawn out of the tubule. It is the site of both phosphate and calcium reabsorption under
control of parathyroid hormone. Aldosterone acts on the distal convoluted tubules. Renin is
secreted by the cells of the juxtaglomerular apparatus in the distal convoluted tubules.
Incorrect
Which of the following do not normally occur as a response to a decrease in core body
temperature?

Single best answer question choose ONE correct option only


Bradycardia

Vasocontriction

A decrease in CNS metabolism

A reduction in plasma catecholamine levels


Correct answer
A rise in plasma thyroxine
Your answer
The hypothalamus and the lower brain stem are the most important neural structures that
regulate body temparature. A fall in core body temparature is associated with a decrease in CNS
activity and can result in bradycardia secondary to depression of cardiac pacemaker cells. The
bodys response to a fall in temparature includes shivering, peripheral vasoconstriction and the
release of metabolic factors including thyroxine, cortisol and catecholamines.
Incorrect
A 72-year-old woman is found to have a serum calcium concentration of 3.12 mmol/l. Which of
the following clinical features, if present, would most direct you towards a specific cause?

Single best answer question - choose ONE true option only

Bone pain
Your answer
Hilar lymphadenopathy
Correct answer
Polyuria

Short QT interval

Ureteric colic

Bone pain can occur with hypercalcaemia secondary to malignancy or hyperparathyroidism.


Polyuria is a feature of severe hypercalcaemia, irrespective of the cause. A short QT interval is
also a feature of hypercalcaemia. Ureteric colic is particularly associated with primary
hyperparathyroidism, but is not specific to this cause. The presence of hilar lymphadenopathy in
a patient with hypercalcaemia should raise a suspicion that the latter is due to sarcoid (in which
the granulomas secrete calcitriol, 1,25-dihydroxycholecalciferol).
Correct
In the respiratory system, physiological shunt?

Single best answer question choose ONE true option only


Is greater than the anatomical shunt
Your answer
Is not present in healthy adult

Affects arterial carbon dioxide more than arterial oxygen tension

Has the same effect on respiratory gas exchange as does physiological dead space

Is abolished when the subject breathes pure oxygen

The physiological shunt is the sum of the anatomical shunt (blood passing from the right
ventricle to the systemic circulation via normal anatomical pathways, e.g. the bronchial vessels,
without passing through the pulmonary alveolar capillaries), and the element of pulmonary
alveolar capillary blood that has passed through non or poorly aerated alveoli. Therefore
physiological shunt is always at least as great as or greater than the anatomical shunt.

There is always a normal anatomical shunt even in the young healthy adult.

The difference in carbon dioxide tension between arterial and mixed venous blood is a little less
than 1 kPa, and therefore even a 50% shunt only increases arterial carbon dioxide tension by
about 0.5 kPa. A 50% shunt would reduce arterial oxygen tension from 13.5kPa to below 9 kPa.

The physiological dead space results primarily in a failure to remove carbon dioxide from
alveolar gas, i.e. a rise in arterial carbon dioxide tension if ventilation not increased.

The breathing of pure oxygen cannot eliminate the anatomical right to left portion of the
physiological shunt.
Correct
A 70-year-old male complains of constantly feeling cold and lethargic. What is the most likely
hormonal deficiency to account for this?

Single best answer question choose ONE true option only


Somatostatin

Cholecystokinin

Testosterone

Thyroxine
Your answer
Insulin

Thyroxine deficiency is the most likely cause, particularly if other features of hypothyroidism are
present such as fatigue, weight gain, dry skin and hair, slowly-relaxing reflexes and non-pitting
oedema. Amongst other roles, thyroxine is important in regulating basal metabolic rate and body
heat production.
Testosterone deficiency is likely to result in loss of libido and secondary sexual characteristics.
Insulin deficiency may present with features of diabetes mellitus, such as thirst, polyuria and
polydipsia. Somatostatin and cholecystokinin are important gastrointestinal regulatory peptides.
Correct
A patient has the following urea and electrolytes results:
Sodium 140 mmol/l
Potassium 4 mmol/l

Chloride 105 mmol/l


Bicarbonate 20mmol/l
Calculate the anion gap.

Single best answer - choose ONE true option only

19 meq/l
Your answer
5 meq/l

10 meq/l

30 meq/l

0 meq/l

Anion gap = ([Na+] + [K+]) - ([Cl] + [HCO3]) (all units mmol/l).

Normal range is 816 meq/l


Correct
The largest contribution to systemic vascular resistance (SVR) is made by the
Single best answer question choose ONE true option only
Aortic valve

Great arteries

Arterioles
Your answer
Venules

Great veins

The capillaries and arterioles each account for around 25% of the SVR. The large surface area of
the capillaries, as well as the low flow and pressure drop through the capillary beds is vital to
their function in exchange of gases and nutrients. The arterioles have abundant smooth muscle
in their walls, and flow is regulated to a large degree by the sympathetic nervous system. They
therefore exert a great deal of control over the flow through the capillary beds, as well as which
capillary beds are open at a given time.
Incorrect
Within normal physiological limits, which of the following factors does not influence cardiac
stroke volume?

Single best answer question choose ONE true option only


Preload

Afterload
Correct answer
Heart rate
Your answer
Cardiac sympathetic nerve activity
Reply to HalaReport

Post #13
Hala Adel wrote5 hours ago
Myocardial contractility

Increases in preload (up to a limit) increase stroke volume by Starlings Law of the heart.
Starlings Law also indicates that increases in afterload (up to a limit), whilst causing ventricular
stretch and an increase in end-diastolic volume, do not increase stroke volume but instead
maintain it. Increases in myocardial contractility increase the force of contraction during systole
and therefore increase stroke volume. If preload does not become limiting, increases in heart
rate increase myocardial contractility, and therefore stroke volume, via the Bowditch effect (a
rate-related phenomenon thought to be due to accumulation of intracellular calcium in the
cardiomyocytes). Cardiac sympathetic nerve activity increases stroke volume by increasing
cardiac contractility and heart rate.
Correct
A 21-year-old man presents with a 2-day history of persistent vomiting and abdominal pain. His
blood gas shows;
pH 7.44
PaCO2 7.3 kPa
PaO2 12.0 kPa
Base excess +12 mmol/l
HCO3- 38 mmol/l
Cl- 90 mmol/l
What does this blood gas demonstrate?
Single best answer question - choose ONE true option only
Compensated metabolic alkalosis Your answer
Compensated respiratory alkalosis

Uncompensated metabolic alkalosis

Respiratory acidosis

Uncompensated metabolic acidosis

It can be seen that the striking features of this blood gas are a pH within the normal
physiological range with elevated bicarbonate and base excess. The vomiting in this patient has
resulted in the loss of hydrochloric acid and loss of total body H+ concentration, causing a
metabolic alkalosis.
This patient has compensated for this through respiratory hypoventilation and retention of CO2
(an acidic gas). Ultimately the acid-base imbalance cannot be normalised by respiratory
retention of CO2, this is merely a compensatory measure and requires renal modulation of H+
and HCO3- levels
Correct
The action potential of skeletal muscle?
Single best answer question choose ONE true option only
Has a prolonged plateau phase

Spread inwards to all parts of the muscle via the T tubes


Your answer
Causes immediate uptake of Ca into the sarcoplasmic reticulum

Is longer than the action potential of cardiac muscle

Is not essential for contraction

The action potential of the skeletal muscle spreads out from the motor end plate, through the T
tube system this causes mobilization of Ca2+ from the sarcoplasmic reticulum to the cytoplasm
and this action potential is essential for contraction.

The action potential of cardiac muscle is longer than that of the skeletal muscle and has plateau
phase.
Correct
In a starving patient, which of the following fluid regimens would be most appropriate for a 70kg
man over a 24hr period?

Single best answer question choose ONE true option only


3L NSaline with 20mmols potassium chloride in each bag
3L Dextrose-saline

3L Hartmanns solution

1L NSaline with 20 mmols potassium chloride and, 2L 5% dextrose with 20mmols potassium
chloride in each bag
Your answer
3L 5% dextrose with 20mmols potassium chloride in each bag
The daily fluid and electrolyte requirements are 1-1.5 mmols Na+ /Kg/24 hours, 1mmols K+
/Kg/24 hours and 40ml H20 /Kg/24 hours.
However, additional fluid should be supplemented if there are 3rd space losses (that commonly
occur for instance in severe acute pancreatitis, burns and post major gastro-intestinal surgery)
and for other sources of fluid loss including vomiting, diuresis and insensible losses.
Correct
Gastric acid secretion is stimulated by:

Single best answer question choose ONE true option only


Somatostatin

Gastrin
Your answer
Secretin

The glossopharyngeal nerve

Cholecystokinin

Gastric acid is stimulated by 3 factors:


* Acetylcholine: From parasympathetic neurones of the vagus nerve that innervate parietal cells
directly.
* Gastrin: produced by pyloric G-cells.
* Histamine: Produced by mast cells. This stimulates the parietal cells directly and also
potentiates parietal cell stimulation by gastrin and neuronal stimulation. H2 blockers such as
ranitidine are therefore an effective way of reducing acid secretion.

Gastric acid is inhibited by 3 factors:


* Somatostatin
* Secretin
* Cholecystokinin
There are 3 classic phases of gastric acid secretion:

* Cephalic (preparatory) phase [significant]: Results in the production of gastric acid before food
actually enters the stomach. Triggered by the sight, smell, thought and taste of food acting via
the vagus nerve.
* Gastric phase [most significant]: Initiated by the presence of food in the stomach, particularly
protein rich food.
* Intestinal phase [least significant]: The presence of amino acids and food in the duodenum
stimulate acid production.

Incorrect
The rate at which a liquid meal leaves the stomach is?
Single best answer question choose ONE true option only
Greater in the upright than in the supine position

Proportional to the volume of stomach content


Correct answer
Greater if the meal contains fat

Slower if the meal is 5% glucose than if it is 50% glucose

Slower if vagotomy and drainage procedure (such as gastroenterostomy or pyloroplasty) has


been performed
Your answer
Gastric emptying accelerates on lying down. The rate of gastric emptying at any moment is
proportional to the volume present in the stomach at that moment

When the fat reaches the duodenum it stimulates mixed hormonal and vagal mechanisms that
slow the rate of stomach emptying.
An isotonic meal will empty at maximal rate but osmotically stronger or weaker solutions will
empty more slowly.

Vagotomy may temporarily slow gastric emptying, but its long term effect is to increase the rate
of gastric emptying or leave it un changed so if a drainage procedure is accompanied by
vagotomy there will be a tendency towards accelerating gastric emptying.
Correct
During digestion of a fatty meal, which hormone causes contraction of the gall bladder and
relaxation of the sphincter of Oddi?
Single best answer question choose ONE true option only
Cholecystokinin
Your answer
Gastrin

Insulin

Secretin

Somatostatin

Cholecystokinin secretion from the duodenal and jejunal mucosa is stimulated by the presence
of fatty acids, amino acids and peptides in the lumen of the duodenum and jejunum. As well as
causing contraction of the gall bladder and relaxation of the sphincter of Oddi, it stimulates
release of pancreatic enzymes, and increases the secretin mediated secretion of HC03- by
pancreatic duct cells. Its release is inhibited by somatostatin.
Correct
In a breathless patient, a pleural effusion with less than 3g of protein per 100ml of fluid is most
likely to be caused by
Single best answer question choose ONE true option only
Bronchial carcinoma

Mitral regurgitation
Your answer
Pneumonia

Tuberculosis

Tricuspid regurgitation

An effusion with less than 3g of protein per 100ml is a transudate. Other biochemical
characteristics of a transudate include LDH < 200 iU/l, WCC < 1000/ml, glucose mmol/l.
Transudative effusions are most commonly due to factors such as decompensated liver failure
and left ventricular failure. Malignancy and infection are causes of an exudative pleural effusion.
Pulmonary embolism can cause either an exudative or transudative effusion, although the
former is more common.

In this question, B is more likely than E to be associated with left ventricular failure, and
therefore a pleural effusion. Tricuspid regurgitation is usually functional and secondary to an
enlarged right ventricle in right ventricular failure, and causes a pulsatile liver, peripheral
oedema and ascites.
Correct
High titres of antithyroid microsomal and antithyroglobulin antibodies would suggest which of
the following diagnoses in a patient presenting with a complaint of tiredness?

Single best answer - choose ONE true option only

Hashimotos thyroiditis
Your answer
Reidels thyroiditis

Graves disease

Hypoparathyroidism

Idiopathic hypothyroidism

This finding in Hashimotos thyroiditis is characteristic, but lower titres can occur in Reidels
thyroiditis and Graves disease. High titres of these antibodies in euthyroid individuals indicate
the possibility of future thyroid failure, but this may be many years away; hence the need for
thyroid function tests every 12 years in such individuals.
Correct
Which of the following constituents is NOT present in Hartmanns solution?

Single best answer question choose ONE true option only


Na+

Cl-

K+

Lactate

HCO3Your answer
The composition of Hartmanns solution is as follows:Na+ = 131 mmol/l
Cl- = 111 mmol/l
K+ = 5 mmol/l

Ca2+ = 2 mmol/l
Lactate = 29 mmol/l

Therefore, the osmolality of Hartmanns solution is (131+111+5+2+29) = 278 mmol/l.


The lactate present in the solution is metabolised in vivo to form HCO3-. Bicarbonate is not
added to Hartmanns solution since this would result in the precipitation of calcium carbonate in
the storage container.
Incorrect
What is the main method by which intracellular pH is regulated?

Single best answer question choose ONE true option only


The bicarbonate buffer system
Your answer
The phosphate buffer system

Cytoplasmic proteins
Correct answer
Carbonic anhydrase

The globin component of haemoglobin

Cytoplasmic proteins provide the main contribution to pH buffering of the intracellular


compartment.

In the interstitial (i.e. extracellular and extravascular) compartment, the bicarbonate system is
the main mechanism of pH buffering.

In the intravascular (plasma) compartment, pH buffering mechanisms include:-

The bicarbonate buffer system: CO2 + H2O H2CO3 H+ + HCO3- catalysed by the enzyme
carbonic anhydrase
The phosphate buffer system: HPO42- + H+ H2PO4 Plasma proteins
The globin component of haemoglobin
Incorrect
A 60-year-old obese smoker has been admitted to hospital with chest pain due to unstable
angina. A nitrate infusion is started to relieve his chest pain.

Which blood vessels are most sensitive to the vasodilatatory effect of nitrates?

Single best answer question choose ONE true option only

Large arteries

Coronary arteries
Your answer
Capillaries

Large veins
Correct answer
Pulmonary arteries

The antianginal and haemodynamic effects are mediated predominantly by vasodilatation of the
venous system, leading to a fall in left ventricular preload and cardiac work.
Correct
A patient on total parenteral nutrition (TPN) regimen presents with drowsiness and abnormal
serum electrolytes.

What is the most likely cause? Single best answer - choose ONE true option only

Hypocalcaemia

Hypercalcaemia

Hypernatraemia

Hypophosphataemia
Your answer
Hypomagnesaemia

Administering carbohydrate lowers serum phosphate by stimulating the release of insulin, which
moves phosphate and glucose into cells. This so-called refeeding syndrome occurs when
starving or chronically malnourished patients are re-fed or given intravenous (iv) glucose.
Phosphate deficiency commonly impairs neurological function, which may be manifested by
confusion, seizures, and coma. Peripheral neuropathy and ascending motor paralysis, similar to
GuillainBarr syndrome, may also occur. Weakness of skeletal or smooth muscle is the most
common clinical manifestation of phosphate deficiency. It can involve any muscle group, alone or
in combination, ranging from ophthalmoplegia to proximal myopathy, to dysphagia or ileus.
Respiratory insufficiency may occur in some patients with severe hypophosphataemia,
particularly when the underlying cause is malnourishment. Impaired cardiac contractility occurs,
leading to generalised signs of myocardial depression. The hypophosphataemic myocardium
also has a reduced threshold for ventricular arrhythmias.
Incorrect
A 60 kg man suffers 20 % burns. What is the estimated volume of intravenous fluid replacement
that should be administered in the first 8 hours from the time of the burn?

Single best answer question choose ONE true option only


800 1,000 ml

1,000 1,200 ml

1,200 2,400 ml
Correct answer
2,400 4,800 ml
Your answer
4,800 5,000 ml

Intravenous fluids (crystalloid or colloid) should be administered if burns of greater than 15 % in


adult or 10 % in paediatric patients are present. The rate of fluid administration ultimately
employed is dependent on clinical indices, such as urine output, capillary refill and peripheral
perfusion, central venous pressure and core:peripheral temperature differentials.

Various formulae are available for estimating initial rates of intravenous fluid replacement in
burns victims. These initial rates of fluid administration are then modified based on clinical
response.

Two widely-used formulae are as follows:-

Parkland formula : 2 4 ml/kg/%burn (full or deep partial thickness) in first 24 h from time of
burn. Half of this calculated volume (crystalloid) should be administered in the first 8 h and the
remainder administered in the subsequent 16 h.

Mount Vernon Formula This formula subdivides fluid administration into discrete time periods
over the first 24 h: 4, 4, 4, 6, 6 and 12 h from the time of burn. The amount of fluid (colloid)

administered in each of these periods is calculated as: (patient weight in kg x %burn)/2


Correct
A 3-week-old baby exhibits projectile vomiting shortly after feeding and failure to thrive. On
examination an olive-shaped mass is palpable in the right upper quadrant of the abdomen. A
clinical diagnosis of pyloric stenosis is made. What biochemical laboratory features would
support the diagnosis?

Single best answer question choose ONE true option only


Hypokalaemia, metabolic alkalosis, low urinary pH
Your answer
Hyperkalaemia, metabolic acidosis, high urinary pH

Hypokalaemia, metabolic acidosis, high urinary pH

Hyperkalaemia, metabolic alkalosis, low urinary pH

Hypokalaemia, metabolic alkalosis, high urinary pH

Following a diagnosis of pyloric stenosis, the first concern is to correct the metabolic
abnormalities that invariably coexist with the condition. The serum electrolytes and capillary
gases should be measured and corrected prior to surgery.

With prolonged vomiting, the infant becomes dehydrated, with a hypochloraemic metabolic
alkalosis. The alkalosis is a result of loss of unbuffered hydrogen ions in gastric juice with
concomitant retention of bicarbonate.

Fluid loss stimulates renal sodium reabsorption, but sodium can only be reabsorbed either with
chloride, or in exchange for hydrogen and potassium ions (to maintain electroneutrality). Gastric
juice has a high concentration of chloride and patients losing gastric secretions become
hypochloraemic. This means that less sodium than normal can be reabsorbed with chloride.

However, it appears that the defence of extracellular fluid volume takes precedence over acidbase homeostasis and further sodium reabsorption occurs in exchange for hydrogen ions
(perpetuating the alkalosis) and potassium ions (leading to potassium depletion). This explains
the apparently paradoxical finding of acidic urine in patients with pyloric stenosis. Potassium is
also lost in the gastric juice and thus patients frequently become potassium-depleted and yet
are losing potassium in their urine.
Correct
A 75-year-old woman is being followed by her GP for suspected developing primary
hypothyroidism.

Which of the following biochemical changes would you most expect to occur first?
Single best answer question choose ONE true option only

Fall in serum free thyroxine

Fall in serum thyroxine-binding globulin

Fall in serum free triiodothyronine

Fall in serum total triiodothyronine

Increase in serum TSH


Your answer
Hypothyroidism develops gradually, often over many months or even years. In the early stages,
free thyroxine concentrations are maintained in the normal range by the increased secretion of
TSH. Patients with a slightly elevated TSH and lownormal thyroxine are said to have
compensated or borderline hypothyroidism. In some individuals, it appears that this state can
be maintained without progression to frank hypothyroidism. Triiodothyronine concentrations tend
to fall later than thyroxine concentrations in hypothyroidism; the concentration of thyroxinebinding globulin does not change significantly.
Incorrect
The acute blood loss of 1.5 liters leads to a decrease in?
Single best answer question choose ONE true option only
The rate of oxygen extraction by the peripheral tissues

Renin secretion

Platelet count
Your answer
The cardiac output
Correct answer
Coronary and cerebral blood flow due to sympathetic overactivity

The rate of oxygen extraction by the peripheral tissues is increased in response to acute blood
loss, renin secretion is also increased due to renal hypoperfusion.

Platelet count is increased and cardiac output decreased as the stroke volume decreases.
The blood flow to the brain and the heart remains unchanged.
Correct
A patient receives too many infusions after an operation resulting in a 20% increase in his blood
volume.

What is the physiological process that is most likely to correct this abnormality?
Single best answer question choose ONE true option only

Reduced activity of arterial pressure sensors

Increased activity of renal sympathetic nerves

Aldosterone release

Atrial natriuretic peptide (ANP) release


Your answer
Venous dilatation

The atria contain granulated cells that release peptides, atrial natriuretic peptide (ANP), in
response to stretch. This natriuretic agent also relaxes the peripheral vasculature and thereby
opposes the actions of the sympathetic and reninangiotensin systems.
Correct
A 54-year-old woman has undergone some blood tests as part of an employment health screen.
She reports she is in good health and, being very health conscious, takes regular vitamin and
mineral supplements. She is taking bendrofluazide 2.5 mg for hypertension and her blood
pressure is 132/82 mmHg. The only abnormality is a serum calcium concentration of 2.94
mmol/l.

Which of the following is the most likely cause? Single best answer question choose ONE true
option only

Diuretic treatment

High dietary calcium intake

High dietary vitamin D intake

Occult malignancy

Primary hyperparathyroidism
Your answer
Thiazides can cause hypercalcaemia but it is usually only mild. Vitamin D itself is physiologically
inactive and, whereas 1-hydroxylated derivatives can be a cause of hypercalcaemia, vitamin D
which has to be metabolised to activate it is less commonly so. Intestinal absorption of calcium
is subject to tight control, and a high intake does not cause hypercalcaemia. The two most
common causes of hypercalcaemia are primary hyperparathyroidism and malignancy. In an
asymptomatic individual, primary hyperparathyroidism is the more likely cause.
Correct
A 56-year-old woman sustains a myocardial infarction. ST elevation and Q waves are present in
leads V4V6, I and AVL.

Which of the following aspects of the heart is most likely to have been involved in the infarct?
Single best answer question choose ONE true option only
Anterior

Anterolateral
Your answer
Anteroseptal

Inferior

Lateral

This combination suggests an anterolateral infarct. Purely anterior infarcts tend to involve the

chest leads only (typically V2V5), anteroseptal V1V3, lateral infarcts chest leads only (I, II, AVL)
and inferior infarcts II, III and AVF.
Correct
A parathyroid adenoma will be most likely to cause
Single best answer question choose ONE true option only
Decreased osteoclastic activity

Decreased urinary phosphate excretion

Hypocalcaemia

Increased osteoblastic activity

Increased osteoclastic activity


Your answer
The parathyroid glands produce parathyroid hormone (PTH) in response to serum calcium levels
via a negative feedback mechanism. High levels of serum Ca2+ inhibit PTH secretion, and low
levels stimulate PTH secretion. The response to Ca2+ levels is very rapid, so effects are seen
very quickly after removal of the glands.

PTH affects calcium levels by its action on the bone, kidney and gut.

In bone, increased osteoclastic activity causes calcium levels to rise. This is due firstly to acid
secretion onto the bone surface, and secondly to proteases dissolving the matrix.

In the kidney, PTH controls the hydroxylation of 25,hydroxy cholecalciferol D to 1,25 hydroxy
cholecalciferol. This has the indirect effect of increasing calcium uptake in the gut. In the
proximal tubule, PTH increases the urinary excretion of phosphate, which in turn increases the
ionisation of calcium. There is also an increase in Ca2+ reabsorption in the distal tubule.
Bicarbonate resorption is inhibited in the kidney, causing a hyperchloraemic acidosis which
increase calcium ionisation and resorption from bone.

PTH excess therefore causes hypercalcaemia, hypophosphataemia and hyperchloraemia, as well


as raised urinary phosphate.
Correct
A 20-year-old man presents with mild jaundice following a flu-like illness. Following review by a
gastroenterologist, he has been told that a diagnosis of Gilberts syndrome is probable.

Which laboratory test is most likely to confirm this diagnosis?


Single best answer question choose ONE true option only

Absence of bilirubin in the urine


Your answer
Decreased serum haptoglobin concentration

Elevated serum aspartate aminotransferase (transaminase, AST) activity

Increased reticulocyte count

Increased urinary urobilinogen excretion

In Gilberts syndrome, the excess bilirubin is unconjugated, and does not appear in the urine.
The same is true for jaundice secondary to haemolysis. However, in haemolytic jaundice, urinary
urobilinogen is increased (increased production of bilirubin, and hence of urobilinogen), the
reticulocyte count may be elevated and serum haptoglobin concentration decreased. Haemolysis
may also cause a slight increase in serum aminotransferase (transaminase) activity.
Correct
Which of the following physiological responses occur in an acute hypoglycaemic episode?

Single best answer question choose ONE true option only


A rise in serum insulin

A decrease in liver glycogen

A decrease in serum glucagon

A rise serum adrenaline


Your answer
A rise in serum ketone bodies
Acute hypoglycaemia commonly occurs in insulin dependant diabetic patients who fail to match
their carbohydrate intake with their insulin dose. It also occurs in patients with beta cell
pancreatic tumours (insulinoma) due to a pathological overproduction of insulin.
The acute response to hypoglycaemia is the result of an increase in serum adrenaline, glucagon
(both of which are gluconeogenic) and due to a lack of glucose available for the brain (termed
neuroglycopenia). These result in flight or fright symptoms, the feeling of hunger and a variety
of neurological symptoms including blurred vision, slurred speech and impaired mental function.
Correct
Which of the following is not a well recognised feature of excessive glucocorticoid levels?

Single best answer question choose ONE true option only


Hypertension
Hyperglycaemia
Alopecia
Your answer
Acne
Reply to HalaReport

Post #14
Hala Adel wrote5 hours ago
Osteoporosis
Cortisol and its analogues are glucocorticoids and levels are raised either endogenously in
Cushings disease, or exogenously causing Cushings syndrome. There are numerous side effects
of glucocorticoid excess. Hypertension as a result of increased renal reabsorption of sodium and
water. Hyperglycaemaia as a result of mineralocorticoid activity. Acne and hirsutism, not
alopecia are a result of androgenic activity. Other side effects are osteoporosis, weakened skin,
muscle wasting, immunosuppression and increased rates of infection, cataracts and fat
redistribution to give the moon face and buffalo hump appearance.
Correct
Which of the following is not typically a cause of hypercalcaemia?

Single best answer question choose ONE true option only

Hyperparathyroidism

Hypothyroidism
Your answer
Milkalkali syndrome

Sarcoid

Squamous-cell carcinoma

Two of commonest causes of hypercalcaemia in the western world are primary


hyperparathyroidism and malignancy. In primary hyperparathyroidism there is excess production
of parathyroid hormone (PTH); although usually from a benign adenoma, this sometimes results
from hyperplasia of the parathyroid glands and, in rare cases, a carcinoma. Thyrotoxicosis can
cause hypercalcaemia as well as osteoporosis. The milkalkali syndrome can occur in patients

who suffer from dyspepsia and drink milk and alkali-containing antacids, which may reduce the
renal excretion of calcium. Around one-fifth of those with sarcoid have increased calcium levels.
Various mechanisms cause raised hypercalcaemia of malignancy.
Correct
Which of the following is the site of renin production?
Single best answer question choose ONE true option only
Collecting ducts
Proximal convoluted tubule
Loop of Henle
Juxtaglomerular apparatus Your answer
Liver
The juxtaglomerular apparatus is formed of specialised juxtaglomerular cells in the wall of
afferent arterioles and macula densa of the distal convoluted ducts. Renin secretion is stimulated
by reduced renal perfusion. Angiotensinogen is produced by the liver and is catalysed by renin to
form angiotensin I. This is in turn catalysed by angiotensin converting enzyme (ACE) to produce
angiotensin. Angiotensin has several functions which aim to increase blood pressure and restore
renal perfusion. It causes vasoconstriction, stimulates the adrenal cortex to produce aldosterone
which promotes renal reabsorption of sodium and water from the distal convoluted tubules and
collecting ducts.
Incorrect
Hypothyroidism due to disease of the thyroid gland is associated with increased plasma level of?
Single best answer question choose ONE true option only
Cholesterol
Correct answer
Albumin

RT3
Your answer
Iodide

Thyroid binding globulin (TBG)

Thyroid hormone lowers circulating cholesterol level. The plasma cholesterol level drops before
the metabolic rate rises.
Incorrect
Which one of the following is MOST likely to increase during exercise?

Single best answer question choose ONE true option only

Peripheral vascular resistance

Pulmonary vascular resistance

Stroke volume
Correct answer
Diastolic pressure

Venous compliance
Your answer
During exercise, increased oxygen consumption and increased venous return to the heart result
in an increase in cardiac output and an increase in blood flow to both skeletal muscle and
coronary circulation, when oxygen utilization is greatest. The increase in cardiac output is due to
an increase in both heart rate and stroke volume. Systemic arterial pressure also increases in
response to the increase in cardiac output. However, the fall in total peripheral resistance, which
is caused by dilatation of the blood vessels within the exercising muscles, results in a decrease
in diastolic blood pressure. The pulmonary vessels undergo passive dilatation as more blood
flows into the pulmonary circulation. As a result, pulmonary vascular resistance decreases. The
decrease in venous compliance, caused by sympathetic stimulation, helps to maintain
ventricular filling during diastole.
Correct
The function of luteinising hormone in the male is:
Promotion of spermatogenesis
Stimulation of testosterone secretion Your answer
Promotion of adrenal androgen secretion
Stimulation of Sertolis cells to produce inhibin
Promotion of spermiogenesis
Follicle-stimulating hormone (FSH) and testosterone are required for spermatogenesis (division
of spermatogonia to form spermatids) and spermiogenesis (maturation of spermatids to mature
sperm). FSH also stimulates Sertolis cells to produce androgen-binding proteins and inhibin.
Adrenal androgen secretion is not affected by luteinising hormone.
Incorrect
Pulmonary gas exchange occurs under which of the following physiological principles? Single
best answer question choose ONE true option only
Gas exchange can occur in the final seven branches of the bronchoalveolar tree
Correct answer
The first 12 branches of the bronchial tree are collectively known as the conducting zone

The equilibration of gases takes about 2.5 s in the resting lung

Only about 0.15% of oxygen is carried in solution in the plasma

Carbon dioxide is less water-soluble than oxygen

Gas exchange can occur in the final seven branches of the bronchoalveolar tree (the respiratory
zone). The first 16 branches of the bronchial tree are collectively known as the conducting zone.
The equilibration of gases takes about 0.25 s in the resting lung. Only about 1.5% of oxygen is
carried in solution in the plasma. Carbon dioxide is more water-soluble than oxygen, between 5
and 10% of and this is the predominant method of carriage of CO2 is carried in dissolved form.
Correct
Reply to HalaReport

Post #15
Hala Adel wrote5 hours ago
Splenectomy increases susceptibility to which of the following organisms?

Single best answer question choose ONE true option only


Streptococcus pyogenes

Schistosoma haematobium

Bacteroides fragilis

Neisseria meningitidis
Your answer
Staphylococcus aureus

The spleen plays an important role in the removal of dead and dying erythrocytes and in the
defence against microbes. Removal of the spleen (splenectomy) leaves the host susceptible to a
wide array of pathogens, but especially to encapsulated organisms.

Certain bacteria have evolved ways of evading the human immune system. One way is through
the production of a slimy capsule on the outside of the bacterial cell wall. Such a capsule resists
phagocytosis and ingestion by macrophages and neutrophils. This allows them not only to
escape direct destruction by phagocytes, but also to avoid stimulating T-cell responses through
the presentation of bacterial peptides by macrophages. The only way that such organisms can
be defeated is by making them more palatable by coating their capsular polysaccharide
surfaces in opsonising antibody.

The production of antibody against capsular polysaccharide primarily occurs through T-cell
independent mechanisms. The spleen plays a central role in both the initiation of the antibody
response and the phagocytosis of opsonised encapsulated bacteria from the bloodstream. This
helps to explain why the asplenic individuals are most susceptible to infection from encapsulated
organisms, notably Streptococcus pneumoniae (pneumococcus), Neisseria meningitidis
(meningococcus) and Haemophilus influenzae.

The risk of acquiring such infections is reduced by immunising individuals against such
organisms and by placing patients on prophylactic penicillin, in most cases for the rest of their
lives. In addition, asplenic individuals should be advised to wear a MedicAlert bracelet to warn
other health care professionals of their condition.
Correct
Which ECG feature is classically present in hypothermia?
Single best answer question choose ONE true option only
Thyroxine

Reduced PR interval

Tachycardia

U waves

J waves
Your answer
The J wave may be present on the ECG in patients with hypothermia and is an additional upward
peak immediately following the QRS complex. The U wave may be present on the ECG in
hypokalaemia and is an additional upward peak which follows the T wave. Tachycardia and a
reduction in the RR interval are ECG features of hyperthermia.
Incorrect
Which of the following is a function of atrial natruretic peptide (ANP)?
Single best answer question choose ONE true option only
Increases renin secretion
Decreases aldosterone secretion Correct answer
Promotes the effects of antidiuretic hormone (ADH) Your answer
Causes renal vasoconstriction
Promotes the feeling of thirst

ANP is released from atrial muscle cells when the atria are stretched due to increased circulating
blood volume. Therefore ANP works to reduce blood volume by inhibiting the release of renin,
aldosterone and ADH resulting in increased sodium and water excretion. It promotes renal
vasodilatation.
Correct
Concerning the salivary glands
Single best answer question choose ONE true option only
They secrete around 150 ml of saliva per day

They secrete saliva with a pH of 4-5

They secrete saliva which is hypertonic

They are supplied by the parasympathetic nervous system

Your answer
They secrete saliva containing trypsinogen

Saliva is secreted from the acini, and transported via the salivary ducts to the oral cavity. The
secretion from the sublingual gland is predominately mucous, the parotid serous and the
submandibular mixed. The pH of saliva varies from 7-8, and around 1.5L is produced per day. As
well as a-amylase, saliva contains lipase and glycoproteins to lubricate food and protect the oral
mucosa. Lysozyme, IgA and lactoferrin act as bacteriostatic agents, and proteins protect the
tooth enamel.

The saliva is isotonic when it is excreted from the acini; Na+ and Cl- are exchanged for K+ and
HC03- in the ducts, and the saliva becomes hypotonic by the time it reaches the mouth.
Correct
Which of the following is the most important direct stimulus to respiration?
Single best answer question choose ONE true option only
Increased pCO2 of the CSF

Increased H+ concentration of the CSF


Your answer
Decreased arterial pO2

Decreased arterial pH

Decreased arterial pCO2

Chemoreceptors involved with the control of respiration are present in the central nervous
system and peripherally. The central chemoreceptors are situated in the ventral medulla, and
increase firing in response to the H+ concentration of the brain extra cellular fluid, which is
directly related to the H+ concentration in the CSF. CO2 / HCO3 cannot cross the blood brain
barrier, but CO2 does so readily. This frees H+ ions, causing a low CSF pH, increased firing of the
central chemoreceptors and increased ventilation.

Peripheral chemoreceptors are found in the carotid bodies and aortic arch, and increase their
firing rate in response to decreased PaO2, decreased arterial pH and increased paCO2. These are
much less important, however, in stimulating respiration than the central chemoreceptors.
Correct
Vasopressin (ADH)
Single best answer question choose ONE true option only
Is synthesised in the posterior pituitary gland

Deficiency leads to a risk of water intoxication

Excessive secretion usually results in diabetes insipidus

Increased plasma osmolarity is the primary physiological stimulus


Your answer
Acts on the proximal convoluted tubules of the kidney

Vasopressin is synthesised in the supraoptic nucleus of the hypothalamus and transported to the
posterior pituitary via the axons. Excessive secretion is associated with the risk of impaired
water excretion. Diabetes insipidus results from deficient secretion or action of this hormone
leading to thirst and polyuria. It acts mainly on the distal convoluted tubules and the collecting
ducts of the kidney.
Incorrect
The following metabolic changes occur in the ebb phase (first 24 hours) of response to injury?
Single best answer question choose ONE true option only
Plasma pH increases

The plasma level of free fatty acids decreases

Hypoglycemia

The plasma level of non protein nitrogen decreases


Your answer
Plasma glycerol increases
Correct answer
There is usually acidosis (pH decreases). Lipolysis increases leading to increase in fatty acids and
glycerol.
There is hyperglycemia and an increased level of non protein nitrogen.
Incorrect
Which of the following fluids would be the most appropriate to replace the fluid being lost in a
patient with a paralytic ileus draining 2 litres of fluid a day through a nasogastric tube?

Single best answer question choose ONE true option only

Compound sodium lactate (Hartmanns solution)


Your answer
5% dextrose

10% dextrose

0.18% sodium chloride with 4% dextrose (dextrose saline)

0.9% sodium chloride (normal saline)


Correct answer
In this situation, it is essential to supply sufficient chloride ions to replace the chloride being lost
in the gastric fluid (gastric juice is essentially dilute hydrochloric acid). If this is not done, a
metabolic alkalosis can ensue. The appropriate fluid is normal saline. The two dextrose
solutions contain no chloride, and dextrose saline contains insufficient for this purpose.
Hartmanns solution could exacerbate any tendency to alkalosis as the lactate it contains is
metabolised to bicarbonate.
Correct
Which one of the following is higher at the apex of the lung than at the base when a person is
standing? Single best answer question choose ONE true option only

V/Q ratio
Your answer
Ventilation

PaCO2

Compliance

Blood flow

The alveoli at the apex of the lung are larger than those at the base so their compliance is less.
Because of the reduced compliance, less inspired gas goes to the apex than to the base. Also,
because the apex is above the heart level, less blood flows through the apex than through the
base. However, the reduction in air flow is less than the reduction in blood flow, so that the V/Q
ratio at the top of the lung is greater than it is at the bottom. The increased V/Q ratio at the apex
makes PaCO2 lower and PaO2 higher at the apex than they are at the base
Correct
A 49-year-old postmenopausal woman of Southern Asian origin complains of muscle weakness.
She is found to have hypocalcaemia, and X-ray examination reveals two Loosers zones in her
left upper femur.

A defect in which of the following physiological processes is most likely to be the cause of her
illness? Single best answer question choose ONE true option only.

Absorption of calcium from the gut


Your answer
Osteoblastic activity

Osteoclastic activity

Parathyroid hormone secretion

Renal excretion of calcium

The findings in this woman suggest osteomalacia, and the most important reason for the
impaired mineralisation of bone is reduced intestinal calcium absorption consequent on vitamin
D deficiency. The decreased availability of calcium to mineralise bone leads to increased
osteoblastic activity (and hence increased osteoid formation). Hypocalcaemia causes increased
parathyroid hormone secretion (secondary hyperparathyroidism), which stimulates renal calcium
reabsorption (hence reduced excretion). Thus, while this woman may have increased
osteoblastic activity and increased PTH secretion, both these are secondary to vitamin D
deficiency and decreased intestinal absorption of calcium.
Osteoporosis (postmenopausal osteoporosis is due to increased osteoclastic activity) is not
associated with hypocalcaemia.
Incorrect
In relation to the nutritional physiology of patients, which of the following would represent
appropriate nitrogen requirements (g N/kg per day) and calorie requirements (kcal/kg per day)?

Single best answer question choose ONE true option only

Reduced food intake: nitrogen requirement 0.3 g N/kg per day, calorie requirement 35 kcal/kg
per day
Your answer
Moderate injury: nitrogen requirement 0.15 g N/kg per day, calorie requirement 25 kcal/kg per

day

Moderate sepsis: nitrogen requirement 0.3 g N/kg per day, calorie requirement 15 kcal/kg per
day

Severe injury: nitrogen requirement 0.3 g N/kg per day, calorie requirement 35 kcal/kg per day
Correct answer
Severe sepsis: nitrogen requirement 0.2 g N/kg/day, calorie requirement 15 kcal/kg/day

Usual ranges for:

reduced food intake:


nitrogen requirement 0.150.2 g N/kg per day
calorie requirement 2530 kcal/kg per day
moderate injury/sepsis:
nitrogen requirement 0.20.3 g N/kg per day
calorie requirement 3035 kcal/kg per day
severe injury/sepsis:
nitrogen requirement 0.30.35 g N/kg per day
calorie requirement 3540 kcal/kg per day
Correct
You are called to see a 56-year-old man 2 h after a cardiac catheterisation. He is actively
bleeding from his catheter site and his dressings and bedclothes are soaked with blood.

Which of the following statements is true? Single best answer - choose ONE true option only

Grade I shock applies with up to a 20% loss of circulating blood volume

Loss of 2 litres of blood is consistent with normal systolic blood pressure

The pulse can remain normal in patients with grade I shock


Your answer
Anuria is pathognomonic of grade III shock

Grade IV shock is seen with a 30% loss of circulating blood volume

Grade I shock
Loss of up to 15% (750ml) of blood volume; blood pressure is normal but there may be a slight
tachycardia
Grade II shock
1530% (750 ml 1.5 l) blood-volume loss, systolic blood pressure is usually normal but a
tachycardia is present
Grade III shock
3040% (1.52 litres) loss, hypotension, tachycardia and fall in urine output seen
Grade IV shock
> 40% (> 2 l) blood-volume loss, anuria and severe shock observed
Correct
Formation of the external genitalia in the male fetus is dependent on:
Testosterone
Adrenal androgens
The Y chromosome
Dihydrotestosterone Your answer
Mullerian inhibiting substance
Dihydrotestosterone is the most potent androgen and is responsible for development of the
external genitalia in the f
Incorrect
Lung compliance:

Single best answer question choose ONE true option only


Is defined as the change in pressure per unit volume

Your answer
Is synonymous with elastance

Is increased in emphysema
Correct answer
Is equal in inflation and deflation

Is reduced by the presence of surfactant

Compliance is expressed as volume change per unit change in pressure. Elastance is the
reciprocal of compliance. The pressure-volume curve of the lung is non-linear with the lungs
becoming stiffer at high volumes. The curves which the lung follows in inflation and deflation are
different. This behaviour is known as hysteresis. The lung volume at any given pressure during
deflation is larger than during inflation. This behaviour depends on structural proteins (collagen,
elastin), surface tension and the properties of surfactant.

Surfactant is formed in and secreted by type II pneumocytes. The active ingredient is dipalmitoyl
phosphatidylcholine. It helps prevent alveolar collapse by lowering the surface tension between
water molecules in the surface layer. In this way it helps to reduce the work of breathing (makes
the lungs more compliant) and permits the lung to be more easily inflated.

Various disease states are associated with either a decrease or increase in the lung compliance.
Fibrosis, atelectasis and pulmonary oedema all result in a decrease in lung compliance (stiffer
lungs). An increased lung compliance occurs in emphysema where an alteration is elastic tissue
is probably responsible (secondary to the long term effects of smoking). The lung effectively
behaves like a soggy bag so that a given pressure change results in a large change in volume
(i.e. the lungs are more compliant). However, during expiration the airways are less readily
supported and collapse at higher lung volumes resulting in gas trapping and hyperinflation.
Incorrect
The infusion of 1 litre of which of the following solutions will initially lead to the greatest increase
in extracellular fluid volume?

Single best answer question choose ONE true option only


Gelatin colloid solution (e.g. Gelofusin or Haemaccel)
Your answer
Hypertonic NaCl
Correct answer
Normal (0.9 %) NaCl

5 % dextrose solution

Pure water

Colloids may be natural (e.g. blood, human albumin and gelatins) or synthetic (e.g. dextrans).
They comprise large branching molecules with molecular weights in excess of 30,000. Assuming
intact capillary integrity, the volume effects of colloid infusion are, at least initially, confined to
the plasma compartment. In contrast, crystalloids, such as NaCl solution, pass more readily from
the plasma fluid compartment and have more of a volume effect on the extracellular fluid
compartment. In the case of 5 % dextrose solution, the dextrose component is rapidly
metabolised and the remaining water distributes itself throughout the entire body water (i.e.
intracellular and extracellular compartments).

Therefore, of the options listed above, infusions of NaCl will have the greatest initial increase in
extracellular fluid volume. Hypertonic NaCl will have an even greater effect than normal
(approximately isotonic) NaCl, since hypertonic solutions will draw additional water from the
intracellular fluid compartment by osmosis.
Correct
Reply to HalaReport

Post #16
Hala Adel wrote5 hours ago
Single best answer question choose ONE true option only
CD4 T-cells

CD8 T-cells
Your answer
B cells

TH1 cells

TH2 cells

Lymphocytes can be divided into two main subtypes T cells and B cells.
B cells (or plasma cells) secrete antibodies.
T cells can be divided into two further subtypes CD4 T-cells and CD8 T-cells. CD4 (helper) Tcells can recognise antigen only in the context of MHC Class II, whereas CD8 (cytotoxic) T-cells
recognise cell-bound antigens only in association with Class I MHC. This is known as MHC
restriction.

CD4 and CD8 T-cells perform distinct but somewhat overlapping functions. The CD4 helper T-cell
can be viewed as a master regulator. By secreting cytokines (soluble factors that mediate
communication between cells), CD4 helper T-cells influence the function of virtually all other
cells of the immune system including other T-cells, B-cells, macrophages and natural killer cells.
The central role of CD4 cells is tragically illustrated by the HIV virus which cripples the immune
system by selective destruction of this T-cell subset. In recent years two functionally different
populations of CD4 helper T-cells have been recognised TH1 cells and TH2 cells, each
characterised by the cytokines that they produce. In general, TH1 cells facilitate cell-mediated
immunity, whereas TH2 cells promote humoral-mediated immunity.

CD8 cytotoxic T-cells mediate their functions primarily by acting as cytotoxic cells (i.e. they are
T-cells that kill other cells). They are important in the host defence against cytosolic pathogens.
Two principal mechanisms of cytotoxicity have been discovered perforin-granzyme-dependent
killing and Fas-Fas ligand dependent killing.
Correct
Reply to HalaReport

Post #17
Hala Adel wrote5 hours ago
MRCS Part 1 Practice Questions ( Anatomy ) - 1 of 2
Here are some questions for revision:
MRCS Part 1 - Anatomy MTF
The right common carotid artery
Incorrect
bifurcates at the level of the upper border of the cricoid cartilage TrueFalse
Incorrect
is a branch of the aortic arch TrueFalse
Incorrect
has the cervical sympathetic chain as an anterior relation TrueFalse
Incorrect
lies lateral to the lateral lobe of the thyroid gland TrueFalse
Incorrect
is separated from the phrenic nerve by the prevertebral fascia TrueFalse
Incorrect
is enclosed within the carotid sheath throughout TrueFalse
The right common carotid artery branches off the brachiocephalic artery. It bifurcates at the level
of the upper border of the lamina of the thyroid cartilage. It lies posterior to the lobes of the
thyroid gland and anterior to both the cervical sympathetic chain and the phrenic nerve on the
scalenus anterior muscle; the latter is separated from the artery by prevertebral fascia.
Recognised complications of sclerotherapy for varicose veins include
Incorrect
trash foot TrueFalse
Incorrect
brown discoloration of the skin TrueFalse
Incorrect
deep vein thrombosis (DVT) TrueFalse
Incorrect
ulceration of the skin TrueFalse
Incorrect
Sudeck's dystrophy TrueFalse
Extravasation of the sclerosing agent may cause skin damage and ulceration. Patients should be
warned about the possibility of brown pigmentation of the skin. Sclerotherapy is indicated for
residual and recurrent varicosities after varicose vein surgery. Sudeck's atrophy is a recognised
complication of trauma.
The superior mesenteric artery
Incorrect
supplies the entire ileum and jejunum TrueFalse

Incorrect
lies to the left of the inferior mesenteric artery TrueFalse
Incorrect
passes posterior to the splenic vein TrueFalse
Incorrect
lies to the right of the superior mesenteric vein TrueFalse
Incorrect
crosses anterior to the third part of the duodenum TrueFalse
The superior mesenteric artery supplies the gut from the mid-second part of the duodenum to a
level just short of the splenic flexure of the colon. It is directed downwards behind the splenic
vein and by the pancreas, with the superior mesenteric vein on its right side. It lies anterior to
the third part of the duodenum.
Clinical signs suggestive of a urethral injury include
Incorrect
blood at the external urethral meatus TrueFalse
Incorrect
a butterfly haematoma TrueFalse
Incorrect
Battles sign TrueFalse
Incorrect
high riding prostate TrueFalse
Incorrect
haematuria TrueFalse
The inability to void, an unstable pelvic fracture, blood at the external urethral meatus, a
butterfly haematoma, or a high riding prostate on digital rectal examination (DRE) are
indications for the surgeon to request a retrograde urethrogram to confirm that the urethra is
intact prior to inserting a urethral catheter. In the case of a disrupted urethra a suprapubic
catheter should be inserted.
Reflux oesophagitis
Incorrect
is always present with hiatus hernia TrueFalse
Incorrect
is preferably treated with surgery TrueFalse
Incorrect
if untreated may cause stricturing of the oesophagus TrueFalse
Incorrect
does not require more than alteration of lifestyle to treat TrueFalse
Incorrect
is treated surgically principally by attempting to narrow the gastro-oesophageal junction
TrueFalse
The mainstay of treatment for symptomatic reflux oesophagitis is with acid suppression therapy.
If untreated, structuring is common. Surgery restores adequate oesophageal length (high
pressure) in the abdomen.
The following statements concern the root values of peripheral nerves:
Incorrect
sciatic nerve (L4,5,S1,2) TrueFalse
Incorrect
phrenic nerve (C2,3,4) TrueFalse

Incorrect
iliohypogastric nerve (L1) TrueFalse
Incorrect
obturator nerve (L2,3,4) TrueFalse
Incorrect
medial plantar nerve (L4,5) TrueFalse
The sciatic nerve is L4,5,S1,2,3; and the phrenic nerve C3,4,5.
Pes planus
Incorrect
is the condition where the medial border of the foot is in contact with the ground when standing
TrueFalse
Incorrect
is often caused by a bony bridge between talus and calcaneus TrueFalse
Incorrect
typically presents with pain around age 6 years TrueFalse
Incorrect
can be treated by arthrodesis of thesubtalar and midtarsal joints TrueFalse
Incorrect
may be due to peroneal muscle paralysis TrueFalse
Pes planus = pes valgus = flat foot. The whole foot is rotated into eversion around its
longitudinal axis. It is asymptomatic in the vast majority of cases. There are two types: mobile
and rigid. Rigid flat foot is often caused by synostosis between two of the tarsal bones:
talocalcaneal and talonavicular. There is typically pain and limitation of movement in the foot
around age of 12 years. Triple fusion is sometimes necessary if pain is the predominant feature,
but most symptomatic cases are treated conservatively with splintage or plaster. Spasmodic
flat foot is due to contraction of the peroneal muscles.
Coronary artery bypass grafting is the usual form of treatment for patients with
Incorrect
stable angina and triple vessel coronary artery disease TrueFalse
Incorrect
single or double vessel coronary artery disease TrueFalse
Incorrect
stenosis of the left main coronary artery TrueFalse
Incorrect
post-myocardial infarction unstable angina TrueFalse
Incorrect
valvular heart disease requiring surgery and coronary artery disease TrueFalse
According to the Coronary Artery Surgery Study the patient groups that derive particular benefit
from coronary artery bypass grafting (CABG) are those with triple vessel disease, and those with
>50% left main stem stenosis. Those with single or double vessel disease are usually more
amenable to percutaneous intervention. Post-myocardial infarction, unstable angina is a primary
indication for urgent CABG. Valvular disease with concomitant coronary artery disease is usually
treated operatively.
The thoracic duct
Incorrect
lies on the posterior intercostal vessels TrueFalse
Incorrect
has no valves TrueFalse

Incorrect
runs through the thoracic inlet to the left of the oesophagus TrueFalse
Incorrect
receives the right bronchomediastinal lymph trunk TrueFalse
Incorrect
arches over the left suprapleural membrane TrueFalse
The cisterna chyli runs between the aorta and the right crus of the diaphragm, passes through
the aortic diaphragm opening and drains into the thoracic duct. The thoracic duct ascends
anterior to the posterior intercostal vessels and has several valves. At the thoracic inlet, it lies to
the left of the oesophagus and arches forward over the dome of the left pleura, draining into the
left brachiocephalic vein. The right bronchomediastinal trunk drains into the right subclavian
vein.
Which of the following are correct? The internal carotid artery
Incorrect
Commences at the level of C6 TrueFalse
Incorrect
Passes through the foramen ovale TrueFalse
Incorrect
Has no extra-cranial branches TrueFalse
Incorrect
Gives off the ophthalmic artery TrueFalse
Incorrect
Divides into the middle and anterior cerebral arteries TrueFalse
The common carotid artery bifurcates into the external and internal carotids at the level of the
upper part of the C4 vertebra ie the upper border of the thyroid cartilage, however this
bifurcation is frequently higher, near the tip of the great horn of the hyoid bone (C3 level). The
internal carotid artery has no extra cranial branches and enters the base of the skull in the
petrous temporal bone through the carotid canal. The internal carotid on entering the skull
passes forwards through the temporal bone upwards into the cavernous sinus, turns forward and
upwards through the roof of the sinus to lie medial to the anterior clinoid process before turning
back on itself above the cavernous sinus and then passing once more lateral to the optic
chiasma to end by dividing into the anterior and middle cerebral arteries. The ophthalmic artery
originates from the internal carotid artery immediately above the roof of the cavernous sinus.
Regarding an anterior dislocation of the shoulder:
Incorrect
it commonly occurs after an epileptic fit TrueFalse
Incorrect
it may produce decreased sensation over the lateral aspect of the deltoid muscle TrueFalse
Incorrect
it always needs reduction under a general anaesthetic TrueFalse
Incorrect
it is not associated with any fractures TrueFalse
Incorrect
it occurs less commonly than a inferior dislocation of the shoulder TrueFalse
Dislocation of the shoulder can occur in thee directions: most common is anteriorly, followed by
posteriorly and rarely inferiorly. An anterior dislocation produces a flattening in the deltoid
muscle; the axillary nerve may be injured causing decreased sensation in the lateral aspect of
this muscle (regimental badge area). Anterior dislocations may be associated with a
compressional fracture of the humeral head known as a HillSach deformity.
Posterior dislocation is more difficult to diagnose; this occurs more commonly following seizures
and has a characteristic light bulb appearance due to rotation of the upper end of the humerus.

With reference to complications of total hip arthroplasty, the following are true:
Incorrect
sciatic nerve injury is a recognised complication TrueFalse
Incorrect
loosening is the commonest cause of long-term failure TrueFalse
Incorrect
there is no evidence that prophylactic antibiotics reduce infection rate TrueFalse
Incorrect
unfractionated heparin is more effective in preventing deep vein thrombosis (DVT) than low
molecular weight heparin TrueFalse
Incorrect
uncemented arthroplasty has better survival than cemented arthroplasty TrueFalse
Most dislocations occur within 6 months of surgery and are treated conservatively. Sciatic nerve
injury complicates 1% of cases. Prophylactic antibiotics, gentamicin-impregnated cement and
ultra-clean air enclosures have reduced infection rate. Loosening may be due to absorption of
cement around the implant, hypersensitivity, low grade infection and/or imperfect prosthetic
design, and is found in about 20% of patients 10 years post-operatively. Cemented arthroplasty
is generally considered to be better than uncemented arthroplasty.
The submandibular gland
Incorrect
lies below the digastric muscle TrueFalse
Incorrect
has the hypoglossal nerve running through it TrueFalse
Incorrect
lies both below and above the lower mandible TrueFalse
Incorrect
is superficial to the hyoglossus muscle TrueFalse
Incorrect
has the facial artery running through it TrueFalse
The submandibular gland consists of a deep and a superficial part. The superficial part lies in the
digastric triangle (above and between the two bellies of the digastric muscle). The hypoglossal
nerve runs medial to the superficial part of the gland. The gland is superficial to the mylohyoid
and hyoglossus muscles. A third of the submandibular gland lies below the lower border of the
mandible and two-thirds above it.
Injury to the sciatic nerve in the buttock causes
Incorrect
loss of active extension at the knee joint TrueFalse
Incorrect
loss of strength of the hamstring muscles TrueFalse
Incorrect
complete loss of sensation below the knee TrueFalse
Incorrect
weakness of dorsiflexion at the ankle joint TrueFalse
Incorrect
weakness of eversion of the foot TrueFalse
The sciatic nerve, arising from nerve roots L4, L5, S13. It is really two nerves the tibial, and the
common peroneal nerve which are bound together in the same connective tissue sheath. The
tibial nerve supplies flexor muscles, and the common peroneal nerve supplies extensor and

abductor muscles. The anterior femoral compartment containing the quadriceps extends the
knee and is supplied by the femoral nerve.
Anatomy of the orbit
Incorrect
the supraorbital nerve passes through the superior orbital fissure TrueFalse
Incorrect
the ophthalmic artery passes through the superior orbital fissure TrueFalse
Incorrect
the optic nerve is surrounded by pia, arachnoid and dura mater TrueFalse
Incorrect
the frontal nerve passes through the tendinous membrane TrueFalse
Incorrect
the nasociliary nerve supplies the cornea TrueFalse
Incorrect
sectioning of the inferior ramus of the oculomotor nerve will produce a ptosis TrueFalse
Incorrect
the ophthalmic artery is a branch of the internal carotid artery TrueFalse
The frontal nerve arises from the ophthalmic division of the trigeminal nerve in the lateral wall of
the cavernous sinus. It enters the orbits through the superior orbital fissure. Just before it
reaches the orbital margin it divides into the supratrochlear and supraorbital nerves. The
supraorbital nerve passes through the supraorbital foramen, and supplies the skin of the
forehead. The ophthalmic artery branches off the internal carotid artery at the cavernous sinus,
and passes through the optic canal with the optic nerve. The optic nerve is surrounded by a
sheath of pia, arachnoid and dura mater. The nasociliary nerve arises from the ophthalmic
division of the trigeminal nerve in the lateral fourth of the cavernous sinus, and enters the orbit
through the superior orbital fissure within the tendinous ring. The branches of the nasociliary
nerve supply the ethmoidal sinuses, sphenoidal sinuses, skin of the upper eyelids and nose. The
inferior ramus of the oculomotor nerve gives off branches to the inferior rectus, medial rectus
and the inferior oblique muscles. The superior ramus of the oculomotor nerves supplies the
levator palpebrae superioris, so may give rise to a ptosis if cut.
Derivatives of the mesonephric ducts:
Incorrect
ureters TrueFalse
Incorrect
uterus TrueFalse
Incorrect
prostate TrueFalse
Incorrect
part of the vas deferens TrueFalse
Incorrect
part of the vagina TrueFalse
The epididymis, vas deferens, seminal vesicle, ejaculatory duct and bladder trigone are derived
from the mesonephric duct.
The pancreas
Incorrect
overlies the right kidney TrueFalse
Incorrect
lies in the transpyloric plane TrueFalse
Incorrect
has an uncinate process lying anterior to the superior mesenteric vein TrueFalse

Incorrect
gives attachment to the transverse mesocolon TrueFalse
Incorrect
has the inferior mesenteric vein passing behind the neck TrueFalse
The head of the pancreas is related to the hilum but does not overlie the right kidney. It is,
however, anterior to the left kidney. The transpyloric plane (L1) transects the pancreas obliquely,
passing through the midpoint of the neck, with most of the head below the plane, and most of
the body and tail above. The transverse mesocolon is attached to the head, neck and body of
the pancreas. The uncinate process lies posterior to the superior mesenteric vessels, and the
inferior mesenteric vein passes behind the body of the pancreas, where it joins the splenic vein.
High anal fistula
Incorrect
Are more common than low fistula TrueFalse
Incorrect
Open into the rectum above the puborectalis muscle TrueFalse
Incorrect
Are associated with Crohns disease TrueFalse
Incorrect
May be laid open without hazard TrueFalse
Incorrect
Can be managed by a loose seton TrueFalse
High fistulae are uncommon but may be due to carcinoma, diverticular disease, tuberculosis,
Crohn's disease, ulcerative colitis, trauma or radiotherapy. Laying open divides the sphincter and
produces incontinence.
Osteochondritis of the navicular bone
Incorrect
is thought to be due to increase vascularity producing early calcification TrueFalse
Incorrect
mainly affects teenagers TrueFalse
Incorrect
usually resolves spontaneously in a year TrueFalse
Incorrect
usually presents with pain and a limp TrueFalse
Incorrect
treatment involves analgesia and continued activity TrueFalse
Osteochondritis of the navicular bone is known as Khlers disease and affects children age 35
years. They complain of pain over the medial side of the foot and noticeably limp. It is thought to
be due to a disturbance of the blood supply. Normally symptoms disappear after a few weeks of
strapping the foot and restricting activity, but, rest in a cast may be necessary if there is severe
pain. Eventually the foot becomes normal clinically and radiologically over a period of months.
Which of the following are correct? Umbilical hernia in children and infants
Incorrect
Occur through the umbilical cicatrix TrueFalse
Incorrect
Is an example of a sliding hernia TrueFalse
Incorrect
Can be treated with a corset TrueFalse
Incorrect
Has a higher incidence in black than white children TrueFalse

Incorrect
Can be treated with a Mayo repair TrueFalse
An umbilical hernia protrudes through the umbilical cicatrix to lie in the subcutaneous tissues.
They will often resolve as the child grows and few will require surgical treatmentthose that do
can be repaired with the Mayo vest-over-pants approach.
The surface of the right lung is indented by the
Incorrect
trachea TrueFalse
Incorrect
oesophagus TrueFalse
Incorrect
superior vena cava TrueFalse
Incorrect
right ventricle TrueFalse
Incorrect
subclavian vein TrueFalse
Impressions on the mediastinal surface of the right lung include the trachea, vagus, superior
vena cava, right atrium and subclavian artery. The oesophagus grooves the left lung above the
arch of the aorta and below the hilum.
For Le Fort I fractures, which of the following are true
Incorrect
Bilateral is more common than unilateral Le Fort I fractures TrueFalse
Incorrect
The fracture line passes above the palate TrueFalse
Incorrect
Is associated with Le Fort II injury TrueFalse
Incorrect
Intercanthal distance is usually increased TrueFalse
Incorrect
Airway should be protected with an oropharyngeal tube TrueFalse
The French surgeon Le Fort performed experiments on cadavers in early 1900s and classified
facial fractures into I, II and III. In Le Fort I injuries the fracture line passes above the palate,
fracturing the pyrimadal processes of the maxilla on each side, the vomer and the lower parts of
the pterygoid processes. Bilateral Le Fort I fractures represent 23% of these injuries, unilateral
11%. Le Fort I and II fractures occur together in 21% of cases. Le Fort II injuries involve the
ethmoids and so increase intercanthal distance. Nasopharyngeal intubation is preferred to
oropharyngeal intubation to secure the airway.
Which of the following are true? The hepatic portal vein
Incorrect
Is formed by the union of the splenic and superior mesenteric veins TrueFalse
Incorrect
Runs behind the epiploic foramen TrueFalse
Incorrect
Lies posterior to the common hepatic artery TrueFalse
Incorrect
Lies anterior to the first part of the duodenum TrueFalse
Incorrect
Forms posterior to the neck of the pancreas TrueFalse

The portal vein forms the anterior boundary of the epiploic foramen, lying behind the bile duct
and hepatic artery. It lies in front of the inferior vena cava, as it lies behind the pancreas and the
first part of the duodenum.
On the dorsum of the foot the
Incorrect
dorsalis pedis artery lies medial to the extensor hallucis longus tendon TrueFalse
Incorrect
deep peroneal nerve lies medial to the dorsalis pedis artery TrueFalse
Incorrect
L5 dermatome is present TrueFalse
Incorrect
great saphenous vein lies anterior to the medial malleolus TrueFalse
Incorrect
inferior extensor retinaculum loops under the medial longitudinal arch TrueFalse
The dorsalis pedis artery lies between the extensor hallucis longus tendon medially, and the
deep peroneal nerve lies laterally. The L5 dermatome lies over the medial half of the dorsum of
the foot. The great saphenous vein is found anterior to the medial malleolus, and the lower limb
of the extensor retinaculum passes under the medial longitudinal arch and blends with the
plantar aponeurosis.
Reply to HalaReport

Post #18
Hala Adel wrote5 hours ago
Which of the following are true? Differential diagnosis of a femoral hernia includes
Incorrect
Varicocele TrueFalse
Incorrect
Psoas abscess TrueFalse
Incorrect
Saphena varix TrueFalse
Incorrect
Troisiers node TrueFalse
Incorrect
Spigelian hernia TrueFalse
Differential diagnosis of a femoral hernia includes: inguinal hernia, saphena varix,
lymphadenopathy, psoas abscess, lipoma, femoral aneurysm, sarcoma, ectopic testes and
obturator hernia. Troisiers node is supraclavicular. Spigelian hernia arises from the semilunar
line.
Which of the following are correct? With regards to the anatomy of the pancreas
Incorrect
It lies along the transpyloric plane TrueFalse
Incorrect
Superior mesenteric vessels pass under the uncinate process TrueFalse
Incorrect
The inferior vena cava (IVC) is a posterior relation TrueFalse
Incorrect
The lesser sac is an anterior relation TrueFalse

Incorrect
The portal vein is formed behind the pancreatic neck TrueFalse
The pancreas has a head, neck, body and tail and lies along the transpyloric plane. The head is
bound laterally by the curved duodenum and the tail extends to the hilum of the spleen. The
superior mesenteric vessels pass behind the pancreas, then anteriorly, over the uncinate
process and third part of the duodenum into the root of the small bowel mesentery. The inferior
vena cava, coeliac plexus, left kidney, and the left adrenal gland are posterior pancreatic
relations.
Which of the following structures are parts of the hindbrain?
Incorrect
cerebellum TrueFalse
Incorrect
cerebral aqueduct TrueFalse
Incorrect
pons TrueFalse
Incorrect
basal ganglia TrueFalse
Incorrect
tectum TrueFalse
The gross structure of the brain can be divided into the forebrain, midbrain and hindbrain. The
main structures that form the hindbrain are the pons, medulla oblongata and cerebellum. The
fourth ventricle and central canal are also found in this region.
Concerning traumatic diaphragmatic injury, which of the following are true?
Incorrect
Is more commonly diagnosed on the left side TrueFalse
Incorrect
Blunt injuries produce larger tears than penetrating traumas TrueFalse
Incorrect
The commonest site for tears is the posterolateral aspect of the diaphragm TrueFalse
Incorrect
Insertion of a naso-gastric tube is absolutely contraindicated in left-sided diaphragmatic rupture
TrueFalse
Incorrect
Abdominal computed tomography is the most sensitive investigation to identify diaphragmatic
injury TrueFalse
Diaphragmatic injuries result from either blunt or penetrating trauma. A traumatic diaphragmatic
rupture is more commonly diagnosed on the left side, perhaps because the liver obliterates the
defect or protects it on the right side. In addition, the appearance of bowel, stomach or a
nasogastric (NG) tube is more easily detected in the left side of the chest. Right diaphragmatic
ruptures are rarely diagnosed in the early post-injury period. The liver often prevents herniation
of other abdominal organs into the chest. This, however, may not be representative of the true
incidence of laterality and autopsy studies have revealed that left- and right-sided ruptures
occur almost equally. Blunt trauma produces large radial tears measuring 515 cm, most often at
the posterolateral aspect of the diaphragm. In contrast, penetrating trauma usually create only
small linear incisions or perforations, which are less than 2 cm in size and may often take some
time, even years, to develop into diaphragmatic hernias. If a laceration of the left diaphragm is
suspected, a NG tube should be inserted. If the tube appears in the thoracic cavity on the chest
film, the need for special contrast studies can be eliminated. Minimally invasive endoscopic
procedures (thoracoscopy) may be helpful in evaluating the injury to the diaphragm in
indeterminate cases. Abdominal computed tomography scan is usually not helpful because of its
poor visualisation of the diaphragm. Magnetic resonance imaging is more accurate in visualising
the anatomy of the diaphragm. It is very sensitive and specific and so is the investigation of
choice. Surgical repair is necessary, even for small tears, because the defect will not heal
spontaneously.

The superior vena cava (SVC)


Incorrect
has a valve at its entry into the left atrium TrueFalse
Incorrect
drains only the head, neck and upper body TrueFalse
Incorrect
receives the thoracic duct TrueFalse
Incorrect
ends behind the second costal cartilage TrueFalse
Incorrect
enters the heart at the level of the sternal angle TrueFalse
The SVC drains all the structures above the diaphragm except the heart and lungs. It also
receives the azygos vein, which drains the lumbar and subcostal regions. The SVC is formed
behind the first costal cartilage by the union of the right and left brachiocephalic veins. It ends
behind the third costal cartilage as it enters the right atrium. The SVC has no valves. The
thoracic duct drains into the left brachiocephalic vein (or sometimes into the subclavian or
internal jugular vein).
In the surgical anatomy of the liver
Incorrect
segment I lies to the left of the portal vein TrueFalse
Incorrect
segment II lies medial to the porta hepatis TrueFalse
Incorrect
the caudate lobe lies anterior to the portal vein TrueFalse
Incorrect
the portal vein lies anterior to the Winslows foramen TrueFalse
Incorrect
three hepatic veins divide the liver into four sectors TrueFalse
The caudate lobe (segment I according to Couinauds 1957 classification) lies posterior to the
portal vein but anterior to the inferior vena cava. Three main hepatic veins divide the liver into
four sectors, each of which receives a portal pedicle, with an alternation between hepatic veins
and portal pedicles. According to this functional anatomy, the liver is divided into hemilivers
(right and left) by the main portal scissura called Cantlies line.
Nerves in direct contact with the humerus include:
Incorrect
median TrueFalse
Incorrect
radial TrueFalse
Incorrect
musculocutaneous TrueFalse
Incorrect
axillary TrueFalse
Incorrect
ulnar TrueFalse
There are three nerves that come into close contact with the humerus: The radial nerve contacts
the humerus in the spiral groove; the axillary at the surgical neck; and the ulnar at the medial
epicondyle.
Dupuytren's disease

Incorrect
is caused by contraction of the palmar fascia TrueFalse
Incorrect
is more common in Caucasians than in people of African origin TrueFalse
Incorrect
most commonly affects the little finger TrueFalse
Incorrect
causes contracture of the intrinsic muscles of the hand TrueFalse
Incorrect
may be associated with retroperitoneal fibrosis TrueFalse
Dupuytrens disease is a condition of unknown aetiology characterised by contraction of the
palmar or digital fascia. It affects 13% of the population of North Europe and the USA. It is rare
in the Far East and Africa. It is three times more common in males. Its incidence increases with
age. It has a strong hereditary disposition. The ring finger is the most commonly affected finger;
the little finger is the next most commonly affected digit.
Dupuytrens disease causes contracture of the intrinsic muscles of the hand leading to flexion of
the metacarpophalangeal joints and extension of the proximal interphalangeal joints the socalled intrinsic plus appearance. The following conditions are associated with Dupuytrens
disease:
knuckle pads (Garrods pads) penile fibrous plaques (Peyronies disease) plantar fibromatosis
(Ledderhose disease).
However, retroperitoneal fibrosis, however, is not associated with Dupuytrens disease.
A CT scan section through the manubriosternal joint will demonstrate
Incorrect
the bifurcation of the brachiocephalic artery TrueFalse
Incorrect
the commencement of the aortic arch TrueFalse
Incorrect
T4 vertebral body TrueFalse
Incorrect
the bifurcation of the trachea TrueFalse
Incorrect
the thoracic duct crossing the midline TrueFalse
A CT section at this level is at the level of T4. At this level, the arch of the aorta is commencing,
the azygos vein enters the superior vena cava (SVC), the left recurrent laryngeal nerve loops
round the ligamentum arteriosum and the bifurcation of the pulmonary trunk can be seen. The
thoracic duct crosses the midline at T5.
At the manubriosternal joint you have:
- bifurcation of the trachea
- start of the aortic arch
- azygous vein entering the superior vena cava
In the main bronchial airways
Incorrect
the left bronchus is longer than the right TrueFalse
Incorrect
the right main bronchus has a wider diameter than the left TrueFalse
Incorrect
aspiration pneumonitis is more common in the right lower lobe than the left TrueFalse

Incorrect
the left main bronchus divides before entering the lung TrueFalse
Incorrect
foreign bodies lodge more commonly in the right than in the left main bronchus TrueFalse
The right main bronchus is shorter (approximately 2.5 cm long), wider and runs more vertically
than the left main bronchus. The right main bronchus gives off the upper lobe branch (before
entering the lung) and passes inferior to the pulmonary artery before entering the hilum of the
lung (approximately T5). It is important to remember the azygos vein, which arches over the
right main bronchus from the posterior aspect as it passes to the SVC, and the pulmonary artery,
which lies inferior and then anterior to it. The left main bronchus is about 5 cm long and, unlike
the right, does not give off any branches before entering the hilum of the left lung at the level of
T6.
The abdominal inferior vena cava (IVC)
Incorrect
runs in the free edge of the lesser omentum TrueFalse
Incorrect
ascends to the right of the aorta TrueFalse
Incorrect
may be directly in contact with the right suprarenal gland TrueFalse
Incorrect
forms the posterior wall of the epiploic foramen TrueFalse
Incorrect
receives direct drainage from both the right and left suprarenal veins TrueFalse
The IVC commences opposite the L5 vertebra. It runs on the right side of the aorta, upwards
beyond the aortic opening of the diaphragm and extends to the central tendon of the
diaphragm, which it pierces at the level of T8. The IVC lies behind the portal vein near the
pancreas and bile duct, and forms the posterior wall of the epiploic foramen of Winslow. The
right vein of the suprarenal gland is usually only a few millimetres long and enters the IVC
directly. The left suprarenal vein is longer and enters the left renal vein.
The spleen:
Incorrect
May be injured by a stab wound over the left tenth rib TrueFalse
Incorrect
Is a retroperitoneal structure TrueFalse
Incorrect
Venous drainage from the spleen passes into the portal system TrueFalse
Incorrect
The splenic artery passes to the splenic hilum in the gastrosplenic ligament TrueFalse
Incorrect
May be absent and replaced with multiple splenunculi TrueFalse
The spleen lies in the concavity of the left hemidiaphragm with its long axis lying along the tenth
rib. It is fully invested in peritoneum, making it a peritoneal structure. The arterial supply is the
splenic artery, which reaches the spleen via the lienorenal ligament. Venous drainage is via the
splenic vein into the portal vein. Occasionally the spleen may be replaced with multiple
splenunculi.
Trench foot
Incorrect
is caused by acute exposure to temperatures between 8C and 2C TrueFalse
Incorrect
is due to microvascular endothelial damage and vascular occlusion TrueFalse

Incorrect
appears black even in the absence of deeper tissue destruction TrueFalse
Incorrect
is characterised by pruritic, redpurple lesions TrueFalse
Incorrect
can lead to gangrene TrueFalse
Reply to HalaReport

Post #19
Hala Adel wrote5 hours ago
Trench foot or cold immersion foot (or hand) is caused due to a non-freezing injury of the hands
or feet. This is typically seen in soldiers, sailors, or fishermen, who are chronically exposed to
wet conditions and temperatures just above freezing, i.e. 1.6C to 10C (35F to 50F). It occurs
due to microvascular endothelial damage, stasis and vascular occlusion. Although the entire foot
may appear black, deep tissue destruction may not be present. An alternating arterial
vasospasm and vasodilatation occurs, with the affected tissue first cold and anaesthetic,
progressing to hyperaemia in 24 to 48 h. This then leads to an intense painful burning and
dysaesthesia, as well as tissue damage characterised by oedema, blistering, redness,
ecchymosis, and ulcerations. Pruritic, redpurple skin lesions are a feature of chilblain or pernio.
Complications of trench foot include local infection, cellulitis, lymphagitis, and gangrene.
The following statements concern neuronal tracts in the spinal cord:
Incorrect
the fasciculus gracilis forms part of the dorsal column TrueFalse
Incorrect
the lateral corticospinal tracts control skilled voluntary movements TrueFalse
Incorrect
fine touch and proprioception are carried by crossed ascending neurones TrueFalse
Incorrect
the fasiculus cuneatus mainly carries sensory neurones from the lower limb TrueFalse
Incorrect
the lateral spinothalamic tracts conduct pain and temperature sensation TrueFalse
The dorsal columns are white matter tracts formed by the fasciculus gracilis and fasciculus
cuneatus. The fasciculus gracilis lies medial to the fasciculus cuneatus. They carry fine touch
and proprioception. Uncrossed fibres are added from medial to lateral as the cord is ascended.
Therefore the fasciculus gracilis mainly carries sensory fibres from the lower limbs, and the
fasciculus cuneatus carries those from the upper limbs. The lateral spinothalamic tracts carry
crossed axons conducting pain and temperature sensations.
Consider the hindgut
Incorrect
The hindgut structures include the distal one-third of the transverse colon, the descending colon,
sigmoid colon, rectum, and anal canal to the pectinate line TrueFalse
Incorrect
Branches of the inferior mesenteric artery supply the hindgut TrueFalse
Incorrect
The descending colon is secondarily retroperitoneal TrueFalse
Incorrect
The rectum contains taeniae coli, haustrations, and omental appendages TrueFalse
Incorrect
Failure of neural crest cells to migrate into the hindgut results in Hirschsprungs disease
TrueFalse

The descending colon, the rectum, and the anal canal are secondarily retroperitoneal. The distal
one-third of the transverse colon and the sigmoid colon are peritoneal. The distal one-third of
transverse colon, descending and sigmoid colons contain taeniae coli, haustrations and omental
appendages. The sigmoidrectal junction marks the end of the taeniae coli, the haustrations, and
the omental appendices. Hirschsprungs disease is caused by a failure of neural crest cells either
to migrate into the hindgut or to differentiate into terminal parasympathetic ganglia in the walls
of the hindgut.
The parotid duct
Incorrect
is approximately 1 cm long TrueFalse
Incorrect
crosses the masseter TrueFalse
Incorrect
is compressed by the buccinator TrueFalse
Incorrect
conveys mainly mucous secretions TrueFalse
Incorrect
lies on the middle third of a line between the intertragic notch of the auricle and the midpoint of
the philtrum TrueFalse
The parotid duct is approximately 5 cm long. It crosses the masseter, turning around its anterior
border to pass through the buccal fat pad and pierce the buccinator. When intraoral pressure is
raised, the submucous part of the parotid duct is compressed by the buccinator. The parotid
gland is mainly a serous gland
When the right main bronchus is dissected, the following structures may be encountered
Incorrect
the right phrenic nerve TrueFalse
Incorrect
the right vagus nerve TrueFalse
Incorrect
the right recurrent laryngeal nerve TrueFalse
Incorrect
the hemiazygos nerve TrueFalse
Incorrect
the azygos vein TrueFalse
The vagus nerve lies just posterior to the right main bronchus and the azygos vein is at first
posterior and then arches over the bronchus. The phrenic nerve is anterior to the bronchus. The
right recurrent laryngeal nerve hooks around the right subclavian artery superior to the right
main bronchus.
The following structures are retroperitoneal:
Incorrect
Pancreas TrueFalse
Incorrect
Spleen TrueFalse
Incorrect
Liver TrueFalse
Incorrect
Abdominal aorta TrueFalse
Incorrect
Descending colon TrueFalse

Both liver and spleen are invested with peritoneum, whereas the other three structures are to be
found attached to the posterior abdominal wall covered by the peritoneum. Note that bleeding
from the liver and spleen will result in peritoneal blood, but pancreatic and aortic bleeding will
cause a retroperitoneal haematoma often manifesting as flank discoloration.
Transection of the anterior division of the mandibular nerve (CN Vc) in the infratemporal fossa
results in
Incorrect
ipsilateral paralysis of the buccinator muscle TrueFalse
Incorrect
dysphagia TrueFalse
Incorrect
ipsilateral anaesthesia of the mandibular teeth TrueFalse
Incorrect
deviation of the jaw to the side of the lesion on protrusion TrueFalse
Incorrect
ipsilateral anaesthesia of the mucosa of the oral vestibule TrueFalse
The anterior division of CN Vc has one sensory branch (the buccal nerve to the skin of the cheek
and mucosa of the vestibule). All other branches are motor to the muscles of mastication (the
masseter, temporalis and lateral pterygoid). The lower jaw teeth are supplied by the inferior
alveolar branch of the posterior division of CN Vc. The buccinator muscle is supplied by the VIIth
cranial nerve. Unopposed contraction of the contralateral lateral pterygoid muscle deviates the
jaw to the side of the lesion during protrusion. None of the muscles of the pharynx are supplied
by the anterior division of CN Vc, so dysphagia is not a feature of damage to this nerve.
Umbilical hernia
Incorrect
is more common in Caucasians than in other races TrueFalse
Incorrect
is commonly associated with hypothyroidism TrueFalse
Incorrect
must be operated on before two years of age TrueFalse
Incorrect
becomes obstructed in 2% of cases TrueFalse
Incorrect
is more likely to resolve spontaneously if small TrueFalse
Umbilical hernias are most frequently seen in low birthweight and black infants. Although it is a
feature of hypothyroidism, most infants with umbilical hernias do not have hypothyroidism. The
majority of hernias will disappear by one year of age, strangulation is rare, and operation is only
indicated if the hernia persists until age 34 years, causes symptoms, or becomes progressively
larger.
Which of the following muscles are found in the medial fascial compartment of the thigh?
Incorrect
Adductor brevis TrueFalse
Incorrect
Sartorius TrueFalse
Incorrect
Obturator externus TrueFalse
Incorrect
Gracilis TrueFalse
Incorrect
Hamstring portion of adductor magnus TrueFalse

The medial fascial compartment of the thigh contains adductor longus, adductor brevis,
adductor portion of adductor magnus, gracilis and obturator externus. Sartorius lies in the
anterior fascial compartment and the hamstring portion of adductor magnus lies in the posterior
fascial compartment of the thigh.
A Kocher's incision
Incorrect
divides the Colles fascia TrueFalse
Incorrect
divides only the anterior rectus sheath TrueFalse
Incorrect
divides the external oblique muscle TrueFalse
Incorrect
involves the area innervated by T10 nerve root TrueFalse
Incorrect
divides the fascia transversalis muscle TrueFalse
Incorrect
divides the rectus abdominis muscle TrueFalse
Scarpas fascia is divided in a Kochers incision. T10 corresponds to the umbilical area. The
rectus sheath is deficient posteriorly only below the arcuate line, but is present in two layers in
the subcostal region.
Concerning the female breast:
Incorrect
the breast extends from the 2nd to the 5th rib TrueFalse
Incorrect
the breast extends from the lateral margin of the sternum to the mid-axillary line TrueFalse
Incorrect
the greater part of the gland lies in the deep fascia TrueFalse
Incorrect
the mammary gland is attached to the skin by Coopers ligaments TrueFalse
Incorrect
the mammary gland consists of 1520 lobes TrueFalse
The breast extends from the 2nd to the 6th rib, and from the lateral margin of the sternum to the
mid-axillary line. The greater part of the gland lies in the superficial fascia, and the gland is
attached to the skin by suspensory ligaments (Coopers ligaments). The mammary gland
consists of 1520 lobes, which radiate out from the nipple.
Direct branches of the coeliac plexus include the
Incorrect
splenic artery TrueFalse
Incorrect
common hepatic artery TrueFalse
Incorrect
superior pancreaticoduodenal artery TrueFalse
Incorrect
right gastric artery TrueFalse
Incorrect
gastroduodenal artery TrueFalse

As soon as the aorta passes below the aortic hiatus, it gives off the celiac plexus (T12). The
plexus has three direct branches: left gastric, hepatic and splenic (mnemonic: left-hand side).
The right gastric and gastroduodenal arteries are branches of the hepatic artery. The superior
pancreaticoduodenal artery is a branch of the gastroduodenal artery.
The right renal artery
Incorrect
branches several times before entering the kidney TrueFalse
Incorrect
gives a branch to the ureter TrueFalse
Incorrect
lies anterior to the renal vein TrueFalse
Incorrect
lies anterior to the inferior vena cava TrueFalse
Each renal artery usually divides into five segmental branches before entering the renal pelvis. It
supplies the ureter and lies posterior to the renal vein, but anterior to the renal pelvis. After
branching off the aorta, the renal artery passes behind the inferior vena cava.
At the level of the T4 vertebra, a computed tomography scan of the body transversely shows the
Incorrect
arch of aorta TrueFalse
Incorrect
bifurcation of trachea TrueFalse
Incorrect
left brachiocephalic vein TrueFalse
Incorrect
azygos vein TrueFalse
In the plane of Louis, the azygos vein arches over the hilum of the right lung before draining into
the superior vena cava.
Regarding fractures of the axis (C2) vertebra, which of the following are true?
Incorrect
Approximately 60% of all axis fractures involve the odontoid process TrueFalse
Incorrect
Type I odontoid fractures occur through the base of the dens TrueFalse
Incorrect
Type III odontoid fractures are the commonest type TrueFalse
Incorrect
Surgical fixation is indicted for type II odontoid fractures TrueFalse
Incorrect
Posterior element fractures are due to an extension type of injury TrueFalse
Acute fractures of the axis (C2) vertebra represent about 18% of all cervico-spinal injuries and
approximately 60% of axis fractures involve the odontoid process. The odontoid process is a
peg-shaped bony protuberance that projects upward and is normally positioned in contact with
the anterior arch of C1. It is held in place primarily by the transverse ligament. Type I odontoid
fractures involve the tip of the odontoid peg, type II fractures are through the base of the dens
(involving the junction of the odontoid peg with the body) and type III fractures occur at the base
of the dens and extend obliquely into the body of the axis. Odontoid fractures are initially
identified by a lateral cervico-spinal film or open-mouth odontoid views. In many cases,
however, a computed tomography (CT) scan is required to further delineate the type and extent
of the fracture. In children younger than 6 years of age, on plain radiography, the epiphysis may
be prominent and may look like a fracture at this level. Type II is the commonest type of
odontoid fractures. They require surgical reduction and immobilisation with a Halo and body

cast. If the fracture is not healed (and so unstable) at 12 weeks, posterior fusion of C1 to C2 may
be indicated. The posterior elements of C2, ie, the pars interarticularis may be fractured (a
hangmans fracture) by an extension type of injury. Patients with this fracture should be
maintained in external immobilisation until specialised care is available. These fractures
represent approximately 20% of all axis fractures.
For acute tongue swelling, which one of the following is true
Incorrect
Occurs as a side-effect of angiotensin-converting enzyme (ACE) inhibitors TrueFalse
Incorrect
Initial treatment is with beta-blockers TrueFalse
Incorrect
Airway obstruction is not a feature TrueFalse
Incorrect
Steroids are contraindicated TrueFalse
Incorrect
Antihistamines usually take 6 hours for full effect TrueFalse
Angio-neurotic oedema is tongue swelling secondary to ACE inhibitors. The most important initial
management is to secure the airway; a nasotracheal tube may be required. Steroids (which may
take 6 hours to take full effect) and antihistamine should be commenced immediately.
Crohn's disease
Incorrect
is typically a submucosal inflammation TrueFalse
Incorrect
is associated with rosethorn ulcers on barium studies TrueFalse
Incorrect
is associated with mouth ulcers TrueFalse
Incorrect
may lead to a patient requiring lifelong parenteral nutrition TrueFalse
Incorrect
most commonly affects the distal colon and then spreads proximally TrueFalse
Inflammation is classically described as transmural. Rosethorn ulcers are deep ulcers that
traverse beyond the lamina propria and have a characteristic appearance. Apthous ulcers occur
anywhere from the mouth to the anus. Short-bowel syndrome is why we try to avoid surgery
whenever possible with Crohns disease, but in very severe cases where less than 50 cm of small
bowel remains, malabsorption of essential fat soluble vitamins (A, D, E and K) as well as other
essential nutrients requires lifelong parenteral nutrition. Crohns colitis is not uncommon but the
small bowel is more often involved and usually seen by the time a colitis occurs. Crohns spreads
as skip lesions and, unlike ulcerative colitis, the spread is not usually in a continuous fashion.
Barretts oesophagus
Incorrect
occurs in approximately 40% of patients with gastro-oesophageal reflux disease (GORD)
TrueFalse
Incorrect
never produces ulceration TrueFalse
Incorrect
presents with malignancy in one-third of cases TrueFalse
Incorrect
produces strictures at the squamocolumnar junction TrueFalse
Incorrect
histamine H2 blockers are the treatment of choice TrueFalse

10% of patients with gastro-oesophageal reflux disease (GORD) will develop Barretts
oesophagus. It may ulcerate if left untreated, but medical treatment should be with proton pump
inhibitors. Strictures are common and may lead to the symptoms of dysphagia with which the
patient presents.
Which of the following muscles abduct the thigh?
Incorrect
Quadratus femoris TrueFalse
Incorrect
Gluteus maximus TrueFalse
Incorrect
Gluteus medius TrueFalse
Incorrect
Gluteus minimus TrueFalse
Incorrect
Piriformis TrueFalse
Both gluteus medius and gluteus minimus abduct and medially rotate the thigh at the hip joint.
Gluteus maximus extends and laterally rotates the thigh at the hip joint. Quadratus femoris and
piriformis both contribute to lateral rotation of the thigh.
Which of the following are true? Regarding the muscles of respiration:

Incorrect
Quiet inspiration is predominantly due to the action of the diaphragm
TrueFalse
Incorrect
Active inspiration is caused by the internal intercostal muscles
TrueFalse
Incorrect
The scalene muscles play an important role in quiet expiration
TrueFalse
Incorrect
Internal and external oblique muscles are important in active expiration TrueFalse
Incorrect
The external intercostal muscles pull the ribs medially and inferiorly during active expiration
TrueFalse
Reply to HalaReport

Post #20
Hala Adel wrote5 hours ago
The predominant muscle of inspiration during quiet breathing is the diaphragm, a dome-shaped
musculofibrous septum separating the thorax from the abdominal cavity. As the diaphragm
contracts, pleural pressure drops, which lowers the alveolar pressure. This draws air into the
lungs due to the pressure gradient from the mouth to the alveoli. Expiration during quiet
breathing is a passive process. This is caused by the relaxation of the respiratory muscles and
the return of the elastic lung and chest wall to their normal resting volume. During exertion or
activity, the external intercostals help in inspiration by raising the lower ribs superiorly and

laterally, so increasing the lateral and antero-posterior diameters of the thoracic cavity. The
scalene muscles and sternocleidomastoids also help by raising and pushing out the upper ribs
and the sternum. Active expiration is helped by the contraction of the abdominal wall muscles
(internal oblique, external oblique, transversus abdominus and rectus abdominus). It increases
the intra-abdominal pressure, which pushes up the diaphragm, so raising the pleural pressure
and drives the air out of the lungs. The internal intercostals also help in active expiration by
decreasing the thoracic volume (by pulling down medially and inferiorly).
The portal vein
Incorrect
is formed behind the body of the pancreas TrueFalse
Incorrect
lies anteriorly to the free edge of the lesser omentum TrueFalse
Incorrect
drains the spleen TrueFalse
Incorrect
forms the central vein of each liver lobule TrueFalse
Incorrect
lies to the right of the superior mesenteric artery TrueFalse
Incorrect
is about 10 cm in length TrueFalse
The portal vein is formed by the confluence of the superior mesenteric vein (lying to the right of
the artery) and the splenic vein, behind the neck of the pancreas. It is about 5 cm long. The
portal vein lies between the two layers of the lesser omentum, running almost vertically in the
free edge where the lesser omentum forms the anterior boundary of the epiploic foramen. The
terminals of the portal vein and the hepatic artery form, with the hepatic ductules, the triads of
the liver in the corners of the hexagonal lobules. The central veins drain into the hepatic veins.
The following could be appropriate management for a gunshot injury to the upper part of the
neck (above the angle of the mandible)
Incorrect
Arteriogram TrueFalse
Incorrect
Endoscopy TrueFalse
Incorrect
Barium swallow TrueFalse
Incorrect
Clinical observation TrueFalse
Incorrect
Surgical exploration TrueFalse
It is not appropriate to observe gunshot injuries: they need to be explored. An arteriogram may
be of benefit: vascular injury is the greatest concern here, and it may be possible to embolise a
bleeding vessel. This area is above the level of the trachea and oesophagus.
Colorectal carcinoma
Incorrect
is associated with a low-fibre, high-fat diet TrueFalse
Incorrect
presents with rectal mass in approx 45% of cases TrueFalse
Incorrect
is found in the rectum in 25% of cases TrueFalse
Incorrect
is commonly associated with Krukenbergs tumours TrueFalse

Incorrect
is treated by abdominal perineal resection as the surgical procedure of choice for tumours about
15 cm from the anal canal TrueFalse
Approximately 45% of colorectal tumours are found in the rectum. Krukenbergs tumours are
more commonly secondaries from gastric and ovarian cancer though can arise from colorectal
tumours. Abdominal perineal resections are used for low rectal tumours, where tumours are
about 8 cm from the anal canal.
In oesophageal disorders
Incorrect
patients with oesophageal achalasia are usually younger than those presenting with
oesophageal carcinoma TrueFalse
Incorrect
in oesophageal carcinoma, dysphagia is progressive unlike achalasia TrueFalse
Incorrect
oesophageal achalasia is treatable by dilating the lower oesophagus TrueFalse
Incorrect
oesophagitis due to Herpes simplex occurs in immuno-compromised patients TrueFalse
Incorrect
medical treatment of gastro-oesophageal reflux is successful in relieving regurgitation or
vomiting TrueFalse
In achalasia, patients have equal difficulty in swallowing both liquids and solids. In carcinoma,
difficulties begin with swallowing solids and progress to include liquids. In 95% of cases, only
surgery, if properly done, has any effect on curing regurgitation.
The recurrent laryngeal nerve
Incorrect
supplies the cricothyroid muscle TrueFalse
Incorrect
partially supplies the trachea TrueFalse
Incorrect
lies alongside the inferior thyroid artery TrueFalse
Incorrect
should be retracted during tracheostomy to avoid damage TrueFalse
Incorrect
runs between the oesophagus and trachea in the neck TrueFalse
Incorrect
supplies the mucous surface of the vocal cords TrueFalse
The recurrent laryngeal nerve supplies all the intrinsic muscles of the larynx except the
cricothyroid and is sensory inferior to the vocal folds. In the neck the recurrent laryngeal nerves
on both sides follow the same course, ascending in the tracheo-oesophageal groove. As the
nerve passes the lateral lobe of the thyroid it is closely related to the inferior thyroid artery. The
superior laryngeal nerve supplies the vocal cord mucosa.
Which of the following are correct? Underwater - sealed chest drains
Incorrect
Avoid build-up of positive intrathoracic pressure in case of alveolar air leak TrueFalse
Incorrect
A 28 Fr drain is usually used to drain blood or pus TrueFalse
Incorrect
Does not get blocked TrueFalse
Incorrect

Reduces the risk of positive-pressure ventilation TrueFalse


Incorrect
Accurately measures blood loss TrueFalse
A chest drain is a conduit to remove air or fluid from the pleural cavity allowing re-expansion of
the underlying lung. Drainage occurs during expiration when pleural pressure is positive. Unless
it is an emergency situation, preprocedure chest X -ray should be performed. The drain is usually
inserted under local anaesthesia using aseptic technique into the 5th intercostal space in the
mid-axillary line, and over the upper border of the rib, to avoid intercostal vessels and nerves. A
large drain (28 Fr or above) should be used to drain blood or pus.
Benign prostatic hyperplasia (BPH)
Incorrect
mainly affects the peripheral zone TrueFalse
Incorrect
is a recognised cause of elevated serum prostate-specific antigen (PSA) TrueFalse
Incorrect
incidence is increased in males castrated before puberty TrueFalse
Incorrect
symptoms improve with oxybutynin TrueFalse
Incorrect
can be treated with 5-alpha reductase inhibitors TrueFalse
Benign prostatic hyperplasia (BPH) mainly affects the inner transitional zone. The outer
peripheral zone is usuallycompressed and feels smooth to digital rectal examination. Any
palpable nodule or irregularity should raise the possibility of malignancy. BPH seems to be an
androgen-driven disease. Castration prior to puberty seems to prevent the disease. Alpha
blockers cause relaxation of smooth muscles and improve symptoms, whereas anticholinergic
drugs could worsen symptoms and precipitate acute urinary retention.
What are the differences between the right and left lungs
Incorrect
The right lung has three lobes TrueFalse
Incorrect
The right lung is shorter than the left TrueFalse
Incorrect
The right lung is larger and heavier TrueFalse
Incorrect
The anterior margin of the right lung is straight, unlike that of the left lung TrueFalse
The right lung has three lobes, the left lung has two. The right lung is larger and heavier than the
left but it is also shorter and wider because the right dome of the diaphragm is higher and the
heart bulge more to the left. The anterior margin of the left lobe has the cardiac notch.
The following are recognised complications of a rolling hiatus hernia:
Incorrect
oesophagitis TrueFalse
Incorrect
gastric volvulus TrueFalse
Incorrect
inhalational pneumonia TrueFalse
Incorrect
intermittent dysphagia TrueFalse
Incorrect
gangrene TrueFalse

The majority of hiatus herniae are sliding or axial in nature, these are often asymptomatic but
are associated with oesophagitis, stricture formation, dysphagia, chronic anaemia and
inhalational pneumonitis. Rolling herniae or para-oesophageal hiatal herniae usually affect
elderly patients who present with intermittent dysphagia, pain after eating due to distension of
the intrathoracic part of the stomach, cardiac symptoms due to pressure effects on the heart,
and hiccough due to phrenic nerve irritation. Complications include incarceration, gangrene and
gastric volvulus.
Posterior hip dislocation
Incorrect
classically occurs when the hip is in the extended position TrueFalse
Incorrect
is a common injury TrueFalse
Incorrect
can occur with avascular necrosis TrueFalse
Incorrect
may include damage to the femoral nerve TrueFalse
Incorrect
is characterised by the leg being held flexed and medially rotated TrueFalse
Posterior hip dislocation is an uncommon injury often occurring when the hip is flexed e.g. a road
traffic accident. The two main complications are sciatic nerve damage and avascular necrosis.
Sciatic nerve damage occurs because the sciatic nerve lies in close proximity to the posterior
aspect of the joint capsule so is at risk in posterior dislocation. Avascular necrosis occurs due to
tearing of the joint capsule, causing a disturbance of the blood supply to the femoral head.
The trigeminal nerve
Incorrect
supplies the buccinator muscle TrueFalse
Incorrect
supplies the muscles of mastication TrueFalse
Incorrect
has ophthalmic and maxillary divisions, which are only sensory TrueFalse
Incorrect
is sensory to the temporomandibular joint TrueFalse
Incorrect
supplies sensation to the angle of the mandible TrueFalse
The trigeminal (V) nerve has sensory fibres to the greater part of the skin of the face, mucous
membranes of the mouth, nose and paranasal air sinuses. It provides motor innervation to the
muscles of mastication (temporalis, masseter, pterygoid). The buccinator muscle is supplied by
the facial nerve. The angle of the mandible is supplied by the great auricular nerve (C2C3).
Reply to HalaReport

Post #21
Hala Adel wrote5 hours ago
Regarding spinal cord syndromes, which of the following are true?
Incorrect
Central cord syndrome results from vascular compromise of the cord along the distribution of the
anterior spinal artery TrueFalse
Incorrect
Position and vibration sense are preserved in anterior cord syndrome TrueFalse
Incorrect

There is ipsilateral motor loss and contralateral loss of pain and temperature sensation in Brown
Sequard syndrome TrueFalse
Incorrect
The arms are more affected than the legs in central cord syndrome TrueFalse
Incorrect
Anterior cord syndrome has the best prognosis among all incomplete spinal injuries TrueFalse
The central cord syndrome is thought to be due to vascular compromise of the cord in the
distribution of the anterior spinal artery. Infarction of the cord in the territory of this artery could
also result in the anterior cord syndrome. Anterior cord syndrome is characterised by paraplegia
and a dissociated sensory loss with loss of pain and temperature sensation. Position, vibration
and deep pressure sensations, all functions of the posterior column, are preserved. Brown
Sequard syndrome, resulting from hemisection of the cord, usually causes ipsilateral motor loss
and contralateral loss of pain and temperature sensation; there is also associated loss of position
sense. The central cord syndrome is characterised by a disproportionately greater loss of motor
power in the upper extremities than in the lower extremities, with varying degrees of sensory
loss. The arms and hands are most severely affected since the motor fibres to the cervical
segments are topographically arranged toward the centre of the cord. Anterior cord syndrome
has the poorest prognosis of all incomplete spinal injuries.
Which of the following are posterior relations of the kidneys:
Incorrect
Psoas major TrueFalse
Incorrect
Subcostal nerve TrueFalse
Incorrect
Peritoneum TrueFalse
Incorrect
Diaphragm TrueFalse
Incorrect
Pleura TrueFalse
The kidneys are retroperitoneal therefore the peritoneum is an anterior relation. Posterior
relations of the kidneys include the diaphragm, quadratus lumborum, psoas major, subcostal
vein, subcostal artery, subcostal nerve and ilioinguinal nerve. The costodiaphragmatic recess of
the pleura is an important posterior relation of the kidney as it can be inadvertently damaged
during nephrectomy resulting in a pneumothorax.
The femoral canal
Incorrect
allows for expansion of the femoral vessels TrueFalse
Incorrect
is a clinically important site of herniation of the small bowel TrueFalse
Incorrect
contains Cloquets node TrueFalse
Incorrect
contains the femoral artery TrueFalse
Incorrect
has the femoral vein lying medially TrueFalse
The femoral sheath is a fascial tube derived from extraperitoneal intra-abdominal fascia. It
extends under the inguinal ligament to surround the femoral vessels. The canal is a small space
between the medial part of the sheath and the femoral vein. It contains fat and Cloquets node.
Femoral hernias can be differentiated from inguinal hernias by locating the neck of a femoral
hernia below and lateral to the inguinal canal.
Compared with the lower end of the ileum, the upper end of the jejunum has
Incorrect

a thicker wall TrueFalse


Incorrect
less fat at the mesenteric border TrueFalse
Incorrect
fewer circular folds TrueFalse
Incorrect
a wider lumen TrueFalse
Incorrect
more aggregated lymphatic follicles (Peyers patches) TrueFalse
Incorrect
more arterial arcades TrueFalse
The jejunum has a thicker wall, less mesenteric fat, more plicae circulares, a wider lumen, fewer
Peyers patches and fewer arterial arcades than the ileum.
The 1st rib
Incorrect
has scalenus anterior muscle inserted onto the scalene tubercle TrueFalse
Incorrect
has the subclavian vein overlying the vertebral transverse processes TrueFalse
Incorrect
has the subclavian vein running lateral to the artery TrueFalse
Incorrect
is related to the pleura TrueFalse
Incorrect
is related to the cervicothoracic (stellate) sympathetic ganglion TrueFalse
Incorrect
is related to the upper two roots of the brachial plexus TrueFalse
The 1st rib has the scalenus anterior muscle attached to the scalene tubercle, separating the
subclavian vein (anteriorly) from the artery (posteriorly).
The right suprarenal gland
Incorrect
lies against the bare area of the liver TrueFalse
Incorrect
extends behind the inferior vena cava (IVC) TrueFalse
Incorrect
receives blood from the right inferior phrenic artery TrueFalse
Incorrect
drains into the right renal vein TrueFalse
Incorrect
lies on the ninth rib TrueFalse
The bare area of the liver is in direct contact with the right suprarenal gland and the diaphragm.
The right suprarenal gland extends medially behind the IVC, separated from the 12th rib by the
diaphragm. It typically has three arterial sources. It receives blood from the inferior phrenic
artery, from a branch of the renal artery and from a branch directly from the aorta. The venous
drainage is into the IVC by a very short vessel. The left suprarenal gland drains into its
corresponding renal vein.
Midline swellings of the neck include
Incorrect

cystic hygromas TrueFalse


Incorrect
plunging ranulae TrueFalse
Incorrect
subhyoid bursae TrueFalse
Incorrect
branchial cysts TrueFalse
Incorrect
arteriovenous fistulae TrueFalse
Swellings of the neck considered to be midline include thyroglossal cysts, pharyngeal pouches,
plunging ranulae, subhyoid bursae, laryngoceles and lesions in the thyroid isthmus.
Stimulation of the parasympathetic nervous system leads to
Incorrect
pupillary constriction TrueFalse
Incorrect
increased heart rate TrueFalse
Incorrect
smooth muscle relaxation TrueFalse
Incorrect
increased glandular secretion TrueFalse
Incorrect
decreased force of contraction of the heart TrueFals
Reply to HalaReport

Post #22
Hala Adel wrote5 hours ago
MRCS Part 1 Practice Questions ( Anatomy ) - 2 of 2
Here's the second part, Good Luck!
Intervertebral disc collapse between L5 and S1
Incorrect
would crush the L5 spinal nerve TrueFalse
Incorrect
would impinge into the sacral segments of the cord TrueFalse
Incorrect
usually causes pain to radiate over the medial malleolus TrueFalse
Incorrect
would exaggerate the tendon reflex at the ankle TrueFalse
Incorrect
may cause reduced sweating over the posterior aspect of the calf TrueFalse
A collapsed L5S1 disc presses on the S1 spinal nerve (the L5 nerve passes above the prolapsed
disc in the intervertebral foramen and so escapes damage). At the level of prolapse, the spinal
canal contains the cauda equina and not cord per se. The S1 dermatome lies over the lateral
malleolus. Exaggerated reflexes are diagnostic of an upper motor neurone lesion. The S2
dermatome occupies the posterior aspect of the calf.
The anal canal
Incorrect
lies below the levator ani muscle TrueFalse

Incorrect
has a longitudinal muscular coat TrueFalse
Incorrect
has a lymphatic drainage via the inguinal lymph nodes TrueFalse
Incorrect
has an external sphincter innervated by the pudendal nerve TrueFalse
Incorrect
possesses valves TrueFalse
The levator ani forms part of the deep external anal sphincter. The anal canal has no longitudinal
muscular coat. Lymph from the lower anal canal drains via the superficial inguinal nodes. The
entire anal sphincter is innervated by the inferior rectal branch of the pudendal nerve (S2S4).
The upper anal canal is thrown into vertical folds called anal columns. The anal valves are
formed by horizontal semilunar folds of mucous membrane joining adjacent columns at their
lower end. Anal valves are remnants of the proctodeal membrane. The anococcygeal body lies
between the anal canal and the coccyx.
The left brachiocephalic vein drains the
Incorrect
cervical vertebrae TrueFalse
Incorrect
bronchi TrueFalse
Incorrect
intercostal spaces TrueFalse
Incorrect
thoracic duct TrueFalse
Incorrect
thyroid gland TrueFalse
The left brachiocephalic vein drains blood from: the cervical vertebrae via both vertebral veins;
the thyroid gland by the inferior thyroid veins; the first left intercostal space via the left superior
intercostal veins; and all the anterior intercostal spaces by the anterior intercostal veins draining
into the internal thoracic veins. The thoracic duct enters the vein at its commencement behind
the left sternoclavicular joint. The bronchial veins drain into the azygos/hemiazygos systems.
Fracture of the following bones is rare and indicates high energy trauma which could be
associated with hidden severe injuries
Incorrect
Scapula TrueFalse
Incorrect
Clavicle TrueFalse
Incorrect
Sternum TrueFalse
Incorrect
10th rib TrueFalse
Incorrect
1st rib TrueFalse
Incorrect
Shaft of humerus TrueFalse
These bones are difficult to break, the finding of such an injury is worrisome, and a further
pathology must be sought.
The tongue

Incorrect
receives sensory innervation from the vagus nerve TrueFalse
Incorrect
protrudes to the side of a unilateral lower motor neurone lesion TrueFalse
Incorrect
is active during the voluntary phase of swallowing TrueFalse
Incorrect
is retracted by the hyoglossus muscle TrueFalse
Incorrect
contains lymphoid tissue TrueFalse
Incorrect
has intrinsic muscles that are not attached to any bone TrueFalse
The sensory innervation to the tongue is from the VIIth, VIIIth and IXth cranial nerves. The
tongue deviates to the side of a XIIth cranial nerve lesion on protrusion, is active during the first
stage of swallowing and contains the lingual tonsil in the dorsum of its posterior third. The
tongue is retracted up and back by the styloglossus muscle, protruded by genioglossus and
depressed by the hyoglossus.
The phrenic nerves
Incorrect
are sensory to the peritoneum TrueFalse
Incorrect
transmit afferent fibres from the mediastinal pleura TrueFalse
Incorrect
receive sensory fibres from the lungs TrueFalse
Incorrect
supply the bronchi TrueFalse
Incorrect
pass anterior to scalenus anterior muscles TrueFalse
The phrenic nerve arises from the spinal cord segments C35 and lie in front of the scalenus
anterior muscle, passing between the subclavian vein anteriorly and the subclavian artery
posteriorly. It crosses over the lateral surfaces of the mediastinal structures passing in front of
the lung root to reach the diaphragm. The phrenic nerve supplies motor fibres to the diaphragm
and carries sensory fibres from the diaphragmatic peritoneum, mediastinal pleura and the
parietal pericardium. The bronchi and lungs themselves are supplied by branches of the
autonomic nerves, principally via the pulmonary plexuses.
Reply to HalaReport

Post #23
Hala Adel wrote5 hours ago
Lymphatic ducts
Incorrect
contract due to filling TrueFalse
Incorrect
have no valves TrueFalse
Incorrect
if obstructed, lead to lymphoedema TrueFalse
Incorrect
have a parasympathetic innervation TrueFalse
Incorrect

empty by pump action of the calf muscles TrueFalse


Incorrect
dilate in oedema TrueFalse
The function of lymphatic vessels is to return the plasma capillary filtrate to the circulation. This
task is achieved by increased tissue pressure, facilitated by intermittent skeletal muscle activity,
contractions of lymphatic vessels and an extensive system of one-way valves. Lymphoedema is
an accumulation of tissue fluid resulting from a fault in the lymphatic system very often,
patients are diagnosed as having lymphoedema when the oedema is due to another cause.
Lymphoedema can occur as a result of lymphatic obstruction secondary to infiltration of lymph
nodes, frequently deep in the pelvis.
Transection of the cervical part of the sympathetic chain at the root of the neck results in
Incorrect
vasomotor changes in the arm TrueFalse
Incorrect
ptosis TrueFalse
Incorrect
pupillary dilatation TrueFalse
Incorrect
ablation of sympathetic supply to the pulmonary plexus TrueFalse
Incorrect
loss of sweating over the C4 dermatome TrueFalse
In Horners syndrome there is: ptosis, pupillary constriction and occasional enophthalmos, and
dryness and flushing of the skin of the head and neck. The sympathetic supply to the lungs is
preserved as this originates below the lesion directly from the T1T4 ganglia of the sympathetic
chain. Sympathetic fibres pass to the arm via grey rami from the middle and inferior cervical
sympathetic ganglia through all the roots of the brachial plexus.
The musculocutaneous nerve
Incorrect
supplies skin over the shoulder TrueFalse
Incorrect
supplies the biceps brachii TrueFalse
Incorrect
becomes the lateral cutaneous nerve of the forearm (lateral antebrachial cutaneous nerve)
TrueFalse
Incorrect
supplies the muscles of the anterior aspect of the arm TrueFalse
Incorrect
supplies skin on the medial aspect of the arm TrueFalse

The musculocutaneous nerve supplies the coracobrachialis, biceps brachii and brachialis
muscles. It pierces the deep fascia just proximal to the elbow joint and becomes superficial. It is
then called the lateral cutaneous nerve of the forearm, supplying skin on the lateral aspect of
the arm.
Abdominal aortic aneurysms (AAAs)
Incorrect
are due to diabetes mellitus in most cases TrueFalse
Incorrect
expand at 10 mm per year TrueFalse

Incorrect
are inflammatory in 7% of cases TrueFalse
Incorrect
measuring 7 cm in diameter have a 5 year rupture rate of 40% TrueFalse
Abdominal aortic aneurysms (AAAs) are caused by atherosclerosis in most cases. They expand
at approximately 4 mm per year. The 5-year rupture rate for aneurysms measuring 7 cm is
approximately 75%.
Inspiration involves
Incorrect
descent of the hemidiaphragms TrueFalse
Incorrect
reduction of the vertical dimension of the chest TrueFalse
Incorrect
upward/forward movement of the first rib TrueFalse
Incorrect
contraction of the intercostal muscles TrueFalse
Incorrect
the long thoracic nerve of Bell (supplying the serratus anterior) TrueFalse
The vertical dimension of the chest increases on inspiration. The ribs move upwards and
outwards. However, the first rib does not move during respiration. The serratus anterior
(supplied by the long thoracic nerve) is involved in respiration.
The following statements concern the ankle joint
Incorrect
The deltoid ligament is attached to the lateral malleolus TrueFalse
Incorrect
The body and articular surface of the talus is wider anteriorly than posteriorly TrueFalse
Incorrect
The ankle is most stable in plantarflexion TrueFalse
Incorrect
The capsular ligaments in front of and behind the ankle joint are weak TrueFalse
Incorrect
The intermalleolar distance increases as the ankle plantarflexes TrueFalse
The ankle (tibiotalar) joint is a hinge joint. As with most hinge joints there is strong support at
the sides but not in front and behind. The deltoid ligament is attached above to the medial
malleolus and fans out to attach below, mainly on the talus, but also on the calcaneus. On the
lateral side there are three smaller ligaments (anterior and posterior talofibular ligaments and
calcaneofibular ligament). The ankle joint is most stable in dorsiflexion. The intermalleolar
distance increases in dorsiflexion due to the increased width of the anterior part of the talus
bone.
Lower limb amputation
Incorrect
through the knee affords the best rehabilitation TrueFalse
Incorrect
above the knee usually heals when equal anterior and posterior flaps are used TrueFalse
Incorrect
below the knee is classically performed with equal flaps TrueFalse
Incorrect
may be performed using a skew flap technique below the knee TrueFalse

Incorrect
using Syme's technique is the operation of choice in patients with peripheral vascular disease
TrueFalse
The best amputation which affords good rehabilitation is a below the knee procedure preserving
the joint. Classically, for a below knee amputation (BKA), a long posterior flap is fashioned
containing muscle and vessels, which is then folded over the base to form an even stump. More
recently, skew flaps have been introduced to make use of areas of tissue where the blood supply
is optimal. Above knee procedures usually heal when equal anterior and posterior flaps are used.
Generally, Syme's amputation should not be used in patients with peripheral vascular disease,
and one-third are subsequently revised to a higher level because of poor healing, ulceration or
poor function.
The processus vaginalis
Incorrect
is formed by visceral peritoneum TrueFalse
Incorrect
forms a sac in which the testis descends through the inguinal canal TrueFalse
Incorrect
when present in adults, predisposes to direct inguinal hernia TrueFalse
Incorrect
forms the tunica vaginalis in the adult TrueFalse
Incorrect
invests the adult vas deferens TrueFalse
The processus vaginalis is a parietal peritoneal sac which passes through the internal ring of the
inguinal canal in the fetus, but which is normally obliterated after birth except for a small part
that becomes the tunica vaginalis of the testis. The testis descends through the canal as a
retroperitoneal structure and is therefore outside and behind the processus vaginalis. In cases of
a persistent processus vaginalis, indirect inguinal hernias can ensue.
Carpal tunnel syndrome is associated with
Incorrect
Pregnancy TrueFalse
Incorrect
Diabetes mellitus TrueFalse
Incorrect
Bennetts fracture TrueFalse
Incorrect
Hypothyroidism TrueFalse
Incorrect
Golfers elbow TrueFalse

Reply to HalaReport

Post #24
Hala Adel wrote5 hours ago
Carpal tunnel syndrome is associated with rheumatoid arthritis, myxoedema, nephrotic
syndrome, acromegaly, multiple myeloma, amyloidosis, diabetes mellitus, alcoholism,
haemophilia, pregnancy, gout, wrist fractures and the menopause. A Bennetts fracture is a
fracture of the first metacarpal and therefore does not affect the wrist.
The nerve roots
Incorrect
of the ulnar nerve are C8, T1, and sometimes C7 TrueFalse

Incorrect
of the musculocutaneous nerve are C5C6 TrueFalse
Incorrect
of the axillary nerve are C5C8 TrueFalse
Incorrect
of the radial nerve are C5C8, and T1 TrueFalse
Incorrect
of the long thoracic nerve are C5C7 TrueFalse
The nerve roots of the musculocutaneous nerve are C5C7, and those of the axillary nerve are
C5 and C6. The nerve roots of the median nerve are C5C8, and T1.
In the base of the skull the:
Incorrect
foramen magnum transmits the basilar artery TrueFalse
Incorrect
foramen spinosum transmits the VIIth cranial nerve (CN VII) TrueFalse
Incorrect
foramen rotundum transmits the maxillary nerve TrueFalse
Incorrect
foramen ovale transmits the greater petrosal nerve TrueFalse
Incorrect
foramen lacerum transmits the mandibular nerve TrueFalse
The foramen magnum transmits the vertebral arteries which unite at the lower border of the
pons to form the basilar artery.
The foramen spinosum transmits the middle meningeal vessels and the meningeal branch of the
mandibular nerve. The foramen rotundum contains the maxillary nerve. The foramen ovale
transmits the mandibular nerve, lesser petrosal nerve and accessory meningeal artery. The
foramen lacerum transmits the internal carotid and greater petrosal nerve, which leaves as a
nerve of the pterygoid canal.
In surgical anatomy of the thyroid gland
Incorrect
the thyroid gland has a definite, fine capsule TrueFalse
Incorrect
Berrys ligament connects the thyroid to the cricoid cartilage and upper trachea TrueFalse
Incorrect
the inferior parathyroid glands are more constant in position than the superior parathyroid
glands TrueFalse
Incorrect
the middle thyroid veins are more constant in position than the superior and inferior thyroid
veins TrueFalse
Incorrect
unilateral recurrent laryngeal nerve division results in the contralateral vocal cord lying in the
mid- or cadaveric position TrueFalse
The thyroid gland has a definite, fine capsule, which allows a capsular dissection to preserve the
recurrent laryngeal nerves. The superior parathyroid glands are more constant in position than
the inferior. Because of their embryological migration, the inferior glands may be situated among
the pretracheal lymph nodes or in the thymus as far as 10 cm from the thyroid. The middle
thyroid veins are the least constant of the thyroid veins. The superior veins drain into the
internal jugular vein; the inferior veins are very constant and drain into the brachiocephalic
veins; and the middle veins are very variable and often multiple. Unilateral recurrent laryngeal
nerve section results in the ipsilateral vocal cord lying motionless in the mid- or cadaveric

position. The voice is hoarse and weak. If both recurrent laryngeal nerves are divided, then the
glottic space is narrowed and stridor develops.
Recognised complications of endoscopic sphincterotomy during ERCP include
Incorrect
Acute pancreatitis TrueFalse
Incorrect
Gastrointestinal haemorrhage TrueFalse
Incorrect
Small bowel obstruction TrueFalse
Incorrect
Small bowel perforation TrueFalse
Incorrect
Enterocutaneous fistula TrueFalse
Division of the sphincter of Oddi with the sphincterotome may cause pancreatitis, duodenal
perforation or bleeding. Many patients have a transiently increased serum amylase but a few
develop true acute pancreatitis with pain and ultrasound evidence of pancreatitis.
The femoral triangle
Incorrect
contains the deep inguinal lymph nodes TrueFalse
Incorrect
is bounded by the inguinal ligament inferiorly TrueFalse
Incorrect
is bounded by sartorius laterally TrueFalse
Incorrect
has a floor formed by the fascia lata TrueFalse
Incorrect
contains the femoral nerve, artery and vein TrueFalse
The femoral triangle is bounded by the inguinal ligament superiorly, sartorius laterally and
adductor longus medially. Its floor is formed by the iliopsoas and pectineus. Its roof is formed by
the fascia lata. It contains the femoral vein, artery and nerve from medial to lateral and also
contains the deep inguinal nodes.
A surgeon dissecting behind the right main bronchus is likely to encounter the
Incorrect
vagus nerve TrueFalse
Incorrect
phrenic nerve TrueFalse
Incorrect
recurrent laryngeal nerve TrueFalse
Incorrect
hemiazygos vein TrueFalse
Incorrect
azygos vein TrueFalse
The azygos vein arches superiorly over the right bronchus. The vagus nerve lies just posterior to
the right main bronchus, whereas the phrenic nerve is anterior.
Complete rectal prolapse
Incorrect
Involves all layers of the rectal wall TrueFalse

Incorrect
Is common in infants TrueFalse
Incorrect
Is commoner in men TrueFalse
Incorrect
May be complicated by rectal gangrene TrueFalse
Incorrect
May be treated by DeLormes procedure TrueFalse
Partial rectal prolapse occurs in children. Complete rectal prolapse is a disease of elderly women.
DeLormes procedure (excision of the mucosal component of the prolapse and plication of the
muscle from below) has a low morbidity but a high incidence of incontinence and recurrence.
Open or laparoscopic rectopexy is the procedure of choice.
Regarding massive haemothorax, which of the following are true?
Incorrect
Results from a rapid accumulation of more than 1500 ml of blood in the chest cavity TrueFalse
Incorrect
Is commonly associated with distended neck veins TrueFalse
Incorrect
Results in a dull percussion note over the affected side hemi-thorax TrueFalse
Incorrect
Immediate evacuation of 1500 ml of blood is an indication for early thoracotomy TrueFalse
Incorrect
Emergency thoracotomy is necessary in about 80% of patients TrueFalse
A haemothorax may result from a blunt (deceleration injury) or penetrating injury (disruption of
the systemic or hilar vessels) to the thoracic cavity. Massive haemothorax results from the rapid
accumulation of more than 1500 ml of blood or one-third or more of the patients blood volume
in the chest cavity. Distension of neck veins is rare; they are usually flat secondary to severe
hypovolemia. Rarely will the mechanical effects of massive intrathoracic blood shift the
mediastinum enough to cause distended neck veins or a tracheal shift. The neck veins, however,
may be distended if there is an associated tension pneumothorax. The important signs and
symptoms of a massive haemothorax include hypoxia, chest pain, decreased chest expansion,
absence of breath sounds on the affected side and percussion dullness over the affected
hemithorax. Chest tube placement to decompress the chest cavity, along with simultaneous
restoration of blood volume, is the first step in the management of massive traumatic
haemothorax. Blood from the chest tube should be collected in a device suitable for autotransfusion. If 1500 ml is immediately evacuated, it is highly likely that the patient will require an
early thoracotomy. In addition, patients who have an initial volume output of less than 1500 ml
but continue to bleed (200 ml/h for 24 h) also require a thoracotomy. The decision should be
made in such patients based on their physiological status rather than the rate of continuing
blood loss. The majority of the patients can be managed conservatively with appropriate fluid
resuscitation and chest decompression. Emergency thoracotomy is required in only about 10% of
patients with massive haemothorax.
Reply to HalaReport

Post #25
Hala Adel wrote5 hours ago
Consider the male external genitalia
Incorrect
The corpora carvenosa forms the glans penis TrueFalse
Incorrect
The corpus spongiosum is a continuation of the crura of the penis TrueFalse
Incorrect
The paraurethral glands of Littr are adjacent to the penile urethra and function to lubricate the

penile urethra TrueFalse


Incorrect
Bucks fascia encloses the three erectile bodies of the penis TrueFalse
Incorrect
Fibromatosis of Bucks fascia may cause Peyronie disease TrueFalse
The corpus spongiosum is situated at the ventral part of the penis, is a continuation of the bulb
of the penis and forms the glans penis. The two corpora cavernosa are extensions of the crura
and are situated on the dorsal aspect of the penis. Bucks fascia encloses the erectile bodies, the
dorsal arteries and veins, and the dorsal nerves of the penis.
Varicose veins
Incorrect
are most common in patients of Mediterranean origin TrueFalse
Incorrect
are more common in patients engaged in occupations involving long periods of standing
TrueFalse
Incorrect
seen in pregnancy tend to regress after parturition TrueFalse
Incorrect
may be associated with a previous history of deep vein thrombosis TrueFalse
Varicose veins affect females of Northern European origin five times more commonly than men
and are particularly associated with previous deep vein thrombosis (DVT). Veins are most
pronounced in patients who stand for long periods.
Which of the following are correct? The right atrium
Incorrect
Is separated externally by the crista terminalis TrueFalse
Incorrect
Is separated internally by the sulcus terminalis TrueFalse
Incorrect
The crista terminalis extends between the two vena caval orifices. TrueFalse
Incorrect
It contains the fossa ovalis in its anterolateral wall TrueFalse
Incorrect
The opening of the coronary sinus contains a valve TrueFalse
The two parts of the right atrium are separated externally by a groove on the posterior aspect of
the atrium known as the sulcus terminalis and internally by the crista terminalis, which extends
between the two vena caval orifices. The fossa ovalis is found on the interatrial septum, which
forms the posteromedial wall of the right atrium. The opening of the coronary sinus is guarded
by a semicircular valve that closes the orifice during contraction of the right atrium.
Which of the following are correct? Parotid neoplasms
Incorrect
Are commonly bilateral TrueFalse
Incorrect
Are malignant in 50% TrueFalse
Incorrect
Presentation with facial nerve palsy implies malignancy TrueFalse
Incorrect
Usually present with maxillary numbness TrueFalse
Incorrect

Magnetic resonance imaging (MRI) rather than computed tomography (CT) is the radiological
investigation of choice for staging of parotid carcinomas TrueFalse
Parotid neoplasms are usually unilateral, only 1520% of parotid tumours are malignant. The
main mode of presentation is a symptom-less swelling often dating back several years.
Presentation with facial nerve palsy is very suggestive of a malignancy. Rare presentations
include trismus and referred pain via the trigeminal nerve. MRI is used for radiological staging as
there is better soft tissue discrimination, imaging can be carried out in multiple planes and it is
easier to detect cervical lymphadenopathy.
Which of the following are correct? Cardiac surgery
Incorrect
Is performed with controlled hypertension TrueFalse
Incorrect
May be complicated by cardiac tamponade TrueFalse
Incorrect
Rarely results in postoperative arrhythmias TrueFalse
Incorrect
Always requires cardio-pulmonary bypass TrueFalse
Incorrect
Routine coronary artery bypass grafting (CABG) is associated with a mortality rate of 6%
TrueFalse
Cardiac surgery is performed with controlled hypotension, together with hypothermia. The
hypothermia is used to decrease cellular metabolism and reduce energy requirements of the
tissues. Cardiac tamponade is a well known complication of cardiac surgery, it usually presents
in the early post-operative period with deteriorating cardiac function and cardiac arrest. CABG
can sometimes be performed without cardiopulmonary bypass (off pump). Arrhythmias (usually
atrial fibrillation) occur in 2040% of patients following surgery. Routine CABG is associated with
a mortality rate of around 2%.
The external jugular vein
Incorrect
receives a branch from the retromandibular vein TrueFalse
Incorrect
lies anterior to scalenus anterior TrueFalse
Incorrect
joins the subclavian vein TrueFalse
Incorrect
has no valves TrueFalse
Incorrect
pierces the deep cervical fascia TrueFalse
The external jugular vein drains most of the scalp and side of the face. It begins near the angle
of the mandible and is formed from the union of retromandibular and postauricular veins,
receiving branches from the posterior external and transverse cervical veins. The external
jugular vein has two pairs of valves which do not prevent regurgitation of the blood, or the
passage of injection from below upward. The lower pair are placed at its entrance to the
subclavian vein, the upper (in most cases) about 4cm above the clavicle. The external jugular
vein lies anterior to scalenus anterior and pierces the deep fascia of the neck, usually posterior
to the clavicular head of the sternocleidomastoid muscle before draining into the subclavian
vein.
Which one of the following is correct? The axillary artery
Incorrect
Gives off no branches in its first part TrueFalse
Incorrect
Is the continuation of the external carotid TrueFalse

Incorrect
Is encompassed by the first digitation of serratus anterior TrueFalse
Incorrect
Is invested in a fascial sheath TrueFalse
The first part of the axillary artery gives off one branch the superior thoracic. The axillary artery
is the continuation of the subclavian. The upper part of serratus anterior lies posterior to the first
part of the axillary artery. It is invested in a fascial sheath arising from the prevertebral fascia.
Regarding traumatic aortic disruption, which of the following are true?
Incorrect
Immediate death may be prevented by pseudoaneurysm contained by the adventitial layer
TrueFalse
Incorrect
Could lead to deviation of the trachea to the right TrueFalse
Incorrect
The aortic knob may be obliterated in plain radiography TrueFalse
Incorrect
Transoesophageal echocardiography is the gold standard in the diagnosis of this condition
TrueFalse
Incorrect
In an unstable patient, helical contrast-enhanced computed tomographic (CT) scan is the most
appropriate first-line investigation TrueFalse
Traumatic aortic disruption, a time-sensitive injury, is a common cause of sudden death after an
automobile collision or a fall from great height. A complete tear through the tunica intima, media
and adventitia usually leads to rapid exsanguination and death. In aortic rupture survivors,
immediate death is prevented due to the vascular continuity maintained by a pseudoaneurysm
within an intact adventitial layer or a mediastinal haematoma. A large mediastinal haematoma
may shift the trachea to the right. This condition has a variable course ranging from a relatively
clinically silent period due to the contained rupture (pseudoaneurysm), to rupture of the
pseudoaneurysm, exsanguination and death. Radiographic findings may include a widened
mediastinum, obliteration of the aortic knob, deviation of the trachea to the right, obliteration of
the space between the pulmonary artery and the aorta (obscuration of AP (aorto-pulmonary)
window), depression of the left main stem bronchus, deviation of the oesophagus (nasogastric
tube) and fractures of the first or second rib or scapula. False-positive and false-negative
findings occur with each radiographic sign and, rarely (12%), no mediastinal or initial chest Xray abnormality is present in patients with great vessel injury. Although transoesophageal
echocardiography is a useful, less invasive diagnostic tool, aortography is the gold standard in
the diagnosis of blunt aortic rupture. Helical contrast-enhanced computed tomography (CT) of
the chest is also an accurate screening method for patients with suspected blunt aortic injury.
However, a patient who is haemodynamically abnormal should not be placed in a CT scanner. In
stable patients, if enhanced helical CT of the chest is negative for mediastinal haematoma and
aortic rupture, no further diagnostic imaging is necessary. If it is positive for blunt aortic rupture,
the extent of the injury can best be ascertained by aortography
The lesser omentum
Incorrect
is supplied by gastroepiploic arteries TrueFalse
Incorrect
is attached to the liver in the fissure of the ligamentum venosum TrueFalse
Incorrect
encloses the right gastric vessels TrueFalse
Incorrect
has the common hepatic bile duct in its free edge TrueFalse
Incorrect
is attached to the first part of the duodenum TrueFalse

Incorrect
has considerable mobility TrueFalse
The right and left gastric arteries supply the lesser omentum as they lie between its two
peritoneal layers. The free edge of the lesser omentum is attached to the first 2 cm of the first
part of the duodenum below and the fissure of the ligamentum venosum above. The common
hepatic duct is joined by the cystic duct to form the common bile duct in the free edge of the
lesser omentum. The greater omentum is quite mobile.
The basilic vein
Incorrect
begins on the medial side of the dorsal venous arch TrueFalse
Incorrect
drains into the subclavian vein TrueFalse
Incorrect
is accompanied by the medial cutaneous nerve of the forearm TrueFalse
Incorrect
pierces the deep fascia in the arm TrueFalse
Incorrect
lies medial to the biceps tendon in the cubital fossa TrueFalse
The basilic vein is a continuation of the ulnar stem of the dorsal venous arch in the hand. It lies
medial to the biceps tendon in the cubital fossa and is medial to the medial cutaneous nerve of
the forearm in the arm. The basilic vein ascends in the superficial fascia on the medial side of
the biceps. It then pierces the deep fascia in the middle of the upper arm, is joined by the venae
comitantes of the brachial artery, and becomes the axillary vein at the lower border of the teres
major muscle.
Reply to HalaReport

Post #26
Hala Adel wrote5 hours ago
The diaphragm
Incorrect
contracts during forced expiration TrueFalse
Incorrect
is partially supplied with motor fibres by the intercostal nerves TrueFalse
Incorrect
has its central tendon pierced by the inferior vena cava TrueFalse
Incorrect
has its costal components derived mainly from the septum transversum TrueFalse
Incorrect
has its left dome at a higher level than its right dome TrueFalse
The central part of the diaphragm is mainly derived from the septum transversum, whereas its
periphery has body wall components. The intercostal nerves provide sensory fibres to the
peripheral parts of the diaphragm, but the phrenic nerves supply all the motor fibres. The
diaphragm contracts during inspiration but relaxes during expiration and is displaced upwards by
raised intra-abdominal pressure. The caval hiatus lies in the central tendon, whereas the
oesophageal hiatus is surrounded by muscle fibres of the diaphragmatic crura.
Squamous-cell carcinoma of the oral cavity
Incorrect
Are more common on the Asian subcontinent TrueFalse
Incorrect
Cervical lymph node involvement is usually treated with radiotherapy TrueFalse

Incorrect
Leucoplakia is premalignant TrueFalse
Incorrect
Radiotherapy is usually not required after complete surgical excision TrueFalse
Incorrect
Does not occur in non-smokers TrueFalse
Squamous cell carcinoma (SCC) of the mouth is related to betel nut chewing, common in Asia.
Lymph node metastasis is treated with radical neck dissection, usually followed by radiotherapy.
Similarly primary excision is usually followed by radiotherapy. Leucoplakia is a risk factor,
especially if associated with severe epithelial dysplasia. Due to the genetic basis of cancer,
anyone is at risk; however it is rare in non-smokers.
The ophthalmic artery
Incorrect
is a branch of the internal carotid artery TrueFalse
Incorrect
enters the orbit through the superior orbital fissure TrueFalse
Incorrect
supplies the ethmoidal air sinuses TrueFalse
Incorrect
supplies the cornea TrueFalse
Incorrect
supplies the skin of the forehead TrueFalse
The ophthalmic artery is a branch of the internal carotid artery. It passes through the optic canal
and supplies the ethmoidal air cells, part of the lateral wall of the nose, external nose, eyelids
and forehead. It also supplies all the muscles of the orbit.
Which of the following are correct? Medial ligament of the ankle
Incorrect
Has three separate bands TrueFalse
Incorrect
The deep part is longest TrueFalse
Incorrect
Inserts into the calcaneum TrueFalse
Incorrect
Has a superficial part TrueFalse
Incorrect
Can be seen on X-ray analysis TrueFalse
The medial, or deltoid ligament of the ankle is attached at its apex to the medial malleolus.
Below, the deep fibres are attached to the non-articular area on the body of the talus. The
superficial fibres are fan shaped and extends to the tuberosity of the navicular, the spring
ligament, the sustentaculum tali and the posterior tubercle of the talus.
Which of the following are correct? Indication for resection of a lung tumour
Incorrect
Is precluded by a malignant pleural effusion TrueFalse
Incorrect
Is contraindicated if ipsilateral hilar lymph nodes are involved TrueFalse
Incorrect
Is curative in 60% of squamous carcinoma TrueFalse

Incorrect
Includes palliation TrueFalse
Incorrect
Includes small cell carcinoma TrueFalse
A lung tumour must be sufficiently localised to be suitable for resection. Involvement of
ipsilateral hilar lymph nodes is not usually a contraindication to resection but the presence of
malignant cells in a pleural effusion is. The 5-year survival rate following complete resection of
non-small-cell lung cancer is in the region of 3040%. Small-cell lung cancers (also known as oatcell carcinoma) are highly malignant tumours that are usually disseminated at presentation. For
the majority of patients chemotherapy is the treatment of choice, small-cell lung cancers are
rarely suitable for surgical management.
The transpyloric plane:
Incorrect
Lies mid-way between the jugular notch and the pubic symphysis TrueFalse
Incorrect
Lies at the level of the first lumbar vertebra TrueFalse
Incorrect
Defines the level at which the coeliac axis leaves the aorta TrueFalse
Incorrect
Is the plane where the portal vein is formed TrueFalse
Incorrect
Crosses the right costal margin at the tip of the ninth costal cartilage, the surface marking of the
gall-bladder fundus TrueFalse
The transpyloric plane is a convenient way to relate anatomical structures. It is an imaginary
transverse plane with a surface marking mid-way between the jugular notch and the pubic
symphysis. This corresponds to the level of the first lumbar vertebra. The surface marking of the
fundus of the gall-bladder is at its junction with the right costal cartilage. It represents the point
at which the superior mesenteric artery leaves the aorta and where the splenic and superior
mesenteric veins join to form the portal vein.
The medial ligament of the ankle
Incorrect
comprises three separate bands TrueFalse
Incorrect
is damaged in a sprained ankle TrueFalse
Incorrect
inserts into the calcaneum TrueFalse
Incorrect
has a superficial part TrueFalse
Incorrect
may be associated with an avulsion fracture on X-ray TrueFalse
The medial ligament of the ankle, otherwise known as the deltoid ligament, has two layers. The
deep part is narrow and much shorter than the superficial part, which is triangular in shape. The
superficial part of the medial ligament is attached to the borders of the tibial malleolus, and has
a continuous attachment from the medial tubercule of the talus along the edge of the
sustentaculum tali and spring ligament to the tuberosity of the navicular bone. The lateral
ligament consists of three separate bands, and it is this ligament which is usually damaged in
inversion injuries (a sprain) of the ankle. The ligaments themselves cannot be seen on X-ray,
although avulsion fractures may be detectable on X-ray.
Portal hypertension may be caused by
Incorrect
Pylephlebitis after acute appendicitis TrueFalse

Incorrect
Splenectomy TrueFalse
Incorrect
Tricuspid valve incompetence TrueFalse
Incorrect
Alcoholic cirrhosis TrueFalse
Incorrect
BuddChiari syndrome TrueFalse
Portal hypertension with a pressure of over 20 mmHg is commonly caused by prehepatic
problems such as portal vein thrombosis, hepatic disease such as cirrhosis and post hepatic
problems such as tricuspid valve incompetence and BuddChiari syndrome of hepatic vein
thrombosis.
Which of the following are true? The right suprarenal gland
Incorrect
Has a longer vein than the left suprarenal gland TrueFalse
Incorrect
Receives an arterial supply directly from the aorta TrueFalse
Incorrect
Is crescentic in shape TrueFalse
Incorrect
Touches the bare area of the liver TrueFalse
Incorrect
Lies anterior to the inferior vena cava TrueFalse
The left suprarenal vein is longer than the right, entering the left renal vein. Both glands receive
an arterial supply directly from the aorta, as well as from the renal and inferior phrenic arteries.
The right suprarenal gland is pyramidal in shape; the left is crescentic in shape. The anterior
surface of the right suprarenal gland is overlapped medially by the inferior vena cava.
Structures superficial to the sternocleidomastoid muscle include the
Incorrect
transverse cervical nerve TrueFalse
Incorrect
transverse cervical artery TrueFalse
Incorrect
great auricular nerve TrueFalse
Incorrect
external jugular vein TrueFalse
Incorrect
inferior thyroid artery TrueFalse
The transverse cervical nerve emerges as a single trunk behind the posterior border of the
sternocleidomastoid and is superficial to the muscle. The transverse cervical artery is found in
the posterior triangle of the neck just above the clavicle. The great auricular nerve (C2C3) is a
large trunk that passes vertically upwards over the sternocleidomastoid. The external jugular
vein commences behind the angle of the mandible, formed by the union of the posterior
auricular vein and the posterior division of the retromandibular vein. It descends obliquely across
to the sternocleidomastoid and drains into the subclavian vein.
For basal cell carcinoma (BCC) of the face, which one of the following is true
Incorrect
Is the commonest malignant facial skin tumour TrueFalse
Incorrect

Basal cell carcinomas commonly metastasise TrueFalse


Incorrect
Radiotherapy is the current treatment of choice for basal cell carcinomas TrueFalse
Incorrect
Small BCCs (less than 1 cm) should be excised with a maximum margin of 5 mm to improve
cosmetic appearance TrueFalse
Incorrect
Aggressive recurrence may occur with BCCs on the forehead if the disease is not eradicated
sufficiently. TrueFalse
BCC is the commonest skin tumour on the face and 90% of all BCCs occur in the head and neck
region. Sunlight exposure and genetic factors are the main risk factors. BCCs rarely, if ever,
metastasise and treatment is with complete surgical excision. Radiotherapy is reserved for
recurrences, which are typically aggressive if they recur on the cheeks, nasolabial folds, medial
canso and preauricular region. The phrase rather a large scar than a small tomb (Sir Harold
Gillies) should always be taken into consideration when planning surgical excision tumours
under 1 cm should have at least a 5-mm excision margin and those over 1 cm should have at
least a 1-cm margin.
In the investigation of varicose veins
Incorrect
a negative A sound using Doppler ultrasound (U/S) signifies valvular incompetence TrueFalse
Incorrect
Doppler U/S has a sensitivity of up to 80% TrueFalse
Incorrect
ascending phlebography is the most valuable technique for identifying perforators TrueFalse
Incorrect
varicography has little value in the assessment of vessels with unusual anatomy TrueFalse
In Doppler ultrasound (U/S), the A sound is produced by squeezing the calf. When pressure is
released there is no sound if the valves are competent, this is termed a positive A sound.
Doppler U/S has a sensitivity of up to 97%; 87% of perforating veins are localised by ascending
phlebography.
The true vocal folds are
Incorrect
lined by respiratory epithelium TrueFalse
Incorrect
formed by the lower free edge of the quadrangular membranes TrueFalse
Incorrect
abducted by the lateral cricoarytenoid muscles TrueFalse
Incorrect
abducted by the posterior cricoarytenoid muscles TrueFalse
Incorrect
tensed by contractions of the cricothyroid muscles TrueFalse
Incorrect
innervated by sensory fibres of the internal laryngeal nerves TrueFalse
The true vocal folds have a stratified squamous epithelium, innervated by the recurrent
laryngeal branch (CN X), and are formed by the vocal ligament (the free edge of the
quadrangular membrane forms the false vocal cord). Above the vocal cords, the larynx is
sensorily innervated by the internal laryngeal nerve (CN X). The cords are adducted by the
lateral cricoarytenoid muscle, abducted by the posterior cricoarytenoid and tensed by tilting the
thyroid cartilage downwards and forwards by contracting the cricothyroid muscle. All the
laryngeal muscles are supplied by the recurrent laryngeal nerve except for cricothyroid, which is
supplied by the external laryngeal nerve.

Which of the following are true? The parotid duct


Incorrect
Is approximately 1 cm long TrueFalse
Incorrect
Crosses the masseter TrueFalse
Incorrect
Is compressed by buccinator TrueFalse
Incorrect
Conveys mainly mucous secretions TrueFalse
Incorrect
Lies on the middle third of a line between the intertragic notch of the auricle and the mid-point
of the philtrum TrueFalse
The parotid duct is approximately 5 cm long. It crosses the masseter, turning around its anterior
border to pass through the buccal fat pad and pierce the buccinator. When intraoral pressure is
raised the submucous part of the parotid duct is compressed by the buccinator. The parotid
gland is mainly a serous gland.
Reply to HalaReport

Post #27
Nezar Mohamed wrote5 hours ago


Reply to NezarReport

Post #28
Hala Adel wrote5 hours ago
An upper midline laparotomy involves incising:
Incorrect
The linea alba TrueFalse
Incorrect
The rectus abdominis TrueFalse
Incorrect
The transversus abdominis TrueFalse
Incorrect
The transversalis fascia TrueFalse
Incorrect
The visceral peritoneum TrueFalse
A midline laparotomy is not a muscle cutting incision. The incision passes through the skin,
subcutaneous flat, the linea alba, extraperitoneal fat, transversalis fascia and parietal
peritoneum.
The (common) bile duct
Incorrect
lies to the left of the hepatic artery in the lesser omentum TrueFalse
Incorrect
is supplied by the cystic artery TrueFalse
Incorrect
usually opens into the duodenum separately from the pancreatic duct TrueFalse

Incorrect
crosses in front of the neck of the pancreas TrueFalse
Incorrect
passes anterior to the right renal vein TrueFalse
The common bile duct lies to the right of the hepatic artery. It joins the pancreatic duct at the
ampulla of Vater. The ampulla itself usually opens into the duodenum. It crosses a groove
between the head of the pancreas and the second part of the duodenum, in front of the right
renal vein.
The facial nerve
Incorrect
pierces buccinator muscle TrueFalse
Incorrect
is the main supplier of the muscles of mastication TrueFalse
Incorrect
is involved in taste TrueFalse
Incorrect
includes the submandibular branch as one of the three main divisions originating from within the
parotid gland TrueFalse
Incorrect
emerges from the skull through the stylomastoid foramen TrueFalse
The buccal branch of the nerve does pierce buccinator muscle after supplying it and the muscles
of the upper lip. The trigeminal (V) nerve is predominantly associated with the muscles of
mastication. These include the masseter, temporalis and pterygoid muscles. There are five, not
three, main divisions of the nerve originating from the parotid gland, and the lowest or fifth
branch is the cervical. The facial nerve does pass through the stylomastoid foramen
A severely displaced Colles fracture
Incorrect
may lead to delayed rupture of the extensor pollicis longus tendon TrueFalse
Incorrect
has the distal radius displaced in a volar direction TrueFalse
Incorrect
usually requires 6 weeks immobilisation TrueFalse
Incorrect
more commonly requires external fixation in the older rather than the younger patient TrueFalse
Incorrect
can be associated with Sudeks atrophy TrueFalse
Rupture of the extensor pollicis longus tendon may occur as a late complication of a displaced
Colles fracture. A Smiths fracture is the reverse Colles fracture where the distal segment is
palmar flexed rather than dorsiflexed. Most Colles fractures are treated in plaster for 6 weeks,
but in young patients it may be necessary to restore normal alignment by internal fixation,
especially when cosmetic appearances or type of occupation may be adversely affected by
residual deformity or loss of movement. Sudeks atrophy can follow trivial hand injuries and is
thought to be related to autonomic dysfunction.
About the knee
Incorrect
the popliteal muscle is intracapsular TrueFalse
Incorrect
the medial longitudinal ligament is attached to the medial meniscus TrueFalse
Incorrect

the menisci are covered in synovial membrane TrueFalse


Incorrect
the anterior cruciate ligament is attached to the medial condyle TrueFalse
Incorrect
the posterior cruciate ligament is stretched when the knee is in full extension TrueFalse
* The popliteal tendon is intracapsular.
* The medial collateral ligament is attached to the medial left meniscus, which is thus less
mobile than the lateral meniscus.
* There is no synovial membrane over the articulating surfaces of synovial joints.
* The anterior cruciate ligament is attached to the lateral femoral condyle.
* Both cruciate ligaments are tense in full extension.
Consider the hip joint
Incorrect
it is a hinge joint TrueFalse
Incorrect
a thick and tight fibrous capsule increases stability TrueFalse
Incorrect
the quadratus femoris is a lateral rotator of the hip TrueFalse
Incorrect
the iliofemoral ligament prevents overextension of the hip joint TrueFalse
Incorrect
the rim of the acetabular labrum increases hip stability TrueFalse
The hip joint is a ball and socket synovial joint. The capsule is thicker and tighter than that of the
shoulder joint. The acetabular labrum encloses the femoral head beyond its equator, increasing
the stability of the joint.
Which of these muscles are commonly innervated by the obturator nerve?
Incorrect
Gracilis TrueFalse
Incorrect
Semimembranosus TrueFalse
Incorrect
Pectineus TrueFalse
Incorrect
Adductor longus TrueFalse
Incorrect
Obturator internus TrueFalse
The obturator nerve (L2, L3, L4) exits the obturator foramen and divides into anterior and
posterior branches. The anterior branch innervates adductor brevis, adductor longus, gracilis
and pectineus. The posterior branch innervates obturator externus and part of adductor magnus.
Semimembranosus is one of the hamstring muscles and is innervated by the tibial portion of the
sciatic nerve. Obturator internus is innervated by the nerve to obturator internus, which, also
supplies the superior glamellus.
Concerning the axillary artery:
Incorrect
it begins at the medial border of the first rib TrueFalse
Incorrect
it ends at the inferior border of the teres major TrueFalse
Incorrect
the pectoralis minor divides the axillary artery into three parts TrueFalse

Incorrect
the first part of the axillary artery gives off the thoracoacromial artery TrueFalse
Incorrect
the lateral thoracic artery branches from the second part of the axillary artery and is larger in
men than in women TrueFalse
The axillary artery begins at the lateral border of the first rib as the continuation of the
subclavian artery, and ends at the inferior border of the teres major. For descriptive purposes the
axillary artery is divided into three parts by the pectoralis minor. The first part has one branch
(superior thoracic), the second part gives off two branches (thoracoacromial and lateral
thoracic), and the third part gives off three branches (subscapular, anterior circumflex humeral,
and posterior circumflex humeral). The lateral thoracic artery is larger in women, and it supplies
the lateral part of the mammary gland.
The recurrent laryngeal nerve
Incorrect
supplies all intrinsic laryngeal muscles TrueFalse
Incorrect
supplies the cricothyroid muscle TrueFalse
Incorrect
supplies sensation to the subglottic region TrueFalse
Incorrect
is sensory to the supraglottic region TrueFalse
Incorrect
supplies the sternothyroid muscle TrueFalse
The recurrent laryngeal nerves are sensory to the subglottic region and supply all the intrinsic
muscles except the cricothyroid muscle.
Phaeochromocytomas:
Incorrect
are usually malignant TrueFalse
Incorrect
are associated with multiple endocrine neoplasia type 2a (MEN2a) TrueFalse
Incorrect
are rarely bilateral TrueFalse
Incorrect
are associated with multiple endocrine neoplasia type 1 (MEN1) TrueFalse
Incorrect
secrete aldosterone TrueFalse
Phaeochromocytomas are tumours of chromaffin tissue that secrete catecholamines. Patients
present with hypertension and sympathetic hyperactivity. 10% of such tumours are malignant,
10% are bilateral, 10% are extra-adrenal and 10% are familial. They are associated with MEN2a
and 2b.
Structures related to the superficial part of the submandibular gland include
Incorrect
platysma TrueFalse
Incorrect
the mandibular branch of the facial nerve TrueFalse
Incorrect
the facial artery TrueFalse

Incorrect
the facial vein TrueFalse
Incorrect
deep cervical fascia TrueFalse
The submandibular gland is a lobulated gland made up of a superficial and a deep part, which
are continuous with each other around the posterior border of the mylohyoid muscle. Part of the
gland lies inferolaterally, enclosed in an investing layer of deep cervical fascia, platysma muscle
and skin. Laterally it is crossed by the cervical branch of the facial nerve and vein. The facial
artery is related to the posterior and superior aspects of the superficial part of the gland.
Which of the following are correct? The following structures would be encountered during a
carotid endartectomy
Incorrect
Hypoglossal nerve TrueFalse
Incorrect
Omohyoid TrueFalse
Incorrect
Facial vein TrueFalse
Incorrect
Superior thyroid artery TrueFalse
Incorrect
Internal jugular vein TrueFalse
All of the above are near to the carotid arteries and would be expected to be encountered during
the course of a carotid endartectomy.
Which of the following are correct? Left coronary artery:
Incorrect
Divides into circumflex and left anterior descending arteries TrueFalse
Incorrect
Supplies the left atrium TrueFalse
Incorrect
Supplies the sinoatrial (SA) node in most cases TrueFalse
Incorrect
Supplies the atrioventricular (AV) node in most cases TrueFalse
Incorrect
Arises from the anterior aortic sinus TrueFalse
The left coronary artery arises from the left posterior aortic sinus behind the pulmonary trunk.
After a short course it divides into two main arteries, the circumflex and the left anterior
descending, otherwise known as the anterior interventricular artery. Around 60% of hearts have
the right coronary artery supplying the SA node and in 40% of hearts the SA nodal artery arises
from the left coronary artery. The right coronary artery supplies the atrioventricular (AV) node.
The left coronary artery supplies the vast majority of the left ventricle and left atrium. Part of the
right ventricle is supplied by the left coronary artery.
Bleeding from the middle meningeal artery following head injury
Incorrect
mainly affects the posterior branch TrueFalse
Incorrect
results in an extradural haematoma TrueFalse
Incorrect
may produce ipsilateral pupillary constriction TrueFalse

Incorrect
is usually caused by a trivial incident TrueFalse
Incorrect
typically causes a biconvex-shaped lesion on CT TrueFalse
Bleeding from the middle meningeal artery following head injury usually leads to an extradural
haematoma. This is usually a tear of the anterior branch of the middle meningeal artery, with an
underlying linear skull fracture. The characteristic picture is of a head injury with a brief episode
of unconsciousness followed by a lucid interval. The patient then develops a progressive
hemiparesis, stupor and rapid transtentorial coning with an ipsilateral dilated pupil. This is
followed by bilateral fixed dilated pupils, tetraplegia and death.
Which of the following are true? The inguinal canal
Incorrect
Is about 1.5 cm long TrueFalse
Incorrect
Has the fascia transversalis along the whole length of its posterior wall TrueFalse
Incorrect
Has a deep inguinal ring lying 5 cm above the middle of the inguinal ligament TrueFalse
Incorrect
Has the lacunar ligament in the medial part of its floor TrueFalse
Incorrect
Has the inferior epigastric artery medial to its deep ring TrueFalse
The inguinal canal is about 4 cm long. The posterior wall of the canal has the conjoint tendon
medially and the transversalis fascia throughout. The deep inguinal ring lies about 1.25 cm
above the mid-point of the inguinal ligament. The floor of the inguinal canal is the unrolled lower
edge of the inguinal ligament, re-inforced medially by the lacunar ligament.
Which of the following are true? Regarding the spinothalamic tract:
Incorrect
Conveys vibration and position sense to the brain
TrueFalse
Incorrect
The secondary axons of the tract synapse in the thalamus TrueFalse
Incorrect
Axons from the lumbar region synapse medially
TrueFalse
Incorrect
Lesion to this tract could lead to loss of pain sensation on the opposite side of the lesion
TrueFalse
Incorrect
Lesion to this tract could lead to diminished temperature and touch sensation from the same
side of the body as the lesion
TrueFalse
The spinothalamic tract conveys pain, temperature, touch and pressure sensations from one side
of the body to the opposite side of the brain. Vibration and position sense are conveyed via the
posterior column. The first neurone of the spinothalamic tract synapses in the posterior horn; the
next neurone crosses to the right side of the spinal cord and synapse in the thalamus, after
ascending through the cord and brainstem; the third neurone arises in the thalamus to pass to
the cortex. The secondary axons of the spinothalamic tract ascend through the brainstem to
synapse in the thalamus. Axons from the cervical region synapse medially while axons from the

lumbar region synapse laterally. A lesion of the spinothalamic tract anywhere in the brainstem
would lead to a loss of pain sensations from the opposite side of the body. Temperature and
touch sensations would also be diminished from the opposite side of the body but not totally lost
because other pathways may also convey these modalities. A lesion of the spinothalamic tract at
the level of the spinal cord would lead to loss of pain sensations on the opposite side, beginning
one level below the level of the lesion.
Reply to HalaReport

Post #29
Khalid Ayad (Egypt) wrote2 hours ago

Reply to KhalidReport

Post #30
Mostafa M.Safwat (Egypt) wroteabout an hour ago

MRCS Part 1

.... onexamination.com


60 ...
....
....

---------------------------- 2008
sba 500 mcq 1300
... ...
...
...
mihd rapidshare
mihd
request ticket mihd ..
): EgyMedicine.Net for free medical books (70,000 members
http://www.egymedicine.net/forumsx/t29406.html#post165201
...
IDM flashget dap
RESUME
40
3

...
..
http://rapidshare.de/files/39918888/Que.rar.html
http://rapidshare.de/files/40132484/Que_2.zip.html

http://mihd.net/4nweykf
)(
Reply to MostafaReport

Reply
Post reply

Advertise

Shoof Kora

More Ads
Facebook

Вам также может понравиться